X



スレッドを立てるまでもない質問スレッド Part 367
レス数が1000を超えています。これ以上書き込みはできません。
0001名無しさん@英語勉強中 (ワッチョイW 1386-t1Bu)
垢版 |
2021/07/23(金) 12:39:03.85ID:flwzF43T0
■ 質問する方へ
・常識的な内容なら何を質問してもOKです。
・できるだけ分りやすく、具体的に質問して下さい。ソースがあれば必ず書いて下さい。

■ 回答する方へ
・YAHOO!知恵袋のコピペを使った嫌がらせが後をたたないので、回答の前に、
 知恵袋に同じ質問が出ていないか、質問文の中のワードでググってみたほうが賢明です。
・回答する方は、質問者の立場になって答えてあげましょう。

■ 辞書
・単語や熟語の意味なら辞書で調べた方が速くて正確です。
 研究社 - ルミナス英和・和英辞典
 http://www.kenkyusha.co.jp/modules/08_luminous/index.php?content_id=1
 無料で引けるオンライン辞書(英英辞典) - おさんぽ英語 おすすめ散歩道
 https://osanpo-english.com/site-book/online-dictionary.html

■ ChMateなどのアプリを使っていて、音声入力で英語を書き込みたいときは:
・(Androidの場合) 書き込む→工具マーク→入力方式の切り替え→音声入力に切り替え 。
・(iOSの場合) キーボードを表示させて左下のマイクのアイコンをタップする 。

過去スレ
スレッドを立てるまでもない質問スレッド Part 366
https://lavender.5ch.net/test/read.cgi/english/1623049320/
スレッドを立てるまでもない質問スレッド Part 365
https://lavender.5ch.net/test/read.cgi/english/1619251629/
0002名無しさん@英語勉強中 (ワッチョイW 1386-t1Bu)
垢版 |
2021/07/23(金) 12:40:51.21ID:flwzF43T0
テンプレではありませんが過去スレで問題提起がありましたので転載します
(このスレ内でレスしている様に見えない様、アンカーのみ削除させていただきました)

881 名無しさん@英語勉強中 (オッペケ Sr03-crGe)[sage] 2020/10/12(月) 18:48:11.75 ID:BiNVjerEr
例文貼りまくり厨はマジ自重すべき
今このスレ、モバイルで開けらんない時あるぞ

924 名無しさん@英語勉強中 (ワッチョイ 9fa1-AMwI)[] 2020/10/15(木) 07:54:22.61 ID:oZ2vtNXk0
・同じ用法に関する例文は2つまで
・文法的な解説以外の、英語の新聞や雑誌の記事などを「同じ単語や表現が出てくるから参考になる」という理由で貼るのは避ける
0004名無しさん@英語勉強中 (ワッチョイ d99d-0CkY)
垢版 |
2021/07/23(金) 23:30:16.80ID:8pEQZzx00
この板の投稿数順位
最終更新日時:2021/07/23 23:15:00
本日の総投稿数: 224 順位: 412/1,028

224投稿じゃ書き込み屋以外の投稿はほぼゼロ
廃墟板と言っいい
0006名無しさん@英語勉強中 (スップ Sd73-FakV)
垢版 |
2021/07/24(土) 14:09:47.04ID:Y9ry4rSMd
credit 〜 for ・・・ 「・・・を〜のおかげだとする」
timing:動詞 time の動名詞形「〜のタイミングを計ること」
information reveals:情報を明らかにすること(revealは名詞)
0007名無しさん@英語勉強中 (THW 0H0b-0CSh)
垢版 |
2021/07/24(土) 22:28:35.53ID:u5JdshwcH
「誰に聞いたらいいですか?」というのは英語では
Who i should ask with?
でいいんでしょうか?

検索するとこう出てくるんですが、これだと「誰が聞くべきですか?」ということにはなりませんか?
0008名無しさん@英語勉強中 (ワッチョイ b368-0CkY)
垢版 |
2021/07/24(土) 23:27:14.65ID:y8GxPW2A0
文法問題で insist that節中の動詞の問題

He insisted that his patient ( ) in the bed for a week.
@ remains A would remain B has remained C should remain

答えはC 
He insisted that his patient should remain in the bed for a week.
彼は患者は1週間ベッドで安静にすべきだと主張した
(解説) insistが要求を主張する意味のときはinsist that節中の動詞は仮定法現在で should remain / remain

だけど、辞書には insist は事実を主張する意味のときは直説法だと書いてる。
それならAもありませんか?
He insisted that his patient would remain in the bed for a week.
彼は患者はベッドに1週間とどまりたいと言ったのだと主張した
あるいは
彼は患者はベッドに1週間とどまるだろうと主張した

これは間違ってますか?
0009名無しさん@英語勉強中 (ワッチョイ 395a-hZWg)
垢版 |
2021/07/24(土) 23:39:23.27ID:Wds7H55z0
>>8
「彼」って医者でしょ?
自分の受け持ちの患者が「だろう」はないんじゃないの?
入院期間なんて自分が決めることなんだから。
0010名無しさん@英語勉強中 (ワッチョイW c9e3-XNQa)
垢版 |
2021/07/25(日) 00:34:09.56ID:tgS72CHV0
河合塾で浪人してるんですが
英語はそれなりに得意で直近記述模試だと偏差値75でマークの模試はリーディング満点でした。

テキストの復習をしないといけないんですが、そんなに力入れなくてもいいと思いますか?
毎日何かしらの英文を読むくらいで大丈夫でしょうか?
英文解釈は難しいやつはかなり難しいんですが全体としてはそこまで難しくは感じませんでし
た。
逆にやらないとこれでいいのかと不安にもなります。
0012名無しさん@英語勉強中 (ワッチョイ 8b89-bSA8)
垢版 |
2021/07/25(日) 04:36:29.63ID:9Qb7UHVs0
>>8
>>He insisted that his patient ( ) in the bed for a week.

質問の趣旨から外れるけど、in the bed なんてこの場合には変でしょう?
in bed でしょうに。問題集に本当に the bed って書いてあったの?
0014名無しさん@英語勉強中 (ワッチョイW 99cb-3w5J)
垢版 |
2021/07/25(日) 09:37:49.11ID:MYZgxGmi0
医師国家試験の受験生が玉石混合と書いた2つの理由は、個人的な経験によります。
第一に、私の出身高校は某男子校の進学校ですが、医学部に進んだ人も少なくなく、同じグループで集まるメンバーにもいます。
彼らの内何人かは頭が良いなと思いましたが、ほとんどは普通です。中高で英語9年間学んで、ようやく英検2級レベルの頭です。
共通していたのは家が裕福であることや、彼らの両親の多くが医師であったこと。彼ら個人の能力は平凡でしたが、家の経済力は中の上以上あったでしょう。
第二に、父(大学教授)が某医師国家試験で100%合格率を継続している医学部で、週一で非常勤講師(理系の一般教養科目)をしていた経験からです。
本業の大学は週4であり、非常勤では年契約により様々な大学で週一で教えています。
父が感じたのは、医学部生の能力の平凡さです。大学の実名を挙げるのは避けますが、仮にも医師国家試験100%合格率なのに、その大学の学生の能力は、例えば早稲田理工などにも劣るのではないか、と感じたようです。
もちろん、旧帝大の医学部とかなら話は別で、学生は皆優秀でしょう。
0016名無しさん@英語勉強中 (ワッチョイ 8b89-bSA8)
垢版 |
2021/07/25(日) 11:35:32.96ID:9Qb7UHVs0
remain in THE bed
stay in THE bed
こういう用例は、確かに見つかるけど in bed に比べるとかなり少ないね。さらに、

(1)
https://forum.wordreference.com/threads/lie-in-the-bed.1385581/
(2)
https://english.stackexchange.com/questions/259398/what-is-the-difference-between-in-the-bed-and-in-bed

この上にあるリンク先の議論を読んでいても、やはり in THE bed と in bed とでは
使い方が違う。通常の睡眠や病気のせいで寝るときには、やはり in bed というふうに
the をつけないのが原則のように見える。the をつけると、一つの特定のベッドに
寝ることを意味したり、その他の特殊な場合に限られているように見える。
0017名無しさん@英語勉強中 (ワッチョイ 8b89-bSA8)
垢版 |
2021/07/25(日) 11:38:49.87ID:9Qb7UHVs0
in THE bed についての別の議論
If you ask someone "Where is she?" and you get the reply "She's in bed", it could mean
she's not well, but it could also mean that she was in her bed, resting or sleeping.
If you get the reply ★"She's in the bed", this would mean that she is in bed and there is only one bed.★
https://forum.wordreference.com/threads/in-bed-in-the-bed.2450344/
0020名無しさん@英語勉強中 (FAX! MMeb-Wmfy)
垢版 |
2021/07/26(月) 08:30:07.14ID:AP6pM6a7MFOX
バブルヘッドってのはググッたらそういう人形だって分かったけど
バブルヘッドナースっていうキャラがいますが首も繋がってるしのっぺらぼうっぽいけど
アレはどういう語源なのでしょうか?
0022名無しさん@英語勉強中 (ワッチョイ 8b89-bSA8)
垢版 |
2021/07/26(月) 13:25:43.03ID:jB3Vu7dB0
>>21
What's going to change going forward to convince you that now you're ready?

俺の考え:
(1) what --- O (目的語), 何を
(2) is going to change --- 述語動詞(って呼ばれるのかな?) , これから先に変わっていくのか?
(3) going forward to convince you that now you're ready --- これが主語
さらにこの先に進んで、今では君たちがその準備ができているということを君たちに
納得させること(は)

直訳:
ここからさらに先に進んで、君たちに対して、君たちが今では(レースを行う)準備が
できているということを納得させることは、何を変えることになるだろうか?

意訳:
さらにここから一歩進んで、君らがもうその準備ができてると納得させたら、
これからどうなっていくんだろうかな?(いい結果が生まれるだろうかな?)

正直を言って、これにはあまり自信がないけど、これ以外に考えようがないような
気もするんだよな。もっといい考えがあったら、誰か教えてくれ。
0023名無しさん@英語勉強中 (ワッチョイ 8b89-bSA8)
垢版 |
2021/07/26(月) 13:29:39.90ID:jB3Vu7dB0
>>21
What's going to change going forward to convince you that now you're ready?

おっと、もしかしたら
0024名無しさん@英語勉強中 (ワッチョイ 8b89-bSA8)
垢版 |
2021/07/26(月) 13:33:24.39ID:jB3Vu7dB0
>>21
What's going to change going forward to convince you that now you're ready?

おっと、もしかしたら going forward は一種の副詞句であり、later on とか in the future
みたいな意味合いかもしれん。そうなると to convince you that now you're ready だけが
主語だということになる。そうだとしたら、この原文の全体は次のようにパラフレーズ
できる。
What's going to change later on if we convince you that now you're ready?
0025名無しさん@英語勉強中 (ワッチョイ 8b89-bSA8)
垢版 |
2021/07/26(月) 13:36:15.80ID:jB3Vu7dB0
>>21
何でそのように俺が解釈したかというと、ネット上を見渡すと
What's going to change going forward?
What will change going forward?
という言い回しが頻繁に使われているように見えるからだ?
一種の set phrase なんじゃないかと思う。
0026名無しさん@英語勉強中 (ワッチョイ 09e2-MszO)
垢版 |
2021/07/26(月) 13:40:33.43ID:+ba+CY4T0
What's going to change, going forward, to convince you that now you're ready?

・going forward=ゆくゆくは、いずれは、将来は。
0027名無しさん@英語勉強中 (ワッチョイ 09e2-MszO)
垢版 |
2021/07/26(月) 13:42:57.47ID:+ba+CY4T0
What needs to change to make you happy?

みたいなのと同じ文かな。
0028名無しさん@英語勉強中 (ワッチョイ 8b89-bSA8)
垢版 |
2021/07/26(月) 13:43:21.82ID:jB3Vu7dB0
>>21
"to convince you that S V" が主語だと言ったけど、実際には What が主語であり、

「何ががこれから先に変わっていって、君たちに〜を納得させることになるだろうか?」

というのが直訳だな。
0029名無しさん@英語勉強中 (ワッチョイ 8b89-bSA8)
垢版 |
2021/07/26(月) 13:44:03.57ID:jB3Vu7dB0
>>27 とかぶった。
0030名無しさん@英語勉強中 (ワッチョイ 8b89-bSA8)
垢版 |
2021/07/26(月) 13:45:21.99ID:jB3Vu7dB0
>>28 を訂正

「★何が(どういうことが)★ これから先に変わっていって、
君たちに〜を納得させることになるだろうか?」
0031名無しさん@英語勉強中 (ワッチョイ 8b89-bSA8)
垢版 |
2021/07/26(月) 13:50:15.95ID:jB3Vu7dB0
>>21
What's going to change going forward to convince you that now you're ready?

意訳:
これから状況がどう変わっていって、君らがもうレースに出られる
んだってことを納得することになるんだろうな?
0035名無しさん@英語勉強中 (ワッチョイ 8b89-bSA8)
垢版 |
2021/07/27(火) 08:50:37.09ID:Th9bj7iq0
emotion を uncountable noun (mass noun) として使ったときと countable noun として
(しかも emotions と複数にして)使ったときとの意味合いの違いについては、
「ウィズダム英和、第4版」では不思議なことにあまり明確なことは書いていない。
しかし「ジーニアス英和、第5版」には、一応ははっきりした意味の違いがある
ことを示唆している。

念のために OED Online を引くと、つい1か月ほど前に改訂したばかりかもしれない
解説文や定義文が出ている。

************ 以下は引用 *************
emotion
b. As a mass noun: strong feelings, passion; (more generally) instinctive feeling as distinguished from reasoning or knowledge.
(この上の mass noun はもちろん、uncountable noun のこと。)
(用例は10例あるが、そのうち2例だけを示す。)
1958 L. Durrell Balthazar iv. 82
   He was delighted to hear some music and listened with emotion to the wild quasidas that the old man sang.
2008 Neurosci. & Biobehavioral Rev. 32 1207/1
   A core concept in the history of neuroscience and emotion.

This entry has been updated (OED Third Edition, June 2011;
most recently modified version published online ★June 2021★).

********* 引用、終わり ***************
0036名無しさん@英語勉強中 (ワッチョイ 8b89-bSA8)
垢版 |
2021/07/27(火) 09:28:50.52ID:Th9bj7iq0
emotion (as a mass noun) と an emotion, emotions との意味合いや使い方が違うことについては、
あちこちの辞書にも書いてあるし、例文を見ていてもその違いがわかるけど、
意味合いが違うということは collocation 的にも違うということに結び付くだろうな。
たとえば The 2020 Tokyo Olympics の slogan の場合は

(1) United by Emotion

だけど、これに似た collocation として辞書に載っているのは

(2) My mother was ★overcome with emotion★ and burst into tears.
(3) be overcome by/with emotion
(Cambridge Advanced Learner's Dictionary, Fourth Edition)

(4) Her voice ★trembled with emotion★.
(Collins COBUILD Advanced Learner's Dictionary, 9th Edition)

(5) with great emotion (大いに感動して、感情を込めて) (得ウィズダム英和、4版)

(6) a voice shaking with deep emotion (感動委にうち震える声) (ジーニアス英和、5版)

(7) to speak with deep emotion
(8) Her voice broke with emotion.
(The BBI Combinatory Dictionary of English, Third Edition, Expanded and Revised)

(9) feel a lot of emotion
be choked with, be filled with, be overcome with, be overwhelmed with (emotion)
(Macmillan Collocations Dictionary for Learners of English)

***************
0037名無しさん@英語勉強中 (ワッチョイ 8b89-bSA8)
垢版 |
2021/07/27(火) 09:29:13.85ID:Th9bj7iq0
こういうふうに見てみると、
[動詞] with emotion という形で「感動して〜する」という意味になる用例のときに emotion
という mass noun (uncountable noun) が使われることが多い。またそれに似た
[過去分詞] with (または by) emotion という形で「感動によって〜されている(している)」
という状態を示すことも多い。

今回の United by Emotion は、この上の一番下の "過去分詞 by emotion" の形が
使われている。このように [過去分詞] by ときたら、そのあとに emotions という
複数形がつく例はおそらく少ないか、あるいは皆無なのかもしれない。
[過去分詞] by のあとは、emotion という mass noun (uncountable noun) が来る
用例ばかりを辞書では載せている。ネット上では overcome (overwhelmed, united) などの
過去分詞のあとに by や with をつけ、そのあとに emotions という複数形が来る例も
出てくるだろうけど、辞書ではこのときは emotion という mass noun の形しか載せていない。
おそらくはそれが一応は原則(というか標準的)なんだろうと思う。
0039名無しさん@英語勉強中 (ワッチョイ 0b23-QsN2)
垢版 |
2021/07/28(水) 00:49:12.22ID:ihmoBpRz0
The management has once again suggested an action plan that would have us here on this marketing team with focusing exclusively on web-based advertising.

このthat節のhaveの用法がよくわかりません、
have O doingとかの使役じゃなさそうだし、どういうふうに考えればいいでしょうか?
0046名無しさん@英語勉強中 (ワッチョイ 8b89-bSA8)
垢版 |
2021/07/28(水) 07:38:49.56ID:FVBn32g00
>>39
>>The management has once again suggested an action plan
that would have us here on this marketing team with focusing exclusively on web-based advertising.

with を消せば問題ないだろう。

an action plan that would assign us to this marketing team, focusing exclusively on web-based advertising
みたいな意味合いだろう。そして focusing は us にかかっていると考えることもできるし、
this marketing team にかかっているとも考えられるだろうし、action plan にかかっている
と考えることもできると思う。どれにかかっていると解釈したとしても、
本質的な問題はないと思う。
0047名無しさん@英語勉強中 (アウアウウーT Sa5d-WgSx)
垢版 |
2021/07/28(水) 07:48:28.58ID:kC/FPqC/a
微妙に誤用?
0048名無しさん@英語勉強中 (ワッチョイ 8b89-bSA8)
垢版 |
2021/07/28(水) 07:51:09.95ID:FVBn32g00
>>39
>>The management has once again suggested an action plan
that would have us here on this marketing team ★with focusing exclusively on★ web-based advertising.

すでに前のレスで言いかけたように、この★の部分は、

(1) with を消して、focusing exclusively on ... とする。
(2) with a focus exclusively on ... とする。

この二つのうちのいずれかにすれば、文法的にも正しくなり、ちゃんと意味が通じる。
おそらく、この記事を最初に書いた人は口頭でしゃべりまくって、それを別の人に
dictate させたんだろう。そのとき原案を作った人は with a focus exclusively と言っていたのに、
with focusing exclusively と聞き間違ってそのように書いてしまったのかもしれない。

あるいは、原案を書いた人が focusing exclusively と書いていたんだけど、
それを with a focus exclusively (on...) に書き換えようとしたんだけど、
うっかり with だけをつけて、そのあとの focusing を a focus と書き換えるのを
忘れたのかもしれない。英文記事が書かれて、その原案を何度も推敲している
うちに、よくそういうことがある。The Economist や The New York Times や
BBC でさえ、そういうケアレスミスがある。(それはネット上では特に顕著だ。)
0049名無しさん@英語勉強中 (ワッチョイ 8b89-bSA8)
垢版 |
2021/07/28(水) 08:22:25.58ID:FVBn32g00
>>39
おっと、そうそう、質問の趣旨は「have の用法は?」というものだったけど、
今までの議論によってそれは明らかだわな。

have us here on this marketing team は "have + 目的語 + 副詞句」というわけで、
直訳すれば「われわれをこのマーケティング班の上で持つ」という感じだわな。

だから action plan that would have us here on this marketing team を paraphrase すれば、

(1) action plan that would assign us to this marketing team
(2) action plan where we would be on this marketing team

みたいなふうになると考えればいい。
0050名無しさん@英語勉強中 (ワッチョイ fb33-WppR)
垢版 |
2021/07/28(水) 08:26:55.05ID:J8JPc31s0
>>39
他の方の繰り返しになりますが、

have 人 on the teamは、【人 is/are on the team】の状態を持つ、というSVOC/SVOAで、
「人をチームに入れる」といった意味になりますね

参考:英辞郎
-------
have someone on the team
(人)をチームに迎える
-------

「with -ing」は、ネイティブもやりがちなよくある誤用だと思います
非常に多い誤用なので、そのうち誤用とは言われなくなるかもしれませんね
0051名無しさん@英語勉強中 (ワッチョイ 293b-0CkY)
垢版 |
2021/07/28(水) 08:46:07.28ID:JwmEQ2zM0
「我々マーケティングチームは専らウェブ広告に注力してはどうかとのアクションプランが再度経営陣から出された」
have 〜with ingという用法はないのかもしれませんが、us here on this marketing teamがfocusすると読んでも意味が通る気がします
文法的にはちょっと無理のある読み方かもしれませんが、アクションプランは経営戦略みたいな感じらしいので、もしかしたらこういう事を言いたいのかもしれません
0052名無しさん@英語勉強中 (ワッチョイ 8b89-bSA8)
垢版 |
2021/07/28(水) 09:16:55.84ID:FVBn32g00
>>51
なるほど、その読み方が最も自然な読み方なんだろうと思う。原文の文脈はわからないけど、
おそらくは TOEIC の問題だろう。本来はもっと長い文脈があって、us が実はすでに
this marketing team のメンバーだということが書いてあったんだろうと思う。

そういう文脈の後に今回の原文を読めば、このスレにいる人たちはみんな、
すんなりと本当の意味がわかっただろう。
0053🤕三年英太郎🤒 ◆3CZBjOt3.Y (ワッチョイW 1386-CZYx)
垢版 |
2021/07/28(水) 18:07:15.00ID:fjs+6MnE0
アンリミにもある薬理凶室の本によると、セントジョーンズワートが市販されてるものでは唯一効用が認めらてるサプリだとか(メンタル系ので)

幸か不幸か、約2年経って、完全にxが抜けてしまったというか、あの高揚感が思い出せないね🙄どんな感じだったっけな〜🙄
0055名無しさん@英語勉強中 (スフッ Sd33-kVM/)
垢版 |
2021/07/28(水) 23:10:37.60ID:KEH49/j0d
君は疲れてそう。早く帰ったほうがいいね
You look tired. You ( )( )go home.

でought to ってダメですか?
0057名無しさん@英語勉強中 (スフッ Sd33-kVM/)
垢版 |
2021/07/28(水) 23:24:05.39ID:KEH49/j0d
早く帰ったほうがいいよ。でした
0058名無しさん@英語勉強中 (ワッチョイW 7bc5-AHyl)
垢版 |
2021/07/29(木) 03:38:43.70ID:VxsNP26+0
had betterってーしたほうがいいというのが恩着せがましいというか、
上から目線すぎというか、子供に向かって大人が偉そうに押し付けている
という印象だから、使わないほうがいいと本に書いてあったけど、
アメリカ人友達にそのことを話したら、いや、普通に使う。問題なし。
と言われたことがあった。日本人の書いた文法書には嘘を書いてる本もあったり、
米語は英語と違ってたりするのかな、と思った。
0059名無しさん@英語勉強中 (ワッチョイW 7bc5-AHyl)
垢版 |
2021/07/29(木) 04:50:44.26ID:VxsNP26+0
had better って一つの助動詞のように本にも書いてるかも知れないけど、
hadって仮定法で現実には存在してないことを表しているんだろうし。
この構文はit would be better that you should do it.
( shouldは判断を表す動詞に続く節で使うやつ。demand とかの。
感情をあらわす形容詞の後の節で使うshouldに似てたかも。米では
should を抜いて仮定法現在の形を使う)
You would be better do it.と繰り込み文法?
だったか、みたいな感じになって、
You had better do it.
となったのかも、なんて勝手に想像しちゃうよね
0060名無しさん@英語勉強中 (ワッチョイ 8b89-bSA8)
垢版 |
2021/07/29(木) 05:00:41.27ID:AodtXnS30
(1) You look tired. You had better go home.
(2) You look tired. You should go home.
(3) You look tired. You ought to go home.

最後の "ought to" に関しては、まだ俺にはよくわかっていない。たまにしか ought to
の使われている用例を見ないから、その細かい意味合いや語気やインパクトが
俺には十分にわかっていないのだ。ought to の意味合いについては、もちろん
辞書にきちんと説明はしてある。ネット上でもいろんな人が説明している。
しかしそんな説明をされても、すぐに ought to を should や had better とうまく
自由自在にネイティブみたいに使いこなせるはずがない。

had better とか should については、すでにそれぞれ数千回あるいは1万回くらいずつ
いろんな会話や映画や小説などで見聞きしてきたから、それを使い分ける自信はある。
しかし ought to を使っている用例は、たぶん数百回しか見聞きしたことがない。
だから俺には ought to を使いこなす自信がない。

should を使った方が無難だから、俺自身もよっぽどじゃないと had better は使わない。
英語ネイティブ並みになったあとでないと、日本人は had better は使わないようにした方が
無難だと判断して、あちこちの解説者はそれを使うことについて注意を喚起しているのだ。
というのも、日本語で「〜した方がいい」がそのまま had better であり、
「〜べき」がそのまま should だと思い込んでいる馬鹿が多いからだ。だからこそ
それは間違っていると解説者たちが言いたいのだろう。その点では正しい。

しかしだからと言って、had better を日常会話の中で「〜した方がいいよ」という
日本語で言いたいときに使うことが「ない」などと言うと、間違いに決まっている。
それについてアメリカ人が「大いに使うよ」と言うのは当然だ。
0061名無しさん@英語勉強中 (ワッチョイ 8b89-bSA8)
垢版 |
2021/07/29(木) 05:00:53.07ID:AodtXnS30
had better は、相手が苦しそうにしているから、「家に帰った方がいいよ。そうしないと
あとで君が困ることになるよ」と気遣いしているときに使うのだ。もちろん
You talk too much. You'd better shut up. などという文脈で使うのは、喧嘩する覚悟がない限りは
やめた方がいいに決まっている。社長が会議に遅れてきたときに、平社員が
You'd better be careful next time. なんてことを言ったら気を悪くされるに決まってるわな。

どういう状況で had better がふさわしいか、あるいは should がふさわしいかを
理解もせずに「〜した方がいい」は had better で、「〜べき」が should だとか、
あるいはそうではなくてどんな場合にだって should にしないといけないのだ、英米人は
had better を使わないのだ、などという短絡的な勉強の仕方しかしてない人には、
「差し当たって日本人は had better を使わせない方が無難」と思われても仕方ない。
0062名無しさん@英語勉強中 (ワッチョイW 7bc5-AHyl)
垢版 |
2021/07/29(木) 05:22:32.66ID:VxsNP26+0
You use had better or ‘d better when you are advising, warning, or threatening someone, or expressing an opinion about what should happen.
▶ It’s half past two. I think we had better go home.
▶ You’d better run if you’re going to get your ticket.

こうビルドの説明

オーレックスは次のように説明

(1) You を主語にすると命令口調になり You should ... より意味が強い.子供や親しい人以外には使わない方がよい.助言には It would be better (for you) to do のような表現を使うのがよい.
(2) (口)では ’d better と短縮する.さらに ’d も落ちて better だけになることもある.
(3) 疑問文
0063名無しさん@英語勉強中 (アウアウウーT Sa5d-WgSx)
垢版 |
2021/07/29(木) 07:49:57.51ID:6TmlCVnua
>You ought to go home.

汝、帰宅すべし。
0064名無しさん@英語勉強中 (ワッチョイ 7bc5-bstj)
垢版 |
2021/07/29(木) 10:07:59.23ID:VxsNP26+0
オーレックスには
shouldとought to
ought toはshouldとほぼ同様に用いられるが, shouldが話者の主観による道徳的判断や権威など内的要因による判断に用いられるのに対し, ought toは社会的習慣・規則など外的要因による判断を表しshouldより若干強意的. 続いて, had better, be to do, need (to), have (got) to, mustの順で強制力は強くなる.

とか
[ought to do](道徳的判断・社会的習慣・期待として)…すべきである, …しなくては[でなければ]ならない; (忠告・勧め・提案として)…した方がいい(!単に意見を述べるだけで, had betterのように従わない場合の重大な結果は暗示しない;

と書いているんだよね。had betterってどれだけきつい言い方なんだと、この
辞書見てたら思うんだけど、だけど、アメリカ友人からはそういうふうに
聞かなかった。(だからhad betterを僕は普通に、ーーしたらいいよーと
気楽に使うことにしている)
0066名無しさん@英語勉強中 (ニククエ 7bc5-bstj)
垢版 |
2021/07/29(木) 14:49:59.78ID:VxsNP26+0NIKU
当然でしょ。ネイティブの言うことが正しいわけで。
外国人が日本人の自分に日本語の使い方がおかしいですよ。歴史的にですねー....
とか言ってたらその外国人って、あたおかです
0068名無しさん@英語勉強中 (ニククエ Sd33-kVM/)
垢版 |
2021/07/29(木) 23:25:32.85ID:ChQ0I0E7dNIKU
質問者です。
皆さんありがとうございます🙇‍♀
テキストにはshouldもought toもhad betterも「した方が良い」でhad betterが1番強いとしか書いてなかったのでどれ選べばいいのかと悩んでました
0074名無しさん@英語勉強中 (ワッチョイ 4e33-i0sM)
垢版 |
2021/07/30(金) 11:35:30.86ID:7IOZG9bM0
had betterは「そうしないとたいへんなことが起きるよ」の含意がありますよね
shoudは主観的当然性、ought toは社会的当然性といったところでしょうか

ミントンさんは「通常のhad betterはそんなにきつくないよ」と言ってますね

ミントン『日本人の英文法 完全治療クリニック』
-----------------
通常の用法でのhad betterは、命令形とはほど遠いものです。...
I think you'd better shut up.と言うのとI think you should shut up.と言うのに
違いはありません。どちらも無礼です!同じように、I think you'd better see the doctor.
と言うのとI think you should see the doctor.と言うのにも違いはありません。
どちらも、他人への気遣いを示しており、したがって、基本的に礼儀正しい発言です。...

... had betterが命令形に近いというこの神話は、私の想像では、
おそらく「脅す」というhad betterの特殊な機能から生じたのではないでしょうか。
-----------------
0077名無しさん@英語勉強中 (ワッチョイ fa2d-By/s)
垢版 |
2021/07/30(金) 19:55:35.74ID:uI6pIa9e0
The conditions under which these fine works were created were usually
of a most difficult kind.

このofってどう考えたらよいのですか?
0078名無しさん@英語勉強中 (ワッチョイ d632-4OD8)
垢版 |
2021/07/30(金) 21:02:53.87ID:2O+d/13B0
OEDのhaveの項にはこう書いてあるよ

22.a. The past Subjunctive had = would have, is used idiomatically with adjectives (or adverbs) in the comparative, as better, liefer, sooner, rather; in the superlative, as best, liefest; or in the positive with 'as', as good, as lief, as soon, as well, to express preference or comparative desirability.

had betterが特別形ではなく、had soonerやhad lieferみたいに使えるイディオム(の例)なんだね
意味的には「嗜好」や「願望」を表すのだから、オーレックスとはずいぶん感じが違う
0079名無しさん@英語勉強中 (ワッチョイ 4d3b-F2ed)
垢版 |
2021/07/30(金) 22:26:12.64ID:ZkPzngqX0
>>77
the conditions of a most difficult kindを文にするとそうなります
これが正しい説明かはわかりませんが、be動詞等を取っ払って名詞として考えてみると意味がわかるときが多々ありますよ
0080名無しさん@英語勉強中 (ワッチョイ 5d9d-By/s)
垢版 |
2021/07/30(金) 22:36:12.92ID:fZM/9BOb0
Cambridge Dictionary

Had better: form and meaning
We use had better to refer to the present or the future, to talk about actions we think people should do or which are desirable in a specific situation. The verb form is always had, not have. We normally shorten it to ’d better in informal situations. It is followed by the infinitive without to:

It’s five o’clock. I’d better go now before the traffic gets too bad.



The democratic movement had better concentrate on the immediate issues of the economy and security. (more formal)

Had better is a strong expression. We use it if we think there will be negative results if someone does not do what is desired or suggested:

She’d better get here soon or she’ll miss the opening ceremony.
0081名無しさん@英語勉強中 (スプッッ Sd7a-hdqv)
垢版 |
2021/07/30(金) 23:51:39.25ID:CWG87aoYd
>>77

16. PREPOSITION [be PREP n]
You use of after the verb 'be' to indicate a characteristic or quality that someone or something has.
[formal]
The crisis faced over the next few months is of an entirely different scale.
Both world wars were of unquestionable importance as economic events.
(Collins COBUILD)

上の辞書に書いてある通り、人やモノが持つ性質や特徴などを表すときに、be動詞の後に、of+名詞がくる場合があるけど
解釈する際には、of+名詞で(補語となる)形容詞句をなしてると考えればよいと思う。
0082名無しさん@英語勉強中 (ワッチョイ 5d9d-Oj7i)
垢版 |
2021/07/31(土) 00:45:08.53ID:cQV0/+AN0
>>76
キャサリン A クラフトさんは「日本人の9割が知らない英語の常識181」という本で
”You had better”は「〜したほうがいい」という穏便な助言ではなく、むしろ警告に近い響きをもっていて、
言われた通りにしないと何か悪いことが起きるという意味合いを含んでいます。
と書いているよ。
ネイティブの間でも多少感じ方に違いがあるようだね。
0084三年英太郎 ◆3CZBjOt3.Y (ワッチョイ 7a86-i0sM)
垢版 |
2021/07/31(土) 01:19:46.59ID:NTE0v+xz0
言語の調査では、よっぽど当然のことでもない限り、揺れがないということはない。
それを踏まえたうえで、辞書や語法書は最大公約数を書くものである。

We use "had better" to give strong advice, or to tell people what to do (including ourselves).

・You’d better turn that music down before your dad gets angry.
・It’s seven o’clock. I’d better put the meat in the oven.

Had better is not used in polite requests. Compare:

・Could you help me, if you’ve got time? (request)
・You’d better help me. If you don’t, there’ll be trouble. (order/threat)

PEU§77
0085三年英太郎 ◆3CZBjOt3.Y (ワッチョイ 7a86-i0sM)
垢版 |
2021/07/31(土) 01:28:57.94ID:NTE0v+xz0
この件に限らず、ミントン本はゴミだから、参考にする価値はない。

日本人がなんちゃらの英文法の3巻だったか、
日本の教科書に見られる is loved by .... のごとき like/love の受動態を
見ると吐き気だするとか言ってた。

はて、と思って、そのような文がでてる英語の教科書を
実際にイギリス人に見せたら、彼は全く問題がないというし、
彼のパートナーと来客に見せても問題がないと言った。
Gogh is loved by a lot of people in Japan. とかそんな感じの文。

ひとたび本を出すなら、自分の語感だけに頼らず、ちゃんと他の母語話者にも聞かなきゃダメ。
著者は、それが分からないの程度の知的レベルということ。

他にも時制の問題など、ごちゃごちゃと付け足してまるで整理できてなかった。
0086名無しさん@英語勉強中 (ワッチョイ 4e33-i0sM)
垢版 |
2021/07/31(土) 10:10:54.72ID:V26Ny6fi0
>>82
had betterに「言われた通りにしないと何か悪いことが起きる」という含意があるのは、
誰も否定していないと思います
その「悪いこと」を話し手が起こすのであれば、脅迫っぽくなる、という話ですね

「さもないとマズイことになるぜ」が「こっちからマズイことをしてやるぞ」になる(黄ロイヤル)

「ちょっとだまっててよ。さもないとたいへんなことになるよ」←脅迫
「医者に診てもらいなよ。さもないとたいへんなことになるよ」←強いアドバイス

緊急性(安藤)や緊迫感(大西)といったイメージも重要そうです
0087名無しさん@英語勉強中 (ワッチョイW 8ec5-mkmt)
垢版 |
2021/07/31(土) 10:19:04.86ID:bvIUJ7fD0
>>78

had as good [well] do (as do)
(…するぐらいなら)…した方がいい

would [had] as lief do (as ...)
(…よりは)…する方がましだ

had as soon do
…した方がいい.


とウィズダムに載っていた。例文は載ってなかった
0093名無しさん@英語勉強中 (ワッチョイ f196-tWs3)
垢版 |
2021/07/31(土) 15:05:42.07ID:btvXPufZ0
よろしくお願いします。

1)Only after a century of confusion was the royal authority restored.
2)Aftrer only a century of confusion, the royal authority was restored.

Q1
上の二つの英文ですが、意味に違いがあるんでしょうか?
また、意味ですが、「わずか一世紀の混乱の後、」ではなく「一世紀も続いた混乱の後、」みたいなニュアンスらしいのですが、
どこでそう解釈すべきなんでしょうか?

Q2
1の方は、onlyを強調するために文頭に出して、そのせいで倒置をした、という理解でよいのでしょうか?
0094名無しさん@英語勉強中 (ワッチョイ d689-By/s)
垢版 |
2021/07/31(土) 16:54:54.66ID:vT7XU0cP0
>>93
(1) Only after a century of confusion was the royal authority restored.

一世紀にわたる混乱のあとになって初めて(一世紀にわたる混乱のあとになってやっと)
---> そういう意味だから、「一世紀も続いた混乱のあとに」と訳すこともできる

(2) After only a century of confusion, the royal authority was restored.

ほんの1世紀にわたる混乱のあとに(わずか1世紀にわたる混乱のあとに)
---> 1っ世紀しか混乱が続かなかった、という意味合い
0095名無しさん@英語勉強中 (ワッチョイ d689-By/s)
垢版 |
2021/07/31(土) 16:55:48.97ID:vT7XU0cP0
>>93
only がどういう意味合いで使われるのかを、再び辞書を見て、その解説と例文を
しっかり読んでほしい。
0096名無しさん@英語勉強中 (ワッチョイ fa33-i0sM)
垢版 |
2021/07/31(土) 17:21:54.50ID:a8eiLF/s0
現在、YouTubeの数学の講義動画を英語の字幕付きで見ています。

文脈から明らかに「vote」と講師が言っているのですが、英語の字幕では、「boat」となっています。

おそらく、AIを使って自動で字幕をつけているのだと思います。

日本人は「v」と「b」の発音の違いが分からないが、ネイティブにはなぜ日本人には違いが分からないのか信じられないというような話をよく聞きます。(「L」と「R」についてがもっと有名ですが。)

ですが、AIが日本人と同じような間違いを犯すということは、実際に「v」と「b」の発音は近いというのが真実なのではないのでしょうか?

このあたり、実際どうなのでしょうか?

「L」と「R」についてもAIが間違えるのか注意してみてみようと思います。
0097名無しさん@英語勉強中 (ワッチョイ d689-By/s)
垢版 |
2021/07/31(土) 17:32:05.17ID:vT7XU0cP0
>>96
動画の内容について質問するなら、なぜその動画の URL を示さないんだよ?
0098名無しさん@英語勉強中 (ワッチョイ fa33-i0sM)
垢版 |
2021/07/31(土) 17:35:23.03ID:a8eiLF/s0
>>97

https://youtu.be/8hX0QePiWlw?t=1780

↑このあたりからになります。文脈から「vote」だと思っています。
0099名無しさん@英語勉強中 (ワッチョイ 4a8a-ApIo)
垢版 |
2021/07/31(土) 18:04:33.14ID:Nxyd1WYO0
文脈をAIは理解していないんでしょ
発音から明らかにvoteならまだしも

数学の先生が英語ネイティブかも不明だし
日本人の先生が英語で数学教えていたら
vote, boatを同じように発音するかもね
0100名無しさん@英語勉強中 (ワッチョイ 4a8a-ApIo)
垢版 |
2021/07/31(土) 18:07:47.22ID:Nxyd1WYO0
>>98読まずに書いた
>>98を見ての感想

how many boat yes
ハウメニー ボー イヤァース
って聞こえるよ
vの音は入っていないように聞こえますが
0101名無しさん@英語勉強中 (ワッチョイ 1a30-+xg5)
垢版 |
2021/07/31(土) 18:19:33.38ID:aLOSQOzZ0
You will learn Japanese.がGoogle翻訳やDeepL翻訳で
「日本語を学ぶことができます」と訳されます。
これは
You will be able to learn Japanese.
You can learn Japanese.
とどう違いますか。
0102名無しさん@英語勉強中 (ワッチョイ d632-4OD8)
垢版 |
2021/07/31(土) 18:59:26.27ID:ky9TXxuT0
>>99
スペイン語の発音でもBとVは区別しないからね
この先生は加大バークレー校卒らしいので、
地域的か人種的にスペイン語の影響はあるかもしれない
知らんけど・・・
0103名無しさん@英語勉強中 (ワッチョイ 1a30-+xg5)
垢版 |
2021/07/31(土) 20:29:51.23ID:aLOSQOzZ0
>>96
とあるユーチューバーさんはスペイン系の英語話者で、
その人の動画のコメント欄には
「あなたはスペニッシュですか? なぜならvとbを区別していないから」
と書かれていたことがあります。

インド=ヨーロッパ系語族の単語に同じルーツを持つ単語がたくさんありますが、
rとl、vとbとで交換が起こっている単語があるかどうかを調べるのはよい考えだと思います。
0104名無しさん@英語勉強中 (ワッチョイ f196-tWs3)
垢版 |
2021/07/31(土) 22:25:11.19ID:btvXPufZ0
>>94-95
ありがとうございました。
onlyの、(1)の用法ですが、辞書でもよくわかりませんでした。バカ覚えします。

only after = 〜の後になってはじめて
0105名無しさん@英語勉強中 (ワッチョイ d684-zf+7)
垢版 |
2021/08/01(日) 08:17:30.25ID:/3YkT0ST0
質問です。
アサヒウィークリーの記事からの一文です。
Jeff Bezos blasted into space July 20 on his rocket company's first manned flight, becoming the second billionaire in just over a week to ride his own spacecraft.

これをDeeplとGoogleで翻訳かけてみました。
D:ジェフ・ベゾスは7月20日、自身が経営するロケット会社の初の有人飛行で宇宙に飛び立ち、この1週間で2人目の億万長者として自身の宇宙船に乗りました。
G:ジェフ・ベゾスは7月20日、ロケット会社の最初の有人飛行で宇宙に飛び込み、1週間強で自分の宇宙船に乗る2番目の億万長者になりました。

ちなみに私の訳は
私:ジェフベゾスは、彼の所有するロケット会社の初の有人飛行で7月20日に宇宙へ飛び立った。1週間を少し超える期間で彼自身の宇宙船に乗る世界で2番目の億万長者になって。

という少し意味の分からない訳になってしまいました。(ちなみにフォーブズリストでは1番のお金持ち見たいです。)
しかし発話者の思いに一番近い役はDだと思います。
読み違えたポイントは、the secondを「(以前いた億万長者に続いて)二人目の」と「(フォーブスリストで)2番目の」だと思います。
そこで「有人飛行 億万長者」でググると、「ヴァージン創業者が自ら宇宙旅行を実現、すぐにベゾス氏も」という記事を発見しました。
なるほど、この「ヴァージン創業者」に続いてってことか、と納得できました。

ということで、自分で30〜40分ほど背景知識を調べて、発話者の思いを理解できました。
最初、こちらで端から質問しようと思ったのですが、翻訳サイトを思い出したので自分で検索したら何とか正解にたどり着いたのですが、時間がかかってしまいました。
ということで質問はなくなりました。途中まで書いたのを消すのは勿体なので投稿しました。
最後までお読みいただいた方、大変ありがとうございました。
0106名無しさん@英語勉強中 (ブーイモ MMf1-Qvf3)
垢版 |
2021/08/01(日) 09:26:24.77ID:44uVYKFyM
「ちなみに私ジェフベゾスは、彼の所有するロケット会社の初の有人飛行で7月20日に宇宙へ飛び立った。1週間を少し超える期間で彼自身の宇宙船に乗る世界で2番目の億万長者になって」

普通に意味わかるが…
>>105の日本語の説明が混乱してて何言ってるかわからんw
0107名無しさん@英語勉強中 (ワッチョイ faf0-QEAr)
垢版 |
2021/08/01(日) 09:34:11.55ID:SUykVuJ40
>>105

NO PROB 
 
0109名無しさん@英語勉強中 (ササクッテロロ Sp05-ZFfg)
垢版 |
2021/08/01(日) 10:04:49.83ID:HG9Y1H9Op
恐らく語学というジャンルにおいて重要なのは、

「ジェフベゾスは、彼の所有するロケット会社の初の有人飛行で7月20日に宇宙へ飛び立った。1週間を少し超える期間で彼自身の宇宙船に乗る世界で2番目の億万長者になって」

まで訳した後に、
この訳から
「ベゾス氏は『自分の宇宙船に乗る億万長者として世界で二番目』」
「一番目の億万長者は一週間ちょっと前(つまりこの『番目』は金持ちランキングの順位ではない)」
といった情報を抽出できる事ですね。
ちょっと読みにくい程度でわけがわからなくなっていては、永遠に「自然な日本語に訳されたもの」しか読めません。

「ジェフベゾスは、彼の所有するロケット会社の初の有人飛行で7月20日に宇宙へ飛び立った。1週間を少し超える期間で彼自身の宇宙船に乗る世界で2番目の億万長者になって」
まで訳せるということは、ある意味、英語力は充分です。そこから先でもう少し頭を使いましょう。
0111名無しさん@英語勉強中 (ワッチョイW 8ec5-mkmt)
垢版 |
2021/08/01(日) 10:43:08.54ID:4YpXT4gh0
>>105
second billionaireで二番目に金持ちの人とするのには少しジャンプがあるかなと。
in just over a week で
一週間をわずかに越えた期間でなんですね。辞書見てわかった。ちょうど一週間の期間で
だと思ってた。



>>他の人
二番目にという表現と、誰それを除いて一番目
という表現ってなんでしたっけ
second richest とかはググると出てくるけどそういうのではなく。
何か熟語みたいな感じであったけど、すぐに出てこない。
0112名無しさん@英語勉強中 (ワッチョイ f196-tWs3)
垢版 |
2021/08/01(日) 10:49:47.70ID:kLQ6NmIo0
>>108
むちゃくちゃよくわかりました! ありがとうございました!
0113名無しさん@英語勉強中 (ワッチョイW 8ec5-mkmt)
垢版 |
2021/08/01(日) 10:49:48.82ID:4YpXT4gh0
>>109
訳が直訳だから拙く見えていることと、
二番目に裕福とした誤訳をしたことは
、切り分けましょう。

直訳の拙さと言いますが直訳でいいじゃないですか。
英文の構造が捉えられればそれでいい。
直訳の拙さを嘆くよりもDeepLの偉大さを讃えるべきでしょう
DeepLアクバーと。
0114名無しさん@英語勉強中 (ワッチョイW 8ec5-mkmt)
垢版 |
2021/08/01(日) 11:02:15.62ID:4YpXT4gh0
>>110
becoming the second billionaire in just over a week to ride his own spacecraft.
こういう書き方するんですね。普段強く意識してないから、110さんの書き込みを見て、
なるほどねと思った。

不定詞の前にnot を置いて、to 以下しないこと、としますよね。それ風に考えられないか
というと考えられないわけじゃない。つまりto以下の部分に入るべきものが、
to の前に来るということですが。not の場合は必ずだけど、それ以外のものについては、
来てもいい、と考えられなくもない。

だけどそういうのをルール化するとなんでもありになりそうなので、と拙く表現させていただいた上で、
次のように考えるのがよりいいんではないか。

一週間で二人目ですよ、と言い切ってしまって、言いたいことは言ってしまって溜飲を下げた後に、
自分の宇宙船に乗る、と書いてるけど乗って宇宙に出ることだろうけど文脈上わかるから乗ると
しておいて、自分の宇宙船に乗るのは、と説明をつける。
文が引き締まる
0119名無しさん@英語勉強中 (ワッチョイ d689-By/s)
垢版 |
2021/08/01(日) 17:56:22.98ID:MK5kzwII0
>>111
>>誰それを除いて一番目という表現ってなんでしたっけ

もしかして、
the richest (the tallest, the cleverest) man but one
みたいな表現のことを言っているのかな?
0121名無しさん@英語勉強中 (ワッチョイ 8ec5-aiGt)
垢版 |
2021/08/01(日) 23:49:53.45ID:4YpXT4gh0
教科書に使ってる本がスチュアートの解析の本だったんじゃなかったっけ?
0124名無しさん@英語勉強中 (ワッチョイ fa33-i0sM)
垢版 |
2021/08/01(日) 23:57:32.55ID:Y/nEOVbK0
>>98

↓同じ講師が、「v」と言っているシーンですが、1回目には「b」と字幕に表示されます。2回目には「v」と表示されます。

やはり、「v」と「b」の発音は似ているからAIが間違えるということのようですね。

https://youtu.be/_FQqk6TFuhM?t=2338
0125名無しさん@英語勉強中 (ワッチョイ f1ac-QEAr)
垢版 |
2021/08/02(月) 03:21:04.90ID:5+z8aZZu0
>>124
「似てない、似ても似つかない」っていうのは、あくまで雑音なしの状況ではっきり発音された場合の話であって、
動画のように雑音だらけの状況ではvとbとかそういうレベルじゃなくて、今の音が何であったか完全に聞き取るのは困難でしょう。
日本語でも同じで、ナ行とマ行、ディーとビーなど、通常ほぼ100%聞き分けられる音でも、少し聞こえにくい状況だと難しくなります。

動画から一部を抜粋して、正しく書き直してみました。複数の単語にわたる部分は""で囲いました。
「vとbとかそういうレベルじゃない」ことがお分かりいただけると思います。

but here's here's a fact for you is(it's) useful if you take a scalar times X what's the length of that
scalar times "only my"(???) legs(x) "I'm glad"(Good) you fell on my track(trap) how many boat(vote) yes "I'll
go"(And don't) be intimidated chilly how many boat(vote) no yes so when I said you fell in my trap you need
absolute values there why because "the mega new star square is angry"(negatives were squared so...)
link-lengths(length) are always positive or zero and if you took a negative staler(scalar)
obviously "you can"(???) cook(took) took negative 2x it still got the same length as 2x does
so you need the absolute value of the scale

邦訳:
先生:ここで有益な事実を紹介しましょう。スカラー倍されたベクトルXの長さはいくらでしょう?
生徒:スカラー倍のX
先生:よっしゃ!罠にかかった!(スカラー倍のXに)賛成の人は?怖がらなくていい。
TA?:落ち着いて。
先生:不賛成の人は?ふむ。私が罠にかかったと言ったとき、絶対値記号を付けるべきだったんだ。なぜ?
生徒:負の数は二乗されているから。
先生:長さは常に正の数もしくはゼロなんだ。もし負のスカラーを、具体的に-2xを取ってみると、
それは2xと同じ長さだよね。だからスケール(長さ)を測るときは絶対値を取らないといけない。
0126125 (ワッチョイ f1ac-QEAr)
垢版 |
2021/08/02(月) 03:35:41.14ID:5+z8aZZu0
補足だけど、雑音がほぼない動画では、自動生成の字幕でもかなり正確になってきてる。
最近、Youtubeでは一昔前にあった視聴者による字幕投稿がなくなってしまったけど、
それでも投稿者による字幕がついてる動画は多々ある。
正直、雑音の多い動画での自動字幕で勉強するのはちょっと効率悪いかなと思う。
0127125 (ワッチョイ f1ac-QEAr)
垢版 |
2021/08/02(月) 03:39:09.64ID:5+z8aZZu0
最後に、>>96さんのそもそものご質問に私なりの回答を。

>>>実際に「v」と「b」の発音は近いというのが真実なのではないのでしょうか?

そうだと思います。
上にも書きましたが日本語のマミムメモとナニヌネノくらいは近いんじゃないかと思います。
0129125 (ワッチョイW f1ac-Xx7c)
垢版 |
2021/08/02(月) 16:03:24.35ID:5+z8aZZu0
>>125
気づいたので訂正します。
最後のabsolute value of the scaleはabsolute value of the “scalar”ですね。
訳は「スカラーの絶対値を取らないといけない」で、文脈上もこっちの方が自然ですね。

あと、私は数学も少々詳しいだけで、まだ細かい間違いはあるでしょうということだけ言っておきます。では失礼します。
0130名無しさん@英語勉強中 (ワッチョイ d629-By/s)
垢版 |
2021/08/02(月) 18:14:11.54ID:WtxpFcSW0
Grammar in use Intermediate(3rd edition)という英文法の本で勉強しています。
問題編の68.1の10.に次の問題があります。
Our (travel / trip) from Paris to Istanbul by train was very tiring.

解説を見ると正解は"trip"になっていますが、私はtravelでも間違って
いないような気がします。
どうしてtravelだと誤りになるのでしょうか?
0131名無しさん@英語勉強中 (ワッチョイ 155a-KzYc)
垢版 |
2021/08/02(月) 18:23:01.44ID:gHECvdVu0
>>130
解説ページに書いてあるじゃん
0132名無しさん@英語勉強中 (ワッチョイ 155a-KzYc)
垢版 |
2021/08/02(月) 18:31:10.60ID:gHECvdVu0
Grammar in Use Countable and uncountable 2
Travel (noun) means ‘travelling in general’ (uncountable). We do not say ‘a travel’ to mean a trip or a journey:
・They spend a lot of money on travel.
・We had a very good trip/journey. (not a good travel)

----

https://www.ldoceonline.com/dictionary/travel
You use travel to talk in general about the activity of travelling:
・Long-distance travel has become much cheaper.
・My interests are travel and photography.

Travel is only used as an uncountable noun.
✗Don't say: a travel | the travel

travels
Someone’s travels are their journeys to many different places:
・On her travels, she visited Egypt, Jordan, and South Africa.
・I’ll call you when I get back from my travels.

Travels is always used as a plural noun in this meaning.
✗Don’t say: her travel | my travel

sなしtravelは概念名詞で個々の旅行を言うのには使えない。
特定の旅行を言う場合は複数形かtrip他を使う。
0133名無しさん@英語勉強中 (ワッチョイ 01e1-nLA/)
垢版 |
2021/08/02(月) 18:41:47.39ID:epnOTFgN0
知人がCMに出てるのを見て
「彼はすごいハンサムに写ってるのを見て驚きました」っていう場合
(I am) Amazed he looks stunningly handsome !

っておかしいですか?

I was amazed that heで 過去形で書き始めたら
時制を一致させてHe lookedって書きますか?

でも「見た時驚いた」というのは過去で

彼がハンサムなのは、今もそうだけど

でも時制を一致させるんでしたっけ?忘れてしまいましたw
教えて下さい!
0134名無しさん@英語勉強中 (ワッチョイ ce8b-Dvcp)
垢版 |
2021/08/02(月) 19:19:21.72ID:tnZUVW1O0
英語の小説(簡単めだけど、ストーリーは幼児向けではない、普通の日常系の話、大学入試レベル)と
その和訳が載ってるサイトがあったら教えて欲しいですm(__)m
前は奥増夫さんって人のサイトを使ってたんですがもう見れなくて
0135名無しさん@英語勉強中 (ワッチョイW 4df0-d0wC)
垢版 |
2021/08/02(月) 19:38:16.94ID:jNu2eFQC0
>>132
丁寧に教えていただいてありがとうございます。
travelは特定の旅行には使えないんですね。解説よく読んだら書いてますね…。
スッキリしました。
ありがとうございました!
0136名無しさん@英語勉強中 (ワッチョイ fa2d-By/s)
垢版 |
2021/08/02(月) 22:00:04.91ID:Q0mc8N4A0
Olympic groups landed amid the threat of the infectious delta variant that has triggered case surges throughout Asia.のcaseは患者?で直前にthat省略ですか?
0139名無しさん@英語勉強中 (ワッチョイ d689-By/s)
垢版 |
2021/08/03(火) 04:59:21.75ID:4ttkWF6w0
>>133
was amazed that he is
っていうふうに書いてもいいか、という質問に対して、英語ネイティブらしき人2人が
was amazed that he WAS
というふうに過去形にしなさい、と言っている。
https://hinative.com/ja/questions/18215460

しかし会話の中でその原則がどのくらい守られているのかは知らないけど。
0140名無しさん@英語勉強中 (ワッチョイ 5d9d-By/s)
垢版 |
2021/08/03(火) 07:11:59.19ID:pOipGrhv0
質問です。

「セリフ」という言葉の英訳にいくつかの選択肢があり、どれを使うのが適切か分かりません。
以下のような内容の場合、どの単語を使うべきでしょうか?

・100円を払えばこの映画の金切り声の音声が得られます。
・300円を払えばさらに日本語のセリフの音声も得られます。
 ただし英語のセリフは無いので注意してください。

これをDeepLで翻訳すると以下のようになります。

・If you pay 100 yen, you can get the sound of the screeching voice of this movie.
・If you pay 300 yen, you can also get the Japanese dialogue.
 Note that there is no English dialogue.

この日本語映画のセリフは、役者1人が視聴者に話かける一人称視点のものなのですが
この場合「dialogue」と「line」のどちらが適切なのでしょう?
0142名無しさん@英語勉強中 (ワッチョイ 4d3b-By/s)
垢版 |
2021/08/03(火) 08:02:00.48ID:N+pTOfhx0
>>140
For 100 JPY, you can get the sound file of the screeching voice.
For 300 JPY, you can get the audio file of all the lines in this movie in Japanese, as well as the above.
(Note that there's no English-dubbed version.)

dialogueだとdiaだから二人の対話ということになるので、この場合使うならmonologueのほうが適切でしょう
0146名無しさん@英語勉強中 (アウアウウーT Sa09-7QjZ)
垢版 |
2021/08/03(火) 11:18:24.60ID:jK9nYw3Ba
シンプルにtranscriptでいいだろう
(場合によってはtext)
0148名無しさん@英語勉強中 (ワッチョイ fa2d-By/s)
垢版 |
2021/08/03(火) 16:17:03.08ID:PJBiiytq0
PCで見れている表示されているレスの英文フォントは何ですか?
すごく見やすくて
0150名無しさん@英語勉強中 (ワッチョイ ce30-aiGt)
垢版 |
2021/08/03(火) 19:10:21.00ID:Gf4Y70lI0
>>148
フォントのスクショ撮って
https://mao.5ch.net/test/read.cgi/dtp/1400344348/l50 このスレにimgurでアップすれば誰かが教えてくれるかもよ
それかhttp://www.identifont.com の一番左のところでセリフかサンセリフか選ぶか
https://www.myfonts.com/WhatTheFont/ に画像アップロードするか
ともかくここはスレチだから質問したいなら上にあげたスレに行け
0151名無しさん@英語勉強中 (ワッチョイ 5d9d-Ln56)
垢版 |
2021/08/03(火) 22:28:17.24ID:JDImvbEa0
>この板の投稿数順位
>最終更新日時:2021/08/03 22:17:00
>本日の総投稿数: 159 順位: 459/1,028
>分あたり投稿数: 0 順位: 482/1,028


投稿159じゃ書き込み屋君やる気ゼロだね
コロナデルタ株で英検、TOEICは中止だから書き込むネタもないよね
書き込み屋君の活躍はこのスレくらいかな(笑)
0152名無しさん@英語勉強中 (ワッチョイ 659d-Ln56)
垢版 |
2021/08/04(水) 05:37:59.13ID:7GgJnRf30
発音についての議論の方向性が完全にアウトだね
重要なのは
https://www2.nhk.or.jp/gogaku/news/2107/28.html
これを聞いてすぐイギリス発音と分かり、自分の英米の発音がごっちゃにならないことだね。
ネイティブのレッスンを受けてないと議論が変な方向にいく。
俺は中西部のアメリカ人のレッスンしか受けない。
0154名無しさん@英語勉強中 (ワッチョイ 7a9d-i0sM)
垢版 |
2021/08/04(水) 13:41:50.35ID:ImdC+hi30
Where are you from? --- I'm from Japan.

中1レベルの表現ですが、返答がJapanやOkinawaなど名詞になるので文法的には

What are you from?であるべきではないでしょうか?

以下、私の仮説です。ご意見、訂正をお願いします。

仮説1:場所をたずねるので、whereが分かりやすい。Whereを名詞的に使っている

仮説2:from 名詞 をひとまとめで副詞扱いする
0155名無しさん@英語勉強中 (ワッチョイ d689-Ln56)
垢版 |
2021/08/04(水) 14:58:31.64ID:Ge9VJ7p90
>>154
確かに出身地を尋ねるときに What are you from? なんていう言い回しは見聞きしたことがない。
Where are you from? だわな。ほんで、あなたの言う通り、from where で副詞句扱いしている
んだろうな。

それに似た例として、where you're at というのもあるな。そのイディオムの意味合いについては、
この下のリンク先を参照。
https://idioms.thefreedictionary.com/where+we+are+at#:~:text=colloquial%20The%20way%20in%20which,or%20emotional%20status%20or%20condition.
0156名無しさん@英語勉強中 (ワッチョイ bde2-nzU0)
垢版 |
2021/08/04(水) 15:05:29.84ID:edX69rjH0
>>154
what使うならWhat country are you from?

疑問副詞whereは「〜から」は含意しない。それは他の場所の副詞でも同様。He came from abroad/there/home.
0157名無しさん@英語勉強中 (スププT Sd9a-QEAr)
垢版 |
2021/08/04(水) 17:06:40.57ID:kIXTLt0cd
circumstanceに同格のthatって使える場合ある?
the circumstance that 完全文 ってきたら関係副詞のthatですよね
0158名無しさん@英語勉強中 (ワッチョイ d689-Ln56)
垢版 |
2021/08/04(水) 17:32:16.07ID:Ge9VJ7p90
>>157
"circumstance that S V (完全文)" という形で、that が同格を表しているのか関係副詞なのか
を判断するのは、難しいな。たとえば次のような用例では、that がそのどちらなのかが
俺にはよくわからん。

For instance, it is not a ★circumstance that★ a brave man acts bravely for the sake of the good of courage;
instead, it is a ●circumstance that● he acts bravely for the sake of freeing his city, or for the sake of the
Christian people, or for something else of this sort.
The same holds for what?. For instance, it is not a circumstance of someone’s pouring water on
another that he makes him wet; but it is a ★circumstance that★, in pouring the water, he makes him cold or
hot, or heals him or harms him.
https://www3.nd.edu/~afreddos/summa-translation/Part%201-2/st1-2-ques07.pdf
0160名無しさん@英語勉強中 (ワッチョイ d689-Ln56)
垢版 |
2021/08/04(水) 18:28:35.89ID:Ge9VJ7p90
>>159
なるほど、ありがとう。
0161名無しさん@英語勉強中 (ワッチョイ 0a5f-i0sM)
垢版 |
2021/08/04(水) 20:48:34.04ID:WfH9t80F0
The large elephant moved its bulk with legs as strong as tree trunks.
その大きな象は巨体を「木の幹のように力強く」 or 「木の幹のように力強い」脚で動かす。

基礎的なことでしたら申し訳ないのですが、このas strong as tree trunksの部分のstrongは副詞という解釈でよろしいでしょうか?
「木の幹のよう力強く」という表現がしっくりこないせいで混乱しています。
象の脚が木の幹に似ていて、legsを比喩的に表しているような気もするのですが、strongが形容詞だとそもそも文法がおかしいですよね?
0163名無しさん@英語勉強中 (ワッチョイ 4d3b-By/s)
垢版 |
2021/08/04(水) 21:35:33.19ID:O0ln+9qQ0
形容詞だと思います
strong legsを、asを使って機械的に修飾するとas strong as tree trunks legsとなりますが、
一般的に修飾する部分が長いと後ろに置かれるので、legs as strong as tree trunksとなったんだとおもいます。

trunkは象の鼻という意味もあるので、もしかしたら縁語的な感じで言葉遊びを狙ったのかもしれません
0164名無しさん@英語勉強中 (ワッチョイ 0a5f-i0sM)
垢版 |
2021/08/04(水) 22:50:22.00ID:WfH9t80F0
>>163
なるほど
その発想は浮かびませんでした
ありがとうございます
0165名無しさん@英語勉強中 (スププT Sd9a-QEAr)
垢版 |
2021/08/05(木) 03:17:40.84ID:yphfKkzzd
>>158-159
レスありがとうございます
ちなみにthatが同格か関係副詞か悩んでいたのは以下の文です
It is of course true that in a certain sense the individual is predestined to talk, but that is due entirely to the circumstance that he is born not merely in nature, but in the lap of a society that is certain, reasonably certain, to lead him to its traditions.
0166名無しさん@英語勉強中 (アウアウウーT Sa09-7QjZ)
垢版 |
2021/08/05(木) 03:22:44.64ID:J5cZd4L3a
>>161
...legs (that are ) as strong as tree trunks.
という形じゃないかな?
0167名無しさん@英語勉強中 (ワッチョイ d689-Ln56)
垢版 |
2021/08/05(木) 06:13:36.71ID:OVW8sZy70
>>165
その文章の中に出てくる the circumstance that S V の要点をまとめると

the circumstance that [the individual] is born in [a certain environment]

ということになるけど、ここでの that が同格なのか関係副詞かなんて、どうでも
いいという気がしない?要は「〜という状況」という意味なんでしょう?

ただし、文法学者になりたいか、あるいはそれ並みに文法に詳しい人になりたい、
あるいは文法についての議論が好きでたまらない人は別だけど。
0168名無しさん@英語勉強中 (ワッチョイ 5d9d-By/s)
垢版 |
2021/08/05(木) 06:14:03.06ID:FVMdlBD50
「鼻歌部分の曲」というのはどう英訳すればいいのでしょうか?

配布ファイルの中に著作権表示が必要な曲の鼻歌バージョンが入っており、その表記方法で迷っています。

「鼻歌部分の曲:著作権所持者の名称」
という形を英語表記しようとしています。
0169168 (ワッチョイ 5d9d-By/s)
垢版 |
2021/08/05(木) 06:31:40.12ID:FVMdlBD50
「鼻歌:歌唱者の名称」
という表示も同時にする予定なので、こちらは
「Humming:歌唱者の名称」
で良いでしょうか?
0170名無しさん@英語勉強中 (ワッチョイ d689-Ln56)
垢版 |
2021/08/05(木) 06:42:17.39ID:OVW8sZy70
>>168 >>169
(1) 「鼻歌部分の曲:著作権所持者の名称」
Hummed tune: Copyright held by Janet Jackson

(2) 「鼻歌:歌唱者の名称」
Hummed tune: performed by Lady Gaga
0171名無しさん@英語勉強中 (ワッチョイ 015f-i0sM)
垢版 |
2021/08/05(木) 07:23:24.80ID:qKKJ87EB0
3人1組の歩兵部隊20チームが争い合うゲームをしています
「我々は複数の敵部隊に囲まれている。位置を変えましょう。」をDeepLに掛けると
「We're surrounded by multiple enemy units. Let's change our position.」と出てきます

これをできるだけ短い言葉で伝えたい場合、1文目は「multiple units」のみでいいかなと思っているんですが
こういう場合の敵部隊ってforceとかtroopではなくunitで大丈夫でしょうか?
また、2文目をこれ以上縮める方法はあるでしょうか?
よろしくおねがいします
0172名無しさん@英語勉強中 (ワッチョイ d689-Ln56)
垢版 |
2021/08/05(木) 07:47:55.38ID:OVW8sZy70
>>171
(1) We're surrounded by multiple enemy units.

by multiple units だけだったら、「味方の部隊に囲まれている」という
意味に解釈できなくもない。だから enemy を入れた方がいいと俺は思う。
multiple の代わりに some を使ってもいいと思うし、その方が短い。
あなたはどうやら、できるだけ短い英文を書かないといけないような
立場に立たされているんだろうな。

We're surrounded by some enemy units.
We're surrounded by some enemy troops.
We're surrounded by some enemy forces.
どれでもいいと思うけど、units が短くていいな。

(2)
Let's change our position. については、
Let's get out of here. とすれば短くなる。
0173名無しさん@英語勉強中 (ワッチョイ faf0-IKHw)
垢版 |
2021/08/05(木) 08:05:50.12ID:FsF/EU/b0
>>171
Besieged! Move immediately!
0174名無しさん@英語勉強中 (ワッチョイ ce30-60o5)
垢版 |
2021/08/05(木) 08:24:31.72ID:KtAMU1jq0
>>171
そのゲームをよくわかっている人に通じればいい発言で文が多少不自然でもいいならいろいろ省略できそう
・囲まれているsurrounded だけである程度たくさんいるのは当たり前なのでsomeはなくてもいい
・enemyといえばunitとか補わなくても通じる
もしそうならWe're surrounded by enemies.でいいかも
位置を変えましょうは逃げましょうと言い換えてもいいならLet's run awayで
0175名無しさん@英語勉強中 (ワッチョイ d689-Ln56)
垢版 |
2021/08/05(木) 08:59:39.64ID:OVW8sZy70
>>173
うわっ、最高だ。参ったな。質問者は、この回答を全面的に信じて、
俺の回答は完全に無視してくれていい。
0176名無しさん@英語勉強中 (ワッチョイW a65d-v7Wr)
垢版 |
2021/08/05(木) 11:37:13.04ID:uRGFO3OP0
下の2つのどちらを信じればいいのか迷っています。

フォニックスっていう英語圏で生活する子どもたちが正しい発音を身に着けるために作られた、英語の読み書きのルールがあるそうなのですが。
下のサイトの発音を参考にするとCANONをクアヌアヌと発音することになるので疑問に感じています。


https://egao21.com/sp/01/alphabet/phonics_1.html

こちらでは「N」は「ン」と読んでいます。
https://allabout.co.jp/gm/gc/189300/
0177名無しさん@英語勉強中 (ワッチョイW a65d-v7Wr)
垢版 |
2021/08/05(木) 11:54:33.05ID:uRGFO3OP0
>>176
このコメントを打ったあとに見た知識ですが、
英語は綴りと発音の規則性が弱い言語
ということなので、フォニックスとしてどっちが正しいかは英語を学習するうえで重要そうではないですね、すみません。
0179名無しさん@英語勉強中 (ワッチョイ 4a8a-e6v6)
垢版 |
2021/08/05(木) 12:57:34.35ID:zCamMpDz0
否定疑問文に答える時のYES/NOについて
"Don’t you like sushi?"と聞かれた場合、好きならYes、嫌いならNo
つまり相手の質問が正しい時の回答はNoになると習いますが
アメリカドラマ見てるとNoではなくYeahで答えているシーンがありました
YeahはYesとは違う意味にこの場合なるのでしょうか?
0181名無しさん@英語勉強中 (ワッチョイ d689-Ln56)
垢版 |
2021/08/05(木) 14:45:23.98ID:OVW8sZy70
>>179
Yeah と Yes とは同じような意味であるはずだけど。
0183名無しさん@英語勉強中 (ワッチョイ bde2-nzU0)
垢版 |
2021/08/05(木) 15:24:12.86ID:Fq5tR3uc0
相手の言う内容丸ごと受けて、それがその通り、という意味ならyeah, that's rightになる。
0185名無しさん@英語勉強中 (ワッチョイ bde2-nzU0)
垢版 |
2021/08/05(木) 19:57:14.32ID:Fq5tR3uc0
SVO
0186名無しさん@英語勉強中 (ワッチョイ eb32-WmgW)
垢版 |
2021/08/06(金) 00:26:34.14ID:7siogZcI0
>>179
紛らわしさを避けるために
Yes(Yeah), I do like sushi.
とdoを強調して加えることはあるな
0188名無しさん@英語勉強中 (ワッチョイ eb89-NW/4)
垢版 |
2021/08/06(金) 04:23:01.60ID:GLOdjFEb0
They discussed topics as diverse as baseball and architecture.

= They discussed topics (that are) as diverse as baseball and architecture.

形容詞の後置修飾ってやつかな?
0190名無しさん@英語勉強中 (スプッッ Sd73-Pxyi)
垢版 |
2021/08/06(金) 07:19:24.62ID:wOqJSeu0d
>>187
下の表現でも文法的に問題ない
0191名無しさん@英語勉強中 (ワッチョイ b35f-mBWR)
垢版 |
2021/08/06(金) 08:04:23.35ID:Sd0miiy50
He was the only person ingenious enough to repair the machine.
彼はそのマシンを修理するのに十分な程に器用な唯一の人物だった

https://ameblo.jp/centum100/entry-12405159709.html
この記事によれば、後ろから名詞を修飾する場合は、that isやwhich isが省略されていると考えればよいということですが、上述のingeniousもその用法で合っていますか?

また、この記事には

> ただし、いくつかの形容詞は、一語であっても意味によって名詞を後ろから修飾するものがあるので、別途覚えておく必要があります。

という記載がありますが、これはまさに上述のenough等を指しているのでしょうか?
加えて、もしenough以外に一語で後ろから名詞を修飾する形容詞をご存知であれば教えていただきたいです。
0192名無しさん@英語勉強中 (ワッチョイ b15a-lZJ8)
垢版 |
2021/08/06(金) 08:10:41.08ID:XVTx+HCw0
present
0193名無しさん@英語勉強中 (ワッチョイ eb89-NW/4)
垢版 |
2021/08/06(金) 09:24:54.07ID:GLOdjFEb0
>>191
available もそうだな。

他にもいろいろあるけど、それを検索してみたら、次のような解説文がみつかった。

In obscure but irreplaceable phrases: battle royal, body corporate, body politic, corporation sole, fee tail, heir apparent, heir presumptive, knight errant, letters patent,[2] letters testamentary, to trip the light fantastic, time immemorial, treasure trove[3]

In professional or honorary titles: bishop emeritus, professor emeritus, attorney general, consul general, governor general, postmaster general, surgeon general, Astronomer Royal, Princess Royal, airman basic, minister plenipotentiary, minister-president, notary public,[4] poet laureate, president-elect, prime minister-designate, prince regent, sergeant major, queen consort, queen regnant, prince consort, directorate-general, director-general, etc.

In heraldry: dexter and sinister (as in bend dexter, bend sinister), and several referring to attitude, as in eagle displayed, lion passant guardant, griffin rampant, phoenix rising, bird vigilant, etc.

https://en.wikipedia.org/wiki/Postpositive_adjective
0194名無しさん@英語勉強中 (ワッチョイ b35f-mBWR)
垢版 |
2021/08/06(金) 10:17:20.33ID:Sd0miiy50
>>192,193
ありがとうございます

Postpositive adjectiveという名称だったんですね
名称が判明して色々と調べられそうで助かりました
感謝します
0196名無しさん@英語勉強中 (スプッッ Sd73-Pxyi)
垢版 |
2021/08/06(金) 10:53:18.87ID:wOqJSeu0d
>>191
そもそも、その文での enough は形容詞でなくて副詞だが・・

> ただし、いくつかの形容詞は、一語であっても意味によって名詞を後ろから修飾するものがある

これは、限定用法と叙述用法で意味の異なる形容詞ということだろうから、そのようなキーワードで検索すれば
色んな解説サイトがヒットする

例えば一番上に来たのはこれ
https://english-lab-japan.com/archives/2499
0197名無しさん@英語勉強中 (ワッチョイ 613b-NW/4)
垢版 |
2021/08/06(金) 11:00:47.74ID:b2CYW8qK0
>>187
as 形容詞 a 名詞 asとは言いますが、
as 形容詞 複数名詞 asという形は避けたほうが無難だった気がします
でもas many 複数名詞 asなら大丈夫らしいのでややこしいですね・・・

>>291
ingeniousが後ろにつく形容詞というわけではなくて、こういうときは名詞+形容詞+enough to...という語順になるっぽいですよ

He is a soldier brave enough to attempt it.「彼はそれをやって見るような勇敢な軍人だ。」(ジーニアス英和)
=He is a brave enough soldier to attempt it.の語順でもいいみたいです。
0199名無しさん@英語勉強中 (ワッチョイ a196-vNQU)
垢版 |
2021/08/06(金) 11:31:52.48ID:+7QBisgZ0
>>197
ありがとうございます
一つ疑問に思ったんですが、このasas構文は何と何を比較しているんでしょうか?
完全な文になおすと、
They discussed topics as diverse as they discussed baseball and architecture topics.
という文になるんですかね?
0200名無しさん@英語勉強中 (ワッチョイ 613b-NW/4)
垢版 |
2021/08/06(金) 13:45:37.70ID:b2CYW8qK0
>>198
ロイヤル英文法には
「名詞を修飾する形容詞にas, how, so, tooがつくときは、冠詞a[an]は形容詞の後に置かれる。一般に堅い言い方になる。また、名詞の前にa[an]がついていないときは用いられない。」
と書いてあったので、原則as 形容詞 複数名詞 asは使わないほうがよいと思いました。
>>199
文法的に考えると頭がこんがらがってきたので、たぶんこれはas many asとかas often asみたいな感じで使われてるんだと思います
例:She visits Paris as often as ten times in a year. 「彼女は1年に10回もパリを訪れる。」(as以下が言いたいことで、as often asの部分で強調しているニュアンスとでもいえばいいんでしょうか…)

こう考えると>>198さんのご指摘の通り、as diverse topics as...という語順で使うのも一理あって、間違いとは言い切れない気がしてきました。
0201名無しさん@英語勉強中 (スプッッ Sd73-Pxyi)
垢版 |
2021/08/06(金) 14:21:27.31ID:wOqJSeu0d
>>200
自分でも調べてみたが、文法的には確かにあなたの言う通りだった。

"as diverse topics as" で検索すると信頼のおけそうなサイトでの使用例が沢山出てきて
いずれも「as以下で多岐にわたる複数のトピックスを例示」するのに用いる一見定型的な表現のようにも思える。

・We cover ★as diverse topics as★ the chemical attack in Syria, LGBTQ rights in the United States, the intersection of HIV and human rights, and the issue of child marriage around the world. (The University of Alabama at Birmingham)
・It takes up women's history writ large, focusing on ★as diverse topics as★ the Amazons and Virginia Wolff. (Harvard University)
・The Ladybug podcast hosts discussed their experiences with “conference-driven development” preparing and delivering presentations on ★as diverse topics as★ debugging, GraphQL and Gatsby.(Nasdaq)
・I tried to write about ★as diverse topics as★ possible, ...(UC Berkeley)
・Current projects he is working on are concerned with ★as diverse topics as★ public homoeroticism, vaporwave, and radical leftwing activism.(University of Southern California)
・They cover ★as diverse topics as★ distance education; teacher talk in preschool settings; the problems of young aboriginal children and children who live in caravan parks or isolated areas.(オーストラリア学術論文)
0202名無しさん@英語勉強中 (スプッッ Sd73-Pxyi)
垢版 |
2021/08/06(金) 14:22:56.13ID:wOqJSeu0d
>>200
(続き)
しかし、PEU(4th)では、次のように書いてあり、「単数」の場合のみに認められた語順で、「複数」の場合はNGのようだ。
(複数形である★の部分の記述が重要)

After as, how, so, too and this/that meaning so, adjectives go before a/an.
This structure is common in a formal style.
as/how/so/too/this/that + adjective + a/an + noun
I have as good a voice as you.
She is too polite a person to refuse.

The structure is not possible without a/an.
★Those girls are too kind to refuse. (NOT They are too king girls to refuse.)★

なので、"as diverse topics as"は、文法的には厳密にはNGだが、よく間違えられて用いられる表現なのかもしれない。
0204名無しさん@英語勉強中 (スプッッ Sd73-Pxyi)
垢版 |
2021/08/06(金) 16:51:55.14ID:wOqJSeu0d
She is too kind a girl to refuse.
*They are too kind girls to refuse.

確かに、上がOKなら、下でも良さそうなのにな
以下は、複数の場合でも、many なら良いが numerous だと不可というCaGELの例

a. He made [so many mistakes].
b. *He made [so numerous mistakes].

CaGEL見ると色々と説明しているが、文法マニアではないので、これ以上深入りはしないでおこう。
0207名無しさん@英語勉強中 (ワッチョイW 9bc5-limt)
垢版 |
2021/08/06(金) 23:29:53.70ID:xuLgdiMe0
187 adjectives: position after as, how, so, too

After as, how, so, too and this/that meaning so, adjectives go before a/an. This structure is common in a formal style.
as/how/so/too/this/that + adjective + a/an + noun
I have as good a voice as you.
How good a pianist is he?
She is too polite a person to refuse.
I couldn’t afford that big a car.

がpeuに載ってるよ
0210名無しさん@英語勉強中 (ワッチョイ b15a-lZJ8)
垢版 |
2021/08/06(金) 23:45:02.67ID:XVTx+HCw0
>>206
文法辞典みたいに調べ物がある時に見る物であって、通読するような代物じゃないでしょ。
0211名無しさん@英語勉強中 (ワッチョイW 9bc5-limt)
垢版 |
2021/08/07(土) 09:43:02.74ID:fHZlbFGg0
peuってtesolの課程をやって終えた人がめちゃくちゃ勧めてたんだよね。
tesol のための必読書だと言ってた。どう使うのかと尋ねたらわからないところ
を調べるんだとは言ってたけど。
0215名無しさん@英語勉強中 (アウアウウー Sa55-lHUs)
垢版 |
2021/08/07(土) 10:31:16.75ID:RBnCORDZa
I hope you find this information helpful
これを訳すとこの情報があなたの役に立つことを願ってますになるそうですが、information helpfulは形容詞が名詞の後ろから修飾してるのは駄目なのではないですか?
もし、その置き方で問題ないなら「役に立つこの情報をあなたが見つけることを願ってます」になりそうですが、どのように解釈していけばよろしいでしょうか?
0217名無しさん@英語勉強中 (ワッチョイ b15a-lZJ8)
垢版 |
2021/08/07(土) 10:56:58.47ID:vLV/tIaw0
大半はわかり切った内容しか書いてないし、元々usage guide本であって通読学習用に書かれた参考書の類ではない。そんなもん頭から読んで悦に入ってるのは何か勘違いしてる暇な日本人英語オタクくらいのもの。
0218名無しさん@英語勉強中 (アウアウアー Saab-xSQA)
垢版 |
2021/08/07(土) 12:35:55.42ID:NIlnoq/Ja
>大半がわかりきった内容

だからこそ通読して知らないところは潰していくべきって話なのもわからないのか…

あと君みたいな英語できない人にまで読めとは誰も言ってないからねw
0220名無しさん@英語勉強中 (ワッチョイ b15a-lZJ8)
垢版 |
2021/08/07(土) 13:00:01.20ID:vLV/tIaw0
いつまでたっても「参考書ルート」の延長線で受験英語の「おべんきょう」してる人間だと、タイトルからして思いっきり実用書なPEUまでもが「ネクステ」だの「解釈教室」だのの受験本の上級編的な珍妙な崇められ方をするんだろう。

Introductionに使用法が普通に書いてある。

PEU3版 The purpose of this book: When do we use past tenses to be polite? ... Practical English Usage is A GUIDE TO PROBLEMS OF THIS KIND. It deals with over 600 points which regularly cause difficulty to foreign students of English....

Being a REFERENCE book, it contains information at various levels....

Problems are mostly explained in short separate entries: the book is MORE LIKE A DICTIONARY than a grammar in form.

4版からテーマ別にまとめられてある程度系統立った体裁になりはしたけれど、元々はアルファベット順に項目がズラーっと並んでいた代物で基本部分は変わらない。
0228名無しさん@英語勉強中 (スップ Sd33-Vvx7)
垢版 |
2021/08/09(月) 22:50:32.03ID:v1rbFmjvd
よろしくお願いします
初歩的な質問で恐縮ですが、

私は車の特徴を彼に話した。
I described the car to him.

上記の文は普通の第三文型だと思いますが、以下のように書き換えは出来るものでしょうか?

I described to him the car.
0229名無しさん@英語勉強中 (スップ Sd33-Vvx7)
垢版 |
2021/08/09(月) 23:02:00.16ID:5uGgEO6Td
Oがwhat節なら辞書に例文が出ているのですが…

I described to him what the car was like.
0231名無しさん@英語勉強中 (ワッチョイ 613b-NW/4)
垢版 |
2021/08/09(月) 23:13:44.06ID:AHyp/VjI0
I described to him the car which I bought the day before yesterday.
のように目的語の部分が長くなってしまったときは、to himを先に持ってきたほうが視認性が良くなると思います。
0232名無しさん@英語勉強中 (ワッチョイ 31e1-C+sF)
垢版 |
2021/08/10(火) 12:05:47.42ID:ny5ZiXlN0
五輪の試合を棄権した選手が日本への感謝の意を伝えるツイートで
some of、not以下の用法がいまいちわからないので
解説してくださる方!お願いします!
I’ll forever be thankful for Junetendo ❤
for allowing me to come train separately to try to get my skills back.
The japanese are some of, if not the sweetest people I’ve ever met.

日本人は今まであった中で最高に親切な人々ですっていう意味だそうですが
文法的に、お願いします!
0234名無しさん@英語勉強中 (スプッッ Sd73-Vvx7)
垢版 |
2021/08/10(火) 13:10:33.54ID:N9MwphFid
>>230
>>231
どうもありがとうございました
おかげ様で大変すっきりしました
0237名無しさん@英語勉強中 (ワッチョイ 613b-6hcM)
垢版 |
2021/08/10(火) 15:14:58.11ID:U1nOcnPB0
if not
perhaps even (used to introduce a more extreme term than one first mentioned).
"hundreds if not thousands of germs"
if notで検索したら一番上にこのオックスフォードの定義がでてきました
今はこっちの意味のほうが主流みたいですよ
0241名無しさん@英語勉強中 (ワッチョイ 1b33-mBWR)
垢版 |
2021/08/10(火) 23:23:35.67ID:cDEyBO2Y0
>>240
「とは言わないまでも」しか書いてないとしたら、その辞書はもう古いと思いますよ

237さんも言っているように、「A if not B」=「A、いやおそらくはB」の方が多いくらいでしょう
0242名無しさん@英語勉強中 (ワッチョイ 613b-NW/4)
垢版 |
2021/08/11(水) 00:00:36.69ID:aVTNAMv80
もし〜でなければという純粋な省略のif notを除けば
if notの解釈としては大きく分けて3つあるようです

@「いや〜かもしれない」
A「〜ではないとしても」
B「〜はともかく」、「〜はさておき」(意味的にはAに近いのでA'といったほうが適切かもしれません)
They welcome China's investment, if not influence.
「中国の影響はともかく投資は歓迎している。」
0248名無しさん@英語勉強中 (スプッッ Sd73-Pxyi)
垢版 |
2021/08/11(水) 20:42:36.58ID:SHQdBs11d
全5種類であることを表したいなら a full set of five 〜s
例えばそれが本の種類なら a full set of five books 

(店頭で何か食べ物なんかを一つずつ全種類欲しい場合は
 Can I have one of each (of these five flavors)?
 とか普通は言うとは思うけど)
0250名無しさん@英語勉強中 (スプッッ Sd73-Pxyi)
垢版 |
2021/08/11(水) 20:50:12.08ID:SHQdBs11d
本の種類というのは例としてあまりよくなかったかも
0252名無しさん@英語勉強中 (ワッチョイ 613b-NW/4)
垢版 |
2021/08/12(木) 00:40:15.84ID:mKgAc5N/0
A sound mind in a sound body, is a short, but full description of a happy state in this world:
he that has these two, has little more to wish for; and he that wants either of them, will be little the better for anything else.

最後のlittle the betterはどう解釈すればいいんでしょうか?
0253名無しさん@英語勉強中 (ワッチョイ eb89-NW/4)
垢版 |
2021/08/12(木) 04:22:22.45ID:4ktu5N1o0
... he that wants either of them, will be ★little the better★ for anything else.

その(健やかな体と健やかな心のうちの)いずれかが欠けている人は、
他に何を持っていたとしても(ほとんど)良くはならない

(1) little the better for [something]
これは
(2) none the better for [something]
に似ている。(2) については、「ジーニアス英和、第5版」には載っている。
他のたくさんの辞書にも載っているだろう。
0254名無しさん@英語勉強中 (ワッチョイ eb89-NW/4)
垢版 |
2021/08/12(木) 04:28:06.50ID:4ktu5N1o0
... he that wants either of them, will be ★little the better★ for anything else.

その(健やかな体と健やかな心のうちの)いずれかが欠けている人は、
他に何を持っていたとしても(ほとんど)良くはならない

(1) little the better for [something]
これは
(2) none the better for [something]
に似ている。(2) については、「ジーニアス英和、第5版」には載っている。
他のたくさんの辞書にも載っているだろう。
0255名無しさん@英語勉強中 (ワッチョイ eb89-NW/4)
垢版 |
2021/08/12(木) 04:30:25.79ID:4ktu5N1o0
(1), (2) に似た形として
all the better, all the more, all the worse などというフレーズもあるわな。
一般的に、
[副詞] + the + [比較級]
というフレーズがいろいろと出回っている。
0256名無しさん@英語勉強中 (ワッチョイW 9949-rSax)
垢版 |
2021/08/12(木) 05:08:43.87ID:NktURL4w0
https://transcripts.cnn.com/show/sn/date/2021-03-03/segment/01
>Experts say it looks like a new building has appeared at the facility right at the spot where two underground tunnels lead to it.

このwhereは関係副詞でしょうか?
その場合、関係副詞が前置詞+関係代名詞の形に置き換えられるとすると、
which two underground tunnels lead to.
となりitが余計に思えます。
引用元の文章をどのように考えればいいか教えて下さい
0257名無しさん@英語勉強中 (オッペケ Sr5d-qgnb)
垢版 |
2021/08/12(木) 06:37:28.70ID:HOna4XyJr
このasってどういった意味でしょうか

・Imagine a mother giving her two-year-old daughter a bath, allowing the girl to daydream with her bath toys 《as》 she makes up stories and learns to be alone with her thoughts, all the while knowing her mother is present and available to her.
0258名無しさん@英語勉強中 (ワッチョイ eb89-NW/4)
垢版 |
2021/08/12(木) 06:49:31.53ID:4ktu5N1o0
>>256
(1) 原文: at the spot where two underground tunnels lead to it.

これは、
(2) at the spot two underground tunnels lead to.
(3) at the spot which (または that) two underground tunnels lead to.
(4) at the spot to which two underground tunnels lead to.

と同じような意味だけど、(2), (3), (4) では書いてるうちに(しゃべっているうちに)
あとで to をつけるべきだったかどうか(その前に to which と書いたか、あるいは
which だけを書いたか、あるいは口頭で言ったか)を忘れてしまって、うっかり
to が不要なのに入れてしまったりしてややこしいから、where を使うことが
少なくとも口語では多いのではないかという気がする。そしてこの where
を使うと、
(5) (*)at the spot where two underground tunnels lead to
と書いたら文法的に間違いだし、かといって
(6) (*) at the spot where two underground tunnels lead
と書いたりしたら、最後の lead が何を意味しているのか一瞬にして理解できる
というわけでもないかもしれないと不安になるから、仕方なく
原文 (1) のような書き方が許容されるようになってきたのではないかと推察する。
0259名無しさん@英語勉強中 (ワッチョイ eb89-NW/4)
垢版 |
2021/08/12(木) 06:50:42.01ID:4ktu5N1o0
訂正
間違い --- (4) at the spot to which two underground tunnels lead to.
訂正後 --- (4) at the spot to which two underground tunnels lead.
0260名無しさん@英語勉強中 (ワッチョイ eb89-NW/4)
垢版 |
2021/08/12(木) 07:01:20.52ID:4ktu5N1o0
>>257
>>allowing the girl to daydream with her bath toys 《as》 she makes up stories and learns to

as は、daydream するのと makes up stories and learns to するのとが同時(またはそれに違い)
ということを表す言葉だろう。それを直訳に近い形でありながら自然な日本語に
するのは難しいかもしれんけど。意訳でも直訳でも、自由に訳したらいいと思う。
俺ならば、前から訳して、次のように訳すかな?

・Imagine a mother giving her two-year-old daughter a bath,
   母親が2歳の娘を入浴させているところを想像してみよう。

allowing the girl to daydream with her bath toys
   それで、娘が風呂場用のおもちゃをいくつか使って「ごっこ」遊びをし始めるのだが、
  
《as》 she makes up stories and learns to be alone with her thoughts,
   娘はいろんな作り話をして、一人だけで想像の世界にどっぷり浸かるようになるけれども、

all the while knowing her mother is present and available to her.
  その間も母親はきちんとそばにいることが娘にはわかっているのだ。

あくまでも意訳だ。
0262名無しさん@英語勉強中 (ワッチョイ 13f0-Lto6)
垢版 |
2021/08/12(木) 08:21:46.23ID:a6pi2Ch60
>>256
the facility ミサイル発射基地その他がある施設(全体)
the spot その中の一箇所、一地点

で、筆者の頭の中では lead to it の it は the facility を意味しているとも
考えられるかも。   説明を加えて、ぎこちないが・・・和訳してみると、

「2本の地下トンネルの到達点である、まさにその施設と(トンネル)の接続箇所と
 寸分も違わない地点に、新たな建物が現れたようである」

*(文法、文脈の観点から可能性ゼロ・レベルで間違っていたら、・・・ゴメンな)
0263名無しさん@英語勉強中 (アウアウクー MMdd-COCn)
垢版 |
2021/08/12(木) 11:05:58.21ID:SLrcZJmBM
米・英において日本でよく用いられる「○○スペシャル」(たとえばプロレスの技名とか)というものは、
語感・ニュアンスは通じるものですか?

たとえば山田さんがプロレス技を繰り出すときに相手に向かって「It's Yamada special !!」という吹き出しを付けるとき、
これはアメリカ人の読者に通じるかな
0267名無しさん@英語勉強中 (ワッチョイ 613b-6hcM)
垢版 |
2021/08/12(木) 13:58:05.96ID:mKgAc5N/0
>>253
比較級にtheがついているのが気になっていたんですがやはりちゃんと理由があったんですね
little the betterという形で出てくるのは初めてだったので確証が持てず質問させていただきました。

wantについても普通の欲しいという意味でも通じなくはないですが、不足しているととったほうが確かにいいですね
ありがとうございました。
0268名無しさん@英語勉強中 (ワッチョイ 1bd7-uBeU)
垢版 |
2021/08/12(木) 14:58:27.90ID:7xDBZfRf0
否定のnoについて質問が有ります。

I have no such right.(私にそんな権利はない)
という発言に対して、
「そうだ。ないよ。」
と言いたい場合、
Yes, Noどちらで答えるのでしょうか?
noの場合もnotと同じく否定文(意味的には文否定ですから)
と捉えて、No you don't.でいいのでしょうか?
noの応答はnotに変換すると言った(機械的な)ルールってありましたっけ?
0269名無しさん@英語勉強中 (ワッチョイ eb89-NW/4)
垢版 |
2021/08/12(木) 16:36:45.14ID:4ktu5N1o0
>>268
俺ならそういうとき、次のように言う。

You're right. You have none.

英語ネイティブでも、否定疑問に対して Yes. で答えるべきか No で答えるべきか
戸惑って、いろいろと揺れてしまうそうだから。
0270名無しさん@英語勉強中 (オッペケ Sr5d-qgnb)
垢版 |
2021/08/12(木) 17:07:01.08ID:TE2jrEqSr
My parents and I have a warm relationship, even though, or perhaps because, I don’t speak to or visit them frequently; until my most recent trip there, 《the previous July》, I hadn’t seen them in six years.

このthe previous Julyは文法的にはなんと説明されるのでしょうか
意味は分かっているつもりです
同格や挿入,副詞的対格などを複数の文法書で見ても類例といえるものが見つかりませんでした
0273268 (ワッチョイ 1bd7-QnL3)
垢版 |
2021/08/12(木) 17:51:12.73ID:7xDBZfRf0
>>269
レスありがとうごじます。
そのような言葉も一つの手ですね。none=no such rightですか。
No, you don't (have any such right.)
というのもno=not anyというのを考えれば言えると思うのですが、英語圏を検索してもあまり例が
でてきません。あんまり考えすぎず、No,you don't で通じるのでしょうか・・・
0280名無しさん@英語勉強中 (オッペケ Srf1-mh3b)
垢版 |
2021/08/13(金) 03:07:40.82ID:FlWBwWvIr
A, Bで単にBがAの日時を表すような同格って存在するんでしょうか
少し今回のものと異なりますがTwitterで見かけた佐藤誠司さんの独自?の説明は近いようには思えますね。現代英文法講義などで記述を見つけられるといいのですが

(佐藤誠司さんのツイートより)
The Osaka government plans to ask bars and restaurants in the city to close an hour earlier than now, at 8 p.m.(同上)
学校文法に沿って考えた場合,「an hour earlier than nowとat 8 p.m.は同格だ」という説明は成り立ちません(同格とは名詞を並べたものと定義されるから)。佐藤文法では,at 8 p.m.を「補足説明用法の副詞句」と説明します。名詞にも同じ用法はあり,例えばShe got angry ー a natural reaction. のようなもの。これはShe got angry, which was a natural reaction. と同義であり,名詞句が前文に補足説明を加えている(つまり同格句ではない)ということ。まとめると,これらの文は「前の陳述に補足説明を加える働きをする語句」を含んでおり,名詞・形容詞・副詞のどの品詞もその働きを持つ,ということです。これは一例ですが,このように現在の学校文法は,英文中で頻繁に見られる形を十分に説明できていない面があるように思います。
0281名無しさん@英語勉強中 (ワッチョイ 2e89-pBez)
垢版 |
2021/08/13(金) 04:28:16.41ID:AU3A+whZ0
>>270
>> until my most recent trip there, 《the previous July》, I hadn’t seen them in six years.

"until 名詞句, 副詞的目的格, S V" っていう形でしょ?ということは、
"until my visit last year" とか "until my trip there" とかいうのと同じで、
"until 名詞 + 副詞" という形に過ぎないから、別に難しい構造ではないように
俺には思えてしまう。この下の用例と似たような感じ。

参考:

(1) 1994 · ‎Economic assistance, American
I might tell you that in ★my recent trip last month to Nicaragua★ , in not only my public remarks , in a speech I made , but in my conversations with all of ...
(Google Books より)

(2) Senator Holland , when I returned from ★my recent trip last year★ , I brought all that to the attention of Mr. Wilson . I sent him excerpts from my diary ...
(Google Books より)
0282名無しさん@英語勉強中 (ワッチョイ 2e89-pBez)
垢版 |
2021/08/13(金) 04:31:03.45ID:AU3A+whZ0
>>270
>> until my most recent trip there, 《the previous July》

俺が言いたかったのは、until my most recent trip there と the previous July とが同格だとか
対応しているとかいうものではなくて、until は無視すれば
my most recent trip there という名詞句に対して、the previous July という副詞句が
後置修飾しているに過ぎないということ。
0283名無しさん@英語勉強中 (アウアウウーT Saa5-/sHu)
垢版 |
2021/08/13(金) 04:46:53.13ID:s6kZD7BZa
副詞に同格という概念はないだろう
そもそも単に文の説明をより詳細に付け加えてるだけなんだからさ
0285名無しさん@英語勉強中 (オッペケ Srf1-mh3b)
垢版 |
2021/08/13(金) 10:58:45.31ID:jAI/SjVUr
>>270の質問者です
寝てから考え直すとスッキリしました
until my most recent trip thereが"I travel there"のような動作を表す名詞句で、それに対してthe previous Julyという時間を表す名詞句が同格?で修飾するというのは微妙だと思ったのですが、そもそもuntil の目的語になっている時点で"my most recent trip there"は動作というより単に時間表現の一種なわけで、the previous Julyが同格として説明してる(換言してる)ということでいいかもしれませんね
0286名無しさん@英語勉強中 (オッペケ Srf1-mh3b)
垢版 |
2021/08/13(金) 11:25:24.91ID:jAI/SjVUr
>>282
ありがとうございます
I travel thereがmy trip thereのようになるのは全く違和感がないのですが、カンマがあるせいでthe previous Julyが副詞的対格としてtripを修飾する解釈が成り立たない気がしました
少し考えてみます
0289名無しさん@英語勉強中 (ワッチョイ 4630-nhnz)
垢版 |
2021/08/13(金) 14:38:15.21ID:CIoRHJy70
「(不安や緊張で)張り詰めていたが(最後の一押しで)ついに心が折れて恐怖に飲まれた」
のような内容を英語で言うとどうなりますか?
一文でもそうでなくてもいいです。カッコ内はあってもなくてもかまいません。
口語的なものと堅いものがあればどちらもお願いします。
0290名無しさん@英語勉強中 (ワッチョイ 2e89-pBez)
垢版 |
2021/08/13(金) 17:54:25.65ID:AU3A+whZ0
>>289
俺にとっては難しい課題なので、あまり自信がないので、参考程度に読み流してほしい。

>>(不安や緊張で)張り詰めていたが(最後の一押しで)ついに心が折れて恐怖に飲まれた

I was overly tense. Then the last straw broke my back and fear engulfed me.
0292名無しさん@英語勉強中 (ワッチョイ be55-cJil)
垢版 |
2021/08/13(金) 23:40:35.78ID:AFW7qhNe0
すみません、探しても見つからないので教えてください
prefix suffixで語彙を増やしたいのですが、それらの種類じゃなくて
ひとつの単語での活用形を一覧で把握したいのですが、出てこなくて

kind -> kindness , kinder とかはありますが、 kindth, kindee などは無い
というのを一覧で学びたいのですが、そういう情報の探し方とかわからず。
0293名無しさん@英語勉強中 (ワッチョイW 06c5-wONL)
垢版 |
2021/08/14(土) 05:10:22.86ID:tBVVFgcZ0
>>292
erをつけるかmoreをつけるかは文法書。
esをつける場合も、ies にするか、yesにするかも文法書。
weとかstは別の単語になるでしょう。employeeとか。
kinderがあるかどうかは辞書。
英語って体系的にできてないから文法だけで決まっているという
わけではないので。
0294名無しさん@英語勉強中 (ワッチョイ c2f0-sbZG)
垢版 |
2021/08/14(土) 08:01:52.19ID:vngpZyMc0
【・・・・・ニュースでもトピックスでも、同じような内容の複数の英文を聞いたり
 読んだりすることで、多様なパラフレーズの存在と可能性が実感でき、・・それが、
 
 記憶の襞に絡みついた一つの表現から解放され、(一時的な間違いは気にせず)
 学習者をより広くもっと自由な英語表現の世界へと導いてくれるかも知れない・・・】
(以下は参考英文)

https://www.realitycheckdaily.com/12-shocking-habits-of-successful-people/
https://officerspulse.com/12-shocking-habits-of-successful-people/

https://outlandisher.com/12-shocking-habits-of-successful-people/
https://www.salinews.com/2019/11/success-habits.html
http://strivelifedesign.com/2018/08/12/12-shocking-habits-of-successful-people/

(ニュアンスの違いはあるし、また一カ所間違いがあるが、中級学習者限定で)
0295名無しさん@英語勉強中 (ワッチョイ be55-cJil)
垢版 |
2021/08/14(土) 09:04:11.31ID:DaCFWc/x0
>>283
仰ることはごもっともなのですが、単にそれを一覧として持って、勉強したいだけなのです。
He is kindness といったことをやってしまったり、It is difficultyといったことを
別の単語でもやってしまうので、形容詞、名詞のprefixに何があるのかを
代表的なものでも把握しておきたいなと。

prefixの説明はあっても、それがそのベースとなる単語であるのか?
find -> findness? findable? findic? findful? findic?
そんなものがあるのかないのか、一つ一つ辞書を引くまえにおおよそのものを掴みたいなと
0297名無しさん@英語勉強中 (ワッチョイ 2e89-pBez)
垢版 |
2021/08/14(土) 10:14:33.04ID:DOYaYQyY0
>>find -> findness? findable? findic? findful? findic?
>>そんなものがあるのかないのか、一つ一つ辞書を引くまえにおおよそのものを掴みたいなと

find の派生語として、「そういうものはない」といちいち列挙してくれる本なんて
ないか、あるいは余程の暇人でない限りそんなものを作らないだろう。
第一、一つの単語について存在する派生語が仮に 5種類あるとしたら、
存在しない派生語は100種類くらいあると言ってもいい。

そもそも、「これとこれは存在する」と言って、存在する派生語を辞書などは
きちんと並べてくれているんだ。それを覚えさえすれば、「そこに書いていない派生語は、
ないのかもしれない」と思っておくしかない。そして何かの拍子に特殊な珍しい
派生語がちゃんと使われていることに気づいたら、改めてそれを追加して覚えるんだ。
0298名無しさん@英語勉強中 (ワッチョイ c2e5-YgaZ)
垢版 |
2021/08/14(土) 10:18:57.72ID:PRglJBec0
>>270 >>285
えっと…、そもそも、until の形容詞句がそこにあるのは、その形容詞句を強調したいから倒置してる。そして、July はどうでもいい付加情報なので継続的に挿入されているだけです。

構文の解釈で納得したいなら当てはまりそうなのを当てればいい。

until の形容詞句内の travel there と同格関係にあり、非制限同格と考えてもいいだろうし、副詞的目的格の挿入と考えてもいいだろうし。

名詞句、節が並んでいれば同格関係にあると言います。そして同格関係にあるフレーズを「説明する」機能です。ただそれだけです。
https://en.wikipedia.org/wiki/Apposition

例えばこういうのも実は同格関係です。fear と that 節。カンマが無いのは制限用法だからです。that 節がないと意味が分からないでしょ?君が示した文章は非制限なのでどうでもいい情報なのです。
文法書から同格と説明されている例文を引用:An Advanced English Grammar with Exercises
My fear that the bridge might fall proved groundless.
0299名無しさん@英語勉強中 (ワッチョイ be55-cJil)
垢版 |
2021/08/14(土) 11:22:47.94ID:DaCFWc/x0
>>297
そうですよね。これだけ調べてもなかったので、世の中にないのだろうなと思います。
取り急ぎエクセルでprefix/suffixをつなげてAPIに自動で投げるようなものを作って
存在しないようなものは削るといったものが作れたので、それでしばし自学します。

native(日英双方)と話してても「〜ってなんで無いんですか?」と聞いたら
「なんでなんすかねー」と説明も出来ないみたいですし、日本語で同じようなこと聞かれても
答えられないから、自分の単語の貯金作ってくしかないですね。

ありがとうございました。
0301名無しさん@英語勉強中 (ワッチョイ c93b-pBez)
垢版 |
2021/08/14(土) 14:45:45.20ID:bgq5qdHl0
同格は同じ格(役割)という意味ですよね
名詞を名詞で言い換えていて、かつ文法的な働きが一緒なら同格と言えると思います
そういう意味で>>278を書きました
前置詞の目的語となる場合に名詞の後ろに-aiがつくという規則があると仮定すると
until my most recent tripai there, the privious Julyai
こうなりますよね
0303名無しさん@英語勉強中 (ワッチョイ c2e5-YgaZ)
垢版 |
2021/08/14(土) 17:20:21.22ID:PRglJBec0
>>300
失礼、utnilは動詞又は文全体を修飾しているので副詞句です。

>副詞的目的格だとするとなにを修飾してるのでしょうか

my most recent trip (there) です。there も副詞的目的格で修飾してるものは同じす。
0305名無しさん@英語勉強中 (アウアウウー Saa5-pC9E)
垢版 |
2021/08/14(土) 17:42:38.26ID:LBij0uI1a
>>302
that…睡眠を回避すること
than以下…脳を半分ずつ休ませること(ざっくり言うと)

「ずっと起きてる方が、脳を片方ずつ眠らせるよりも容易ではなかったのだろうか?」
「恐らくそうなのだろう(=寝ずにいる方が難しい。)」
0308名無しさん@英語勉強中 (ワッチョイ 2ee1-yvwv)
垢版 |
2021/08/15(日) 15:46:24.78ID:wOxmOZG80
たとえば

allurementを修飾する形容詞で

miraculousが絶対に変だ?ということがあるのか
自由な表現の詩人なら使いうるのか?
調べたいときどうしますか?

アメリカの動詞とか形容詞の品詞ごとのコロケーション検索
https://inspirassion.com/en/adj/allurement
では
miraculous は出て来ませんが
使ってもおかしくないのか?
COCAではまだ調べてないけど
過去に使われたことがないだけで
使っても問題ないかどうか?は調べる方法はないですよねw
おかしいと思いますか?
0309名無しさん@英語勉強中 (ワッチョイ c633-/m/d)
垢版 |
2021/08/15(日) 16:08:35.67ID:9dYrd9i10
>>308
一番シンプルな方法は、ダブルクォーテーションで囲んでググる、だと思います
"miraculous allurement"

で、検索すると4件で、内容を確認すると妥当な用例は1つもありません
なので、「一般的に、miraculous allurementは変だ」と言えると思います
COCAでも同様です

「絶対に変といえるのか?」「自由な表現の詩人なら使いうるのではないか?」
「過去に使われたことがないだけで使っても問題ないのではないか?」
は悪魔の証明なので、結論は出ないでしょう

しっくりくる形容詞を探したいときは、たとえば"a * allurement"で検索したりします

また、「alluring miracle 名詞」「alluring miracle of 名詞」あたりなら用例が見つかるので、
こちら側の表現も検討します
0310三年英太郎 ◆3CZBjOt3.Y (ワッチョイ 4286-/m/d)
垢版 |
2021/08/15(日) 17:35:46.14ID:7hu1TuIj0
ググって出てこなかったら、
それは「よくある組み合わせでない」「googleでは検索されない」とは言えても、
「変だ」とまでは言い切れない。

「幽玄な白鷺の舞」はわたくしの感覚では全く変ではないが、
ググっても出てこないからね。

詩となれば、出来不出来を問わなければ、あらゆることが可能なので、その問いは意味がない。
変な語の組み合わせはシュルレアリスムで散々やられたし、
語の意味なんて関係ねーっていう音響詩というものさえある。
0311😏三年英太郎😉 ◆3CZBjOt3.Y (ワッチョイW 4286-ynEy)
垢版 |
2021/08/15(日) 17:53:27.25ID:7hu1TuIj0
わたくしのお友だちに Can I say "miraculous allurement" ?と聞いたら

You can…👍…but I would be interested in knowing it’s context…😁🤓
It’s not the most popular form of expression…😉…It sounds as if you’re reading a 19th.c. novel…🤔
It could mean:
‘An amazing chance meeting’…🤓

と申していたので、変ではなかろう。
0312名無しさん@英語勉強中 (ワッチョイ 2e89-pBez)
垢版 |
2021/08/15(日) 17:55:53.18ID:ghksIy4B0
a miraculous allurement という言い回しが英語ネイティブに何とか受け入れてもらえるかどうかについて

a miraculous charm
a miraculous beauty
a miraculous temptation
a miraculous attraction
a miraculous appeal
a miraculous charisma

上のように、allurement の類義語を入れて検索してみたら、かなりよさそうな
用例がそれぞれにたくさんついてくるので、"a miraculous allurement" も
悪くはないのではないかという気がしてきた。
0313名無しさん@英語勉強中 (ワッチョイ 4641-YgaZ)
垢版 |
2021/08/15(日) 23:29:55.58ID:Gu+P+SHB0
ネトゲやっていたらチンポをなめろビッチみたいな事言われたのですが
そこでペロペロとでも返そうかと思ったのですが英語で日本語のような擬音みないなものってあるのでしょうか
ぐぐった所 slurp と出てきましたがこれがそうなのでしょうか?これだけで通じるのでしょうか?
もう返す気はありませんがせっかくなのでよろしくお願いします
0314名無しさん@英語勉強中 (ワッチョイ c93b-pBez)
垢版 |
2021/08/16(月) 01:31:23.48ID:1kwIdir00
slurpだと飲み物を啜る音ですね
そういう文脈であれば下品ですがオシ○コを飲んでる音を表すときには使えるとおもいます
普通に *lick lick* でいいんじゃないでしょうか?
他にもsquelchやshlickなどは卑猥な音を表せるっぽいです
0315名無しさん@英語勉強中 (ワッチョイ 2e89-pBez)
垢版 |
2021/08/16(月) 04:52:00.61ID:FtErjoS80
何かを舐めるときの擬声語

べろり
*slurp* *shlurp*
SFX for lapping something up greedily, usually a motion done with the tongue.
See also *perori*.
http://thejadednetwork.com/sfx/browse/berori/

ぺろん
(1) *lick*;
(2) *rolling up*
*rolling down*;
(3) *flipping*
http://thejadednetwork.com/sfx/browse/peron/

ぺろぺろ
*lap lap* *lapping* *lick lick*
Like a dog or kitten
http://thejadednetwork.com/sfx/browse/pero_pero/
0318名無しさん@英語勉強中 (ワッチョイ 2e89-pBez)
垢版 |
2021/08/16(月) 06:25:32.27ID:FtErjoS80
peggin the fuck outta them
なるほど、peg なんていう動詞があるんだね。

peg
A term coined by sex advice columnist Dan Savage that refers to an act of love making that involves a woman with a strap-on dildo anally penetrating her male partner.
Jennifer pegged the shit out of John last night; I bet he won't be able to sit down for a week.
peg
A term coined by sex advice columnist Dan Savage that refers to an act of love making that involves a woman with a strap-on dildo anally penetrating her male partner.
Jennifer pegged the shit out of John last night; I bet he won't be able to sit down for a week.
0319名無しさん@英語勉強中 (ワッチョイW dd96-KWGR)
垢版 |
2021/08/16(月) 12:27:21.24ID:HK56wGqn0
The girl called Jane called on Jack.

この文は「Janeと呼ばれる少女はJackを訪問した」と訳されますが、
「その少女はJackを訪問したJaneを呼んだ」と読めない文法的な理由はありますか?
0325名無しさん@英語勉強中 (ワッチョイ e996-lbFy)
垢版 |
2021/08/16(月) 18:20:56.38ID:CI5dn4p/0
Between the two branches of the Party there is a certain amount of interchange, but only so much as will ensure that weaklings are excluded from the Inner Party and that ambitious members of the Outer Party are made harmless by allowing them to rise.

ここのso much asのところがよくわからないんですがどういう構造になっているんでしょうか?
以下は訳文です

党の二つの機関の間ではある程度の入れ替えはおこなわれるがそれも党内局から劣った者を排除し、また昇格させることによって党外局の野心家を無害化するためだけにおこなわれている。
0327名無しさん@英語勉強中 (ワッチョイ c93b-pBez)
垢版 |
2021/08/16(月) 18:29:56.44ID:1kwIdir00
>>325
certain amountをso much as以下で詳しく説明しています
「ある程度」といわれてもどの程度なのか知らない人にはわからないので範囲を限定しているんです
so much (amount of inter change) as 〜
また、このasは関係代名詞みたいな使われ方をしています
0328名無しさん@英語勉強中 (ワッチョイ 2e89-pBez)
垢版 |
2021/08/16(月) 18:30:47.07ID:FtErjoS80
>>325
only so much as will ensure
(1) that weaklings are excluded from the Inner Party
and
(2) that ambitious members of the Outer Party are made harmless by allowing them to rise.

only so (= as) much as will ensure that S1 V1 and that S2 V2.

確実に〜になるようにするだけの量(程度)に留まる
0329名無しさん@英語勉強中 (ワッチョイ 2e89-pBez)
垢版 |
2021/08/16(月) 18:35:40.21ID:FtErjoS80
>>324
Disruptions of sleep/wake patterns in the stable tubule only popypeptide (STOP)
null mouse model of schizophrenia

STOP については、>>326 さんが突き止めてくれた。とすると、
Disruptions ... (STOP) までが主語だな。そのあとの null が動詞で、
mouse model of schizophrenia が目的語だということになるけど、
mouse model の前に the とか a などをつける必要があると感じてしまう。
0331名無しさん@英語勉強中 (ワッチョイ 2e89-pBez)
垢版 |
2021/08/16(月) 18:37:21.05ID:FtErjoS80
mouse model の前の the または a が、論文の見出しなのだから省略されているとしたら、
その前の in the stable tubule... の the も省略されないと変だということになる。
まあ、こんなことはどうでもいいのだろうけど。
0333名無しさん@英語勉強中 (ワッチョイ 715a-1+Eo)
垢版 |
2021/08/16(月) 18:45:53.71ID:j2OnOQfQ0
Disruptions of sleep/wake patterns in

[ < the Stable Tubule Only Ppopypeptide (STOP) null> mouse model of schizophrenia ]

っていう題名の論文じゃないの?
題名だから文じゃない。
「STOP-null mouse model」で名詞句。
0334名無しさん@英語勉強中 (ワッチョイ 715a-1+Eo)
垢版 |
2021/08/16(月) 18:53:06.29ID:j2OnOQfQ0
欠損させた遺伝子機能名か何かが前にハイフンで付いているっぽい。

null mouse
ヌルマウス、ノックアウトマウス、欠損マウス
自然界で起こる相同組換え現象を利用して,正常な遺伝子を人工的につくった欠陥遺伝子と置き換えることにより,特定の遺伝子機能を欠失させたマウス.仮想の遺伝子Aを例にとると,遺伝子は父方と母方の二つがあるので,正常マウスでは A+/+,片方に欠損があるヘテロ接合体では A+/−,両方とも欠損している場合は A−/− と表示される.
0338名無しさん@英語勉強中 (ワッチョイ 2e89-pBez)
垢版 |
2021/08/16(月) 19:38:55.00ID:FtErjoS80
>>337
情報をありがとう。
0339名無しさん@英語勉強中 (ワッチョイ e996-lbFy)
垢版 |
2021/08/16(月) 21:10:21.58ID:CI5dn4p/0
Far from bringing a return to stability as he promised, it has led to the sort of imagery he insisted would not take place on his watch: scenes of American diplomats fleeing the U.S. embassy by helicopter as the Taliban tightened their noose around Kabul.
という文章を見かけたのですが、これはimageryとheの間にwhichなどの関係代名詞はいらないのでしょうか?
0342名無しさん@英語勉強中 (ワッチョイ c93b-pBez)
垢版 |
2021/08/16(月) 21:38:54.15ID:1kwIdir00
主格の関係代名詞は普通省略できませんが、こういう形のときは省略してもいいみたいです
なんとなく形が目的語が飛び出すタイプに似ているからでしょうか(先行詞の後にすぐSVがくる)
0345名無しさん@英語勉強中 (オッペケ Srf1-mh3b)
垢版 |
2021/08/16(月) 21:49:23.07ID:S0E1xT/1r
だからこそTom is the person whom I thought would be 〜
みたいに主語なのに目的格を使ってしまう例が存在する
ただ今は学校でも主格なら省略不可というより
先行詞SVになるなら省略可能というように習うことも多い
0346名無しさん@英語勉強中 (ワッチョイ c2e5-YgaZ)
垢版 |
2021/08/16(月) 23:33:59.28ID:/TLEtxZ60
ふと気になったのですが、主節が現在形でwhenが導く従属節が過去形の文章ってどういう意味になるのでしょうか?そもそも文法的にイリーガル?

よく知られている副詞節を導くwhenの時制のルールは下2つだと思いますが、
I will kick him when I meet him. ⇒ 私は彼に会った時に彼を蹴るだろう。
I kicked him when I met him. ⇒ 私は彼に会った時に彼を蹴った。

主節が現在形の場合、whenが導く従属節の時制はどうなるのでしょうか?また主節が現在系の時の意味というかニュアンスはどうなるのでしょうか?
I kick him when I met him. ⇒ 意味は?そもそも文法的にイリーガル?
0347名無しさん@英語勉強中 (ワッチョイW 06c5-wONL)
垢版 |
2021/08/17(火) 02:30:52.26ID:NifPRC690
Now playing the game, I live the time when the sucrificed king started his kingdom.
0348名無しさん@英語勉強中 (ワッチョイW 06c5-wONL)
垢版 |
2021/08/17(火) 02:37:51.44ID:NifPRC690
時制の一致というのはなくて、話してる時点から見て
過去だから過去形で書いてると言われたことがある。
whenは同時性を持ってるからwhen以下と、それより前は同じ時制になると思う。
0349名無しさん@英語勉強中 (ワッチョイW 06c5-wONL)
垢版 |
2021/08/17(火) 02:39:26.89ID:NifPRC690
文法的にイリーガルというより状況が考えられない、のでは?
0350名無しさん@英語勉強中 (ワッチョイW 06c5-wONL)
垢版 |
2021/08/17(火) 02:46:32.02ID:NifPRC690
>>339
アップツーデイトですね
0353名無しさん@英語勉強中 (ワッチョイW 999d-mZ4S)
垢版 |
2021/08/17(火) 13:06:38.75ID:8KzGB7EW0
すみません、TOEICスレでNGだったのでこちらで相談させてください。

Netflixの英語音声でアニメ観ようかと思ってるけどこのリストの中でおすすめの作品ありますか?
ttps://ei-raku.com/2017/10/netflix-anime-list/

バイオレットエバーガーデンが良かっただけに、とある科学は全然頭に入らなかった。。
0354名無しさん@英語勉強中 (ワッチョイ 0623-NTPF)
垢版 |
2021/08/17(火) 13:59:19.89ID:tynEW17h0
>>353
Netflixってこんなしょぼいんか
英語学習としてあんまり向いてない作品なのが多くて、そもそもアニメとしてもあんま面白くない作品しかないなw
アニメの英語音声で英語学習って俺も以前やってて50タイトルくらいは見たけど、そこのリストにはほとんど載ってないわ
TOEICスレにいるってことだし、会社がメイン舞台のトネガワとかNEW GAMEなんか良いんじゃないかね
あとアフィブログサイトのリンク貼らないほうがいいぞ
0359名無しさん@英語勉強中 (ワッチョイW 999d-mZ4S)
垢版 |
2021/08/17(火) 16:49:23.86ID:8KzGB7EW0
>>354
>>356
アドバイスありがとうございます!ひぐらしが初見なので良いかなと思っていますが結構過激ですね笑
やっぱりドラマが良いですかね、スーツが良さそうに見えましたが、フレンズが妥当ですかね…?
0360名無しさん@英語勉強中 (スップ Sdc2-N2Vw)
垢版 |
2021/08/17(火) 19:52:33.19ID:SmvWNc8Yd
Johns Hopkins Universityの発音って
ジョンズ・ホプキン「ス」
ではなく
ジョンズ・ホプキン「ズ」
で合ってますよね?

日本の新聞記事なんかだとほとんどすべてが「ス」と書いてあるんですが…
0362名無しさん@英語勉強中 (ワイエディ MM56-iNti)
垢版 |
2021/08/17(火) 21:18:07.45ID:defZv/pGM
すみません。どなたか分かる方、よろしくお願いします。
速読英熟語の中に
Roughly speaking, if you took a 15-minute shower, you probably used about 105 gallons that other people in dry areas of the world need to drink.
という文があるのですが、仮定法の主節は助詞の過去形とならったのですが、このようにただの過去形でもかまわないのでしょうか?

よろしくお願いいたします。
0367名無しさん@英語勉強中 (ワッチョイ c93b-pBez)
垢版 |
2021/08/17(火) 22:51:52.25ID:jK8OmAjq0
>>362
その本はたぶん大学受験用の熟語集ですよね
そういう参考書の中に学校で習う文法から逸脱した文章が掲載されることはまずないと思いました。もしあったとしても何かしらの註釈が付されるはずです。
その文があるパラグラフの最初のところに some of the things you did todayとありますよね、ここがミソです。
ご質問の文は仮定法ではなくただのifの条件文です。
訳は「もしもあなたが(今日)15分シャワーを浴びたとすれば、まず間違いなく約105ガロンの水を使用したことになります。(that other people以下は省略)」となります。

https://auctions.c.yimg.jp/images.auctions.yahoo.co.jp/image/dr000/auc0101/users/790bb6636d74fe9696de186104cbe988b1c75ff7/i-img675x1200-1579118029pm3bcu630395.jpg
0368名無しさん@英語勉強中 (テテンテンテン MM26-Uzpm)
垢版 |
2021/08/17(火) 22:52:46.90ID:Wpz43iOvM
>>366
そもそも仮定法じゃない中学レベルの話
もし君が使ってたのならこんだけスゲー量を使ったんだぞって言ってるだけで直説法で動詞が過去形なだけ
0370名無しさん@英語勉強中 (ワッチョイ c2e5-YgaZ)
垢版 |
2021/08/17(火) 23:38:35.48ID:mOaKfnad0
>>346です。調べました。自分の質問に回答すると、条件文の「習慣的な事実」を述べる用法で主節が現在形か過去形になる。この時の条件節は主節の時制に一致する。海外のESL向けのサイトで説明がありました。
eg. When he plays his music too loud, the neighbors complain.
eg. Unless there is a question, the class is dismissed.

>>360
日本語にするときは、各社内で対訳のリストやルールが決められていて、それに従っているだけです。発音記号なら/z/だが…。
https://www.google.com/search?q=hopkins+pronounce&;pws=0&gl=us&ei=mMkbYbatI5OgoATal4aICg&oq=Hopkins+pronoun&gs_lcp=Cgdnd3Mtd2l6EAMYADIECAAQEzIGCAAQChATMggIABAFEB4QEzIICAAQBRAeEBMyCggAEAUQChAeEBM6CQgAELADEAcQHjoMCAAQsAMQBxAeEIsDOgsIABCABBCwAxCLAzoXCC4QgAQQyAMQsAMQiwMQqAMQmAMQmgM6FAguEIAEEMgDELADEIsDEKgDEJ0DOgUIABCABDoRCC4QgAQQiwMQoQMQqAMQnQM6DQguEBMQiwMQqAMQnQM6EwguEMcBEK8BEBMQiwMQpgMQqAM6BggAEAQQHjoICAAQBRAEEB46CggAEAUQBBAKEB46BAgAEEM6EQguEIAEEIsDEJgDEJoDEKgDOgYIABAeEBNKBQg4EgExSgQIQRgBUPAtWPFDYIBPaARwAHgAgAGFAYgBsguSAQM0LjmYAQCgAQHIAQu4AQLAAQE&sclient=gws-wiz
0371名無しさん@英語勉強中 (ワイエディ MM56-iNti)
垢版 |
2021/08/18(水) 08:42:41.11ID:iN00DKsVM
>>362さん
ありがとうございました。
直接法と仮定法の見分け方も分かりました。
0372名無しさん@英語勉強中 (ワイエディ MM56-iNti)
垢版 |
2021/08/18(水) 08:43:30.32ID:iN00DKsVM
間違えました
>>367さん
ありがとうございました。
直接法と仮定法の見分け方も分かりました。
0373名無しさん@英語勉強中 (ワッチョイ 0623-NTPF)
垢版 |
2021/08/18(水) 08:53:21.51ID:7x2b+hz+0
>>359
自分で良さそうと思うのを見るのが1番良い
俺だけかもしらんが、ドラマは基本的にアニメより聴き取り難しいと思う
なのでアニメで気になるのをまず消化するのがおすすめ
0374名無しさん@英語勉強中 (アウアウキー Sa69-oUe1)
垢版 |
2021/08/18(水) 10:38:02.03ID:Dq2Bj8oTa
個人的にチャットしていて思ったこと
日本人でネイティブと互角にやりとりできる、ジョークも流暢という人は100%海外生まれ海外育ちだった
日本語はぶっちゃけ下手、ノンネイティブレベル

日本育ちの日本人の場合、一応言いたいことを言えるだけの英語力はあるが、英文のレベルはかなり劣る
ネイティブが3〜4行の長文をリアルタイムでポンポン投げてくるのに対して、簡単な短文をいくつか返すくらいが限度(時間をかければ長文もかける)
ネイティブ同士のジョークについていけない

あくまで個人的な観測範囲でしかないけども、日本育ちの場合はこの辺りの層がかなり多いと感じる
この層を抜けて上にいける人って相当少ないんじゃないかと思った
あとやはり海外勢は強い
ネイティブだから当然だけど
0376名無しさん@英語勉強中 (ワッチョイW d28a-ul3T)
垢版 |
2021/08/18(水) 14:13:12.15ID:freFZf4v0
>>374
一段目は、ネイティブとか帰国子女とか交えて複数人でチャットやってたってこと?
二段目はまさによくわかる
ひょんなことでTwitterのDMでチャット的なやり取りすることになったけど、マジで二段目の感じ
とにかく書くのが速い
0377名無しさん@英語勉強中 (アウアウキー Sa69-oUe1)
垢版 |
2021/08/18(水) 15:58:29.44ID:Dq2Bj8oTa
>>376
うんそんな感じ
基本ディスコード使ってる

I’m Japaneseって言ってるから日本人なんだろうけど英語がネイティブ並にうまい
いやこんな上手い日本人いるのかよと思って発言ログを漁ると、若干怪しい日本語とI was born in [an english speaking country].の文面を発見して「ああ海外の方ね」ってなるのが定番パターン

ツイッターやyoutubeを見るに純ジャパでもすごい人がいるのは確かだけども、そういうレベルの人とは一度も遭遇したことないなあ
ああいう人らは本当に一握りの天上人なんだなと思った
0378名無しさん@英語勉強中 (ワッチョイ 79e2-iWai)
垢版 |
2021/08/18(水) 16:00:13.19ID:WIZoEzFU0
ディスコードって何するの?
フリーに参加する音声チャットとかあるん?
0380名無しさん@英語勉強中 (ワッチョイ 2e84-I8f3)
垢版 |
2021/08/18(水) 16:16:44.19ID:aZU1wCoE0
質問です。
出る順パス単1級の例文です。
After making a speech, the mayor proclaimed the sports meet open.
これは「スピーチをした後で、市長はスポーツ大会の開催を宣言した。」と訳されています。
と言うことは”open”を名詞と考えていると思います。
しかし、”open”を動詞として考えることはできないでしょうか?
この文章を仮定法現在として捉えれば、that節(ここでは省略と考え)の中の動詞が原形となったとは言えないでしょうか?
結局、訳は変わらないと思いますが、いかがでしょう?
0382名無しさん@英語勉強中 (ワッチョイ 2e84-I8f3)
垢版 |
2021/08/18(水) 16:31:22.72ID:aZU1wCoE0
>>381
the sports meetでスポーツ大会だと思います。
0384名無しさん@英語勉強中 (ワッチョイ 2e84-I8f3)
垢版 |
2021/08/18(水) 16:51:34.70ID:aZU1wCoE0
>>383
あ!
openは形容詞ですか!
私の思い込みでしたね。
あっけなく納得しました。
ありがとうございました。
0385名無しさん@英語勉強中 (ワッチョイ c93b-pBez)
垢版 |
2021/08/18(水) 16:51:53.23ID:dbuyju8K0
その例文はちょっと微妙だと思います
proclaimを辞書(ジーニアス)で引くと、[SVO (to be) C]《◆Cは名詞》; [SV that 節]
という使い方をすると載っていますが、
declareは、[SVO (to be) C]のみで、Cに制限がなく、[SV (that) 節]となっているので、thatにカッコがついていては省略可能であることが示されています。
裏を返すとproclaimはthatを省略できないということです。openが名詞として使われていると解釈するしかないですが、それも微妙な気がします。

the mayor declared the sports meet open.のほうが適切だと思います
0387名無しさん@英語勉強中 (ワッチョイ 2e84-I8f3)
垢版 |
2021/08/18(水) 17:33:44.29ID:aZU1wCoE0
>>385
レスありがとうございます。
手元にある一番大きな辞書で小学館ランダムハウス(1994年1月1日第2版第一刷発行)によりますと、
proclaim
1.<特に国家的重要事項を>宣言[公告、公布、布告]する。
2.(公然とまたはこれ見よがしに)述べる、公言[揚言]する。
3.<もの・事が>明らかに物語る。
(以下7項まであります。)
declare
1.<自分の態度などを>表明[名言、言明、説明]する。
2.…を宣言[公表、布告]する。(proclaim)
3.…が(…であると)強調する、断言する;宣する。
(declareは以上3項まで)
ここでdeclareの2項の例文で
I hereby declare the opening of the Olympic Games.
とほぼ前出の例文に近いものが掲載されています。
ということで、(どちらかと言うと)proclaimよりもdeclareの方がスポーツ大会に親和すると思われます。
ただ、この前出の例文は”proclaim”が見出し語の例文なんですね。ですので私としてはどちらがとは言えないです。
また、ランダムハウスには
You proclaimed you're a righteous man, didn't you?
と載っていることから、thatは省略不可はないものと思われます。
ご意見ある方、レスお願いします。
0390名無しさん@英語勉強中 (ワッチョイ 2e89-pBez)
垢版 |
2021/08/18(水) 18:25:39.81ID:u/lUDx0Z0
The Loyal Subjects booth dubbed itself The Loyal Subjects Garage,
with a neon sign ★proclaiming it open★ 24 hours a day.
https://www.comicsbeat.com/bandai-the-loyal-subjects-toy-fair/

ただし、この上の英文が絶対に正しいという確証はないけど。ともかくこれは
ニュースサイト上の用例。
0391名無しさん@英語勉強中 (ワッチョイ 2e89-pBez)
垢版 |
2021/08/18(水) 18:34:24.36ID:u/lUDx0Z0
After a few companies like Zillow, Pandora and especially LinkedIn had successful IPOs,
★the window was proclaimed open★--and then shut again as the markets convulsed.
https://www.businessinsider.com/the-way-companies-are-getting-financed-is-completely-changing-2011-11

And in Manhattan ★“Circle of Animals/Zodiac Heads,” which is being termed Mr.
Ai’s first public sculpture, was proclaimed open★ on Wednesday morning in a drizzle
at the Pulitzer Fountain in front of the Plaza Hotel.
https://www.nytimes.com/2011/05/05/arts/design/ai-weiwei-sculpture-near-plaza-hotel-review.html
0392名無しさん@英語勉強中 (ワッチョイ 2e89-pBez)
垢版 |
2021/08/18(水) 18:36:33.47ID:u/lUDx0Z0
https://www.couriermail.com.au
2018/05/12 — ★Brisbane was proclaimed open★ to free settlement in 1842
and by 1848 the first immigrants began to arrive and Brisbane began to blossom, ...
0394名無しさん@英語勉強中 (ワッチョイ 2e84-I8f3)
垢版 |
2021/08/19(木) 15:56:53.78ID:FOjHtMEX0
昨日の>>384 のopenの話ですが、品詞は何になるのでしょう?
少し気になります。
>>385では名詞としてらっしゃる。
>>390〜392では、やっぱり形容詞ですかね?
0395名無しさん@英語勉強中 (ワッチョイ 2e89-iNti)
垢版 |
2021/08/19(木) 16:05:17.63ID:3cG2jbgz0
>>394
形容詞に決まってるじゃんか。>>395 も、決して名詞だなんて言っていない。
よく読んでみたら?せっかくの回答も、読む人がデタラメな読み方をしてしまうと台無し。
0396名無しさん@英語勉強中 (ワッチョイ 2e89-iNti)
垢版 |
2021/08/19(木) 16:06:12.61ID:3cG2jbgz0
アンカーをつけ間違った。

>>394
形容詞に決まってるじゃんか。>>385 も、決して名詞だなんて言っていない。
よく読んでみたら?せっかくの回答も、読む人がデタラメな読み方をしてしまうと台無し。
0397名無しさん@英語勉強中 (ワッチョイ 2e89-iNti)
垢版 |
2021/08/19(木) 16:10:13.48ID:3cG2jbgz0
おっと、俺の方が読み間違いをしてた。
>>385 は確かに open を名詞として解釈する以外にないと言っているな。
いずれにしても、open が名詞のはずがないだろ?
proclaim という動詞の使われ方が declare のそれと似ているけど、
その使われ方を考えたらすぐにわかるだろうに。
0398名無しさん@英語勉強中 (ワッチョイ c93b-pBez)
垢版 |
2021/08/19(木) 16:22:42.86ID:9OMc4ZZE0
ジーニアスの説明どおりに解釈するとopenは名詞として解釈するしかないですが、
意味としては形容詞として取りたいところです。そういう意味で「微妙」と書きました。

いろいろ調べてみたんですが
Steve checked the battery and proclaimed it to be dead.
https://www.oxfordlearnersdictionaries.com/definition/english/proclaim
proclaim the suspect to be guilty
https://www.thefreedictionary.com/proclaim
のようにCの部分に形容詞が来る例も辞書レベルでみつかりました。ひょっとしたらジーニアスが間違ってるのかもしれません
0399名無しさん@英語勉強中 (ワッチョイ 2e84-I8f3)
垢版 |
2021/08/19(木) 16:52:29.78ID:FOjHtMEX0
>>397
>>398
形容詞ですよね。
ジーニアスの《◆Cは名詞》については、またいつかということで。
ありがとうございました。
0400名無しさん@英語勉強中 (ワッチョイW 06c5-wONL)
垢版 |
2021/08/19(木) 17:07:15.33ID:vd2VgN310
proclaimはthat節取るから、should openでshouldが抜けたものとか
to beなんちゃらとるからproclaim sports meet to be open
で形容詞と考えられるけど、to be openとするよりto be openedとするから、
openが動詞の方が最もらしいと思う
0401名無しさん@英語勉強中 (ワッチョイW 06c5-wONL)
垢版 |
2021/08/19(木) 17:09:44.73ID:vd2VgN310
open に始まるという自動詞があるからね。オーレックス
0402名無しさん@英語勉強中 (ワッチョイW 06c5-wONL)
垢版 |
2021/08/19(木) 17:11:02.68ID:vd2VgN310
openが形容詞なら開いているということだけど、
開いていると宣言したとか間抜けな感じがする
0403名無しさん@英語勉強中 (ワッチョイ 2e89-iNti)
垢版 |
2021/08/19(木) 17:15:19.10ID:3cG2jbgz0
辞書を過信してたらダメだと思う。なんでもかんでも辞書に書いてある通りに
なってるはずがないよ。もちろん、ジーニアス(第5版)の当該項目の記事を
書いている人たちは、10年あるいは15年くらい前の自分の調査結果に基づき、
精いっぱいに書いている。彼らがその時期に精いっぱいに調べた範囲内では、
そのようなことが言えた、というだけのことだ。

そのあと 10 年から15年ほど経ったけど、ほんのその短いあいだに検索機能、
ありとあらゆる辞書、コーパスを使う素人たちの知見などがすべて
急速に進展した。だから、その時代の辞書編纂者たちの知見を、15年後のど素人が
追い抜かすことだってたまにはあるだろうよ。

いずれにしても、>>390-392 あたりの用例は続々と見つかるのだから、
すでに古くなってしまった当該ジーニアスの記事のその部分だけに関しては
眉唾ものだということになる。

だからと言って、ジーニアスが間違ってるなんてことを言ったら罰が当たる。
俺たちがそのジーニアスを10年前、あるいは15年前に編纂したら、はるかにはるかに
ひどい間違いを書きまくっていただろう。そういう点で、ジーニアスを初めとする
辞書編纂者たちは偉いと思う。

と言うわけで、辞書に書いてあるからこれこれこうなるはずだとしか言わない
俺たちど素人が悪いのであって、辞書編纂者が悪いわけではない。

なお、俺は辞書編纂者ではない。俺が辞書を作ってたら、喧嘩ばかりしてたろうし、
もっとひどい間違いを書いてたろうから、辞書作りどころじゃないと思う。
0404名無しさん@英語勉強中 (ワッチョイ 2e84-I8f3)
垢版 |
2021/08/19(木) 17:20:44.15ID:FOjHtMEX0
と、言うことは…
>>380 が良い線行ってる?
0405名無しさん@英語勉強中 (ワッチョイW 06c5-wONL)
垢版 |
2021/08/19(木) 17:25:23.14ID:vd2VgN310
openが名詞ならsports meet は形容詞だけど、こういう場合はsport-meetとハイフンを入れてたはずの気がする
0407名無しさん@英語勉強中 (ワッチョイ 06c5-aLkB)
垢版 |
2021/08/19(木) 18:03:09.15ID:vd2VgN310
>>406
じゃあ形容詞しかないのかな
0409名無しさん@英語勉強中 (ワッチョイ 9f2d-FcIJ)
垢版 |
2021/08/20(金) 00:15:47.53ID:Pc/1e8ax0
I'm fully vaccinated against COVID-19 っていうTシャツ作ろうかと思ってるんですが I'm 省略して
Fully vaccinated against COVID-19 てしても意味伝わりますかね?おかしな意味になっちゃう?
0411名無しさん@英語勉強中 (ワッチョイ ff89-zH/y)
垢版 |
2021/08/20(金) 04:08:27.60ID:eJLc9rMu0
>>409
それで正しいよ。それに似た T シャツは、すでに巷にたくさんあるらしいね。

https://www.etsy.com/market/vaccine_shirt
0414名無しさん@英語勉強中 (ワッチョイ ff84-oSMs)
垢版 |
2021/08/20(金) 20:02:33.50ID:Us+LWIyT0
>>413
BOO!
0416名無しさん@英語勉強中 (ワッチョイ ff33-Awm0)
垢版 |
2021/08/21(土) 02:40:13.00ID:7sFpjVeE0
>>412
このjustifiablyは文修飾の副詞ですね
文修飾の副詞は、文頭に置くことが多いですが、文中(中位)や文尾に置くこともできます
文中の場合は、be動詞の後/一般動詞の前/助動詞と本動詞の間、に置くのが普通です
0418名無しさん@英語勉強中 (ワッチョイ 9f9d-W4K4)
垢版 |
2021/08/21(土) 06:32:56.43ID:b/2mIfZD0
「日常会話」を英語で表記したいのですが
「猥談」と区別する意味を強調する場合はどのような表現がいいのでしょうか?
「猥談ではない日常会話」という意図を簡潔に伝えたいです
0424名無しさん@英語勉強中 (ワッチョイ ff89-zH/y)
垢版 |
2021/08/21(土) 17:58:12.50ID:Uf56kD1z0
>>418

I'm open to discussing just about any topic, except casual talk about sex.

参考:
(1)
https://www.google.com/search?q=%22open+to+discussing+just+about+any+topic%22&;newwindow=1&rlz=1C1FQRR_enJP949JP949&sxsrf=ALeKk029fBjCjpEgtOBVtzAvC0_HDAwHvw%3A1629536041120&ei=Kb8gYYDeBtGzmAXQiZWQAQ&oq=%22open+to+discussing+just+about+any+topic%22&gs_lcp=Cgdnd3Mtd2l6EAM6BwgAEEcQsAM6BQghEKABSgQIQRgAUNDLAVj83gFgh-MBaAFwAngAgAGNAYgB6xCSAQUxMS4xMJgBAKABAcgBCMABAQ&sclient=gws-wiz&ved=0ahUKEwjAloTn3sHyAhXRGaYKHdBEBRIQ4dUDCA4&uact=5

(2)
Most of us have friends who are so close that we tell them "everything"…except about sex.
0426名無しさん@英語勉強中 (ワッチョイ ff2c-JiHC)
垢版 |
2021/08/21(土) 19:24:51.88ID:YNDJcmEr0
>>423
Justifiably there is a good deal of emotion to go round.
There justifiably is a good deal of emotion to go round.
There is justifiably a good deal of emotion to go round.

この3つの違いを気にしない人にとっては、
文修飾で皆同じだと思っている方が幸せだね
0427名無しさん@英語勉強中 (ワッチョイ 1fe1-jkMS)
垢版 |
2021/08/21(土) 19:40:28.24ID:0L1jNy/00
downの発音

今はもう辞書のとおりに発音してないネイティブが多いと言う気がしますが
西海岸だけ?

Appleの最初の母音と同じ発音での発音が多い。アナウンサーなんかも
https://www.youtube.com/watch?v=ug9z0YndrLE

この動画でもふたりとも日本の辞書通りは発音してないですよね
0428名無しさん@英語勉強中 (ワッチョイ 7f5f-Awm0)
垢版 |
2021/08/21(土) 20:23:01.76ID:Zp7FcsOU0
Since both sides would not agree, it required someone with tact to make peace.

このitは特定のなにかというよりit rainsなどのitと同じように状況を表す?ためのものに感じるのですが、こういった用法は存在するのでしょうか?
0429名無しさん@英語勉強中 (ワッチョイ 7f5f-Awm0)
垢版 |
2021/08/21(土) 20:28:03.14ID:Zp7FcsOU0
>>428
すみません
よくよく見てみると後ろのto以下を単純に受けているような気がしてきました
この解釈で正しいでしょうか?
0430名無しさん@英語勉強中 (ワッチョイ ff89-zH/y)
垢版 |
2021/08/22(日) 03:22:11.13ID:L4LJu7pf0
>>428
Since both sides would not agree, it required someone with tact to make peace.

またもや文脈もなしに質問しようとしてるだろ?あなたがその英文を見たのはもしかして、

4000 Essential English Words 6 » Unit 22: The Editor’s Choice

tact [tækt] n.
Tact is the ability to avoid offending people when dealing with problems.
→ Since both sides would not agree, it required someone with tact to make peace.
https://www.essentialenglish.review/book/4000-essential-english-words-6/unit-22-the-editors-choice/

上のような文脈で見たのか?もしそうなら、文脈が何もないと言っていいわけだな?
それだったら、it はその場の状況を漠然と表していると考える以外にないわな。
to make peace を意味しているわけはないわな。

しかしもしもこの英文の前に何かがあって、例えば次のようなかんじだったらどうだろう?

They were in a legal quagmire. Since both sides would not agree, it required someone with tact to make peace.

もし上のような感じになっていたら、it はその前の quagmire を指しているということになるだろ?

ともかく、文脈も示さずに質問するなと言ってるだろうが。アホなのかお前らは。
0431名無しさん@英語勉強中 (ワッチョイ ff89-zH/y)
垢版 |
2021/08/22(日) 03:26:05.69ID:L4LJu7pf0
>>427
辞書通りに発音していないって?もし辞書が現実をそのまま表現しないといけないんだったら、
あらゆる辞書が1億ページずつの巨大なものにならないといけなくなる。
そんな辞書を作ったって、誰が使うかよ?辞書が現実をそのまま反映していないのは、
最初からみんなわかってるだろ?apple の最初の母音と同じように down を発音してる
人がけっこういるってことは、誰でも普通に英語を聞いてたらわかるだろ?
0433名無しさん@英語勉強中 (ワッチョイ 9f9d-AI8d)
垢版 |
2021/08/22(日) 12:59:22.20ID:iURix4q20
>>432
Midwest
ウィスコンシン州、オハイオ州、インディアナ州、イリノイ州、ノースダコタ州、サウスダコタ州、ネブラスカ州、アイオワ州、ミネソタ州、ミシガン州、カンザス州、ミズーリ州
0434名無しさん@英語勉強中 (ワッチョイ ff89-zH/y)
垢版 |
2021/08/22(日) 13:07:31.01ID:L4LJu7pf0
アメリカ合衆国をいくつかの地域に分けるやり方は一種類ではなく、いろいろあるらしいことが、
この下のリンク先にあるたくさんの地図を見ればわかる。

USA Regions
https://www.google.com/search?q=midwest+south+east+usa+america+map&;newwindow=1&rlz=1C1FQRR_enJP949JP949&sxsrf=ALeKk03v6R26H71QrpQdxdcSPE6SDMT_CA:1629604832340&source=lnms&tbm=isch&sa=X&ved=2ahUKEwiHg5-J38PyAhWTxosBHSV4CjsQ_AUoAXoECAEQAw&biw=1536&bih=722#imgrc=ns91Ynf1ANydhM
0435名無しさん@英語勉強中 (ワッチョイ 9f9d-AI8d)
垢版 |
2021/08/22(日) 13:11:19.84ID:iURix4q20
>>434
433はBritannica Concise Encyclopediaがソース
ちゃんとしたソースを出しましょう
0436名無しさん@英語勉強中 (ワッチョイ ff89-zH/y)
垢版 |
2021/08/22(日) 13:27:31.30ID:L4LJu7pf0
Britannica だけが「ちゃんとした」ソースだと言うのなら、
勝手にそう信じておればいいだろう。いちいち喧嘩を吹っかけてくるな。
0437三年英太郎 ◆3CZBjOt3.Y (ワッチョイ 7f86-duJi)
垢版 |
2021/08/22(日) 13:45:28.66ID:qfLcUSKz0
深夜から更年期イライラレスかぁ・・・
色々面白い問題を含んでるのに、「辞書が1億ページになるだろ!」(なんねーよ)
で終わっちゃうとこが、この人の知的限界だよね
0438三年英太郎 ◆3CZBjOt3.Y (ワッチョイ 7f86-duJi)
垢版 |
2021/08/22(日) 14:20:27.80ID:qfLcUSKz0
「ざっし」と「ひざ」の「ざ」は、音声的には違うのだが、音韻的には同じ。
それと同様に、

[aʊ][æʊ][ɑʊ]

これらの間には音韻的な差異はない(たぶん)。
ので、ふつーの英和/英英では書き分けない。

/aʊ/が、実際にはどのように発音されるのか、
そーゆーのが辞書の付録としてついてればいいと思うけども。

ワイは確認してないが、↓には出てるかもな。
https://books.kenkyusha.co.jp/book/978-4-7674-0412-7.html
0440名無しさん@英語勉強中 (ワッチョイ ffc5-JiHC)
垢版 |
2021/08/22(日) 14:38:17.22ID:vQdRjra70
>>437
別にいいんじゃないの?アメリカは人口3億3千万人いて、日本は1億3千万人いるから、
2倍以上の規模なんだね。フランスの人口は6千7百万、ドイツの人口は8千3百万、
イギリスの人口は6千7百万しかないから、日本人としては、英独仏に尊敬を
払いながらも、それら小国の主張を甘んじて受け入れることなく、これからも
石炭を使った火力発電を必要に応じて利用していくべきなんだ。

それはともかく、なぜ中西部の話が出てきたかというとGeneral American English
(標準米語)というのは、The Great Gatsbyで、アメリカの本流として描かれている
中西部の言葉だから出てきたんだ。農作地帯かというと、そうでもないよね。
シカゴとかデトロイトを含んでいるから。多分ラジオ、テレビが普及していく時代の
中心がそれらの工業地帯だったから、広まる言葉は、マンハッタンの言葉ではなくて
中西部の言葉だったんじゃないかな?

それはともかく、人ごとに言葉があって、それらを一つ一つ拾っていくのが
辞書を作る人の仕事だと想像するだけでワクワクする。
0446名無しさん@英語勉強中 (ワッチョイ ffc5-JiHC)
垢版 |
2021/08/22(日) 14:51:26.66ID:vQdRjra70
辞書を社員に作らせるというのはお金がかかりそうな気がする。
大学の先生にやらせておけば、著作権料だけで残業代も請求せずにやりそうな
気がする。出版社が作っている辞書ってどんな辞書がある?
0448名無しさん@英語勉強中 (ワッチョイ ff23-FcIJ)
垢版 |
2021/08/22(日) 15:02:45.98ID:Ewjy9OM70
もう16年も経ってしまったか…
それにしても今はもう調べ物で辞書など使わずググるだけという人が多くなってきたらしいな
英語チューバーが辞書はムダみたいに切り捨てて叩かれてたが
0449名無しさん@英語勉強中 (ワッチョイ ffc5-JiHC)
垢版 |
2021/08/22(日) 15:22:59.87ID:vQdRjra70
インターネットブラウザでググるとか無料辞書とかホームページを検索する
よりも、辞書アプリはお金払って買ってるから信用できるし、
馴染んでいるから、使いやすいんだけどね
0451名無しさん@英語勉強中 (ワッチョイ 9fe2-DepB)
垢版 |
2021/08/22(日) 15:27:57.74ID:GTdmcwoc0
LongmanもWebsterも無料だし信用できるけど?
0452名無しさん@英語勉強中 (ワッチョイ 1fe1-ip5u)
垢版 |
2021/08/22(日) 18:29:30.19ID:jm2u/1Pr0
pizza miss miracleって
どういう意味でしょう
0457アメリカ人 (ワッチョイW ff05-PKJV)
垢版 |
2021/08/22(日) 20:54:59.55ID:SvGUvdz80
>>456
専門用語を知らないし、英語が自然に身についているので、あまり説明できません。
でも、わからない単語を聞いてほしいとか、文章を添削してほしいとか、そういうことなら喜んで手伝いしますよ
0458名無しさん@英語勉強中 (ワッチョイ ffe1-ip5u)
垢版 |
2021/08/22(日) 21:27:24.93ID:JItWuIbQ0
ありがとう!
なるほど!クリスマスにひっかけて ピザマス!
ピザを食べると決めてる日!っていう意味かも!
そういう楽しいラッキーデー!に何かいいこと(miracle)がある!っていう意味かも!w
0459名無しさん@英語勉強中 (ワッチョイ 1f6c-W4K4)
垢版 |
2021/08/23(月) 09:59:42.01ID:h5BqiGKq0
ポレポレって受験参考書の本にある英文

And since the kings were more likely than farmers to influence world events
and to have biographies written about them, what we know of history has
been largely shaped by the careers of famous men.

have biographies written about them 
のとこ haveを使役動詞で解釈されてんだけど
written about themを biographiesにかけて解釈したほうが自然にみえるんだが

最初の解釈だと
さまざまな伝記について自分らのことを書かせるって意味にならんのか

https://www.youtube.com/watch?v=fnRG5f02Ii8&;list=PLTVJQzNVadQf-Ghy4JAz5PZgz-_QhaeIu&index=13
0460アメリカ人 (ワッチョイW ff05-PKJV)
垢版 |
2021/08/23(月) 11:10:08.08ID:Ju71Ux5n0
>>459
日本語の文法用語を知らないので(使役動詞とか)、この説明では不十分かもしれません。

"biography "は通常、その人物以外の人が書いたものという意味で使われます。
自分自身についての伝記の場合は、"autobiography"が使われます。

"have biographies"だけでは、伝記を所有していることと混同される可能性があるので、ここではよくありません。

王様が自分についての伝記を書くように命令しているのか、それともただ受動的に自分についての伝記を書いてもらっているのかが不明ですが、それを聞いているのだとしたら。
0461名無しさん@英語勉強中 (ワッチョイ 1f92-JiHC)
垢版 |
2021/08/23(月) 11:23:39.77ID:Cgb0+EVp0
I will, nevertheless, make an effort, and try anew the same path on which I had entered yesterday, that is, proceed by casting aside all that admits of the slightest doubt, not less than if I had discovered it to be absolutely false.

このnot less thanなのですが、よくわかりません。
次のように考えました。
僕(デスカルテスくん)はそれでも努力するつもり。
そして昨日入った同じ道を新たに試してみるつもり。
んとね、ちょっとでも疑わしいと思ったらなんでも傍に
避けて進もうと思う。
ちょっとでも疑わしいケース数は、絶対間違っていると
すでにわかっているケース数よりも、数的には多い(少なくない)んだよね。

どうか教えていただけたらと思います
0462名無しさん@英語勉強中 (ワッチョイ 1f92-JiHC)
垢版 |
2021/08/23(月) 11:31:11.78ID:Cgb0+EVp0
アメリカ人さんがいるので
>>461の参考書訳も転写します。高橋さん曰く
私は努力して、自分が最近歩み出したのと同じ道を改めて
歩もうと、つまり、どんなにわずかでも疑いの余地のあるものは。
それが全く誤りであるということに気づいた場合に勝るとも劣らず、
全て捨て去って進むつもりです。
0463名無しさん@英語勉強中 (ワッチョイ ff89-zH/y)
垢版 |
2021/08/23(月) 11:34:12.00ID:Vt97Mf/U0
>>461
>>(I will...) proceed by casting aside all that admits of the slightest doubt,
★not less than★ if I had discovered it to be absolutely false.

not less than って、at least っていう意味だと辞書には書いてある。もしここでもそういう
意味なんだとしたら、次のような意味だと解釈したらいいと思うんだが、違うかな?

「ほんの少しでも疑わしいものは脇に追いやって進む(つもりだ)。
●少なくとも●、絶対に嘘だとわかったものは少なくとも(わきに追いやる)。」
0465名無しさん@英語勉強中 (ワッチョイ 1f92-JiHC)
垢版 |
2021/08/23(月) 11:45:24.74ID:Cgb0+EVp0
ちなみに>>462の訳は英文読解講座p143に載っていました。
インターネットにわからないところが載っているものなのですが、
この文についてはたくさんの外国サイトしか見つかりませんでした。
多分、ここまでたどり着く人は少数なのでしょう
0466名無しさん@英語勉強中 (ワッチョイ 1f6c-W4K4)
垢版 |
2021/08/23(月) 12:17:33.84ID:h5BqiGKq0
>>460

have biographiesだけなら伝記の所有になるとおもうけど
biographies written about them
をhaveするってのは
自分の伝記を書かせるって意味にならないの?

have O Cってとれるの?この構文
なんか違和感ある
伝記一般に自分のこと書かせるっておかしくない?
0467名無しさん@英語勉強中 (ワッチョイ 1f6c-W4K4)
垢版 |
2021/08/23(月) 12:24:27.68ID:h5BqiGKq0
同じ疑問もってるひといるんだな
https://www.quen.jp/posts/41e5d010-73c0-11ea-b10f-0f929038e487

have O C なら
have their biographies writtenとか
have biographies of them writtenとかにならんの?
英語表現あってるかどうかまったくわからんけどww
0468名無しさん@英語勉強中 (ワッチョイ 1f6c-W4K4)
垢版 |
2021/08/23(月) 12:24:27.68ID:h5BqiGKq0
同じ疑問もってるひといるんだな
https://www.quen.jp/posts/41e5d010-73c0-11ea-b10f-0f929038e487

have O C なら
have their biographies writtenとか
have biographies of them writtenとかにならんの?
英語表現あってるかどうかまったくわからんけどww
0469名無しさん@英語勉強中 (ワッチョイ 9f3b-zH/y)
垢版 |
2021/08/23(月) 12:25:26.08ID:W8MP2noT0
>>463
私も辞書を引いてみましたが、not less thanを「少なくとも」の意味で使うときは後ろに数詞を伴うとあります
だから「勝るとも劣らず」のほうの意味で取ったほうがいい気がします…たぶんですが

たぶんデスカルテスっていうのはデカルトDescartesのことで方法序説か省察あたりの本文の英訳なのではないかと予想します
>少しでも疑いうるものはすべて偽りとみなしたうえで,まったく疑いえない絶対に確実なものが残らないかどうかを探る態度。
↑は適当に調べたらでてきた方法的懐疑の説明です。ちょっとズルですが、これになぞらえて考えてみると、

少しでも疑いを差し挟む余地のあるものに対して、明らかな誤りであるものと同様手加減はしないよってことを言いたいんだと思います。
0470名無しさん@英語勉強中 (ワッチョイ 1f6c-W4K4)
垢版 |
2021/08/23(月) 12:40:12.61ID:h5BqiGKq0
not less than if I had discovered it to be absolutely false.

ふつうに高橋の訳でいいとおもうんだが

絶対間違っている場合に(cast aside)するのと
少しも劣らず(not less)
すこしでも疑わしい場合はcast asideする
0471アメリカ人 (ワッチョイW ff05-PKJV)
垢版 |
2021/08/23(月) 12:41:56.91ID:Ju71Ux5n0
>>466
>biographies written about themをhaveするってのは自分の伝記を書かせるって意味にならないの?

そんなことはありえません。
その意味では、名詞に言及する必要があります。
"have people write (auto)biographies about themselves"
この文章から「people」を省略することはできません。peopleがすでに文章に存在していなければ、themselvesは存在できないです。だから、受け身ダメです。
他の質問に答えることはできませんが、そのような文章は受け身ではよくあることではないでしょうか?"I had my order taken." "I had my son punished" 'I had my hair cut"など
0473名無しさん@英語勉強中 (ワッチョイ 1f6c-W4K4)
垢版 |
2021/08/23(月) 12:49:27.20ID:h5BqiGKq0
>>471
そうなの?これ以外に解釈しにくいんだけど

have people write (auto)biographies about themselvesはわかるけど
have biographies written about themは
これとはちがうやん

"I had my order taken." "I had my son punished" 'I had my hair cut"
これはわかるけど
I have hair cut about meとかいうのか?www
不自然な英語に思うんだけど
0474名無しさん@英語勉強中 (ワッチョイ 1f6c-W4K4)
垢版 |
2021/08/23(月) 12:54:13.43ID:h5BqiGKq0
biographies (written about them)
biographies 名詞
written about them 形容詞句

have biographies written about them
自分に関する伝記をもつ(=自分の伝記が書かれる)

この解釈でなんでだめなんだ
0476名無しさん@英語勉強中 (ワッチョイ 1f92-gaxM)
垢版 |
2021/08/23(月) 13:19:41.15ID:Cgb0+EVp0
>>469
ありがとうございます。ググったときに
他の文章が出てくるのですが、その文章が
そういう内容でした。有名な話だったんですね?
デカルトの方法論序説を知らないなんてどれだけ教養がないか>僕

>>470
おかげでよくわかりました。高橋訳の意味も
おかげでわかりました。ありがとうございます
0477名無しさん@英語勉強中 (ワッチョイ 1f92-gaxM)
垢版 |
2021/08/23(月) 13:22:47.74ID:Cgb0+EVp0
>>472
ありがとうございます。
weblioとウィズダムには>>469さんが
指摘してくださったような、数詞を伴う
というようなことが書いていることに気づきました。
オーレックスは例文しか載ってない。
0480名無しさん@英語勉強中 (ワッチョイ 1fe1-ip5u)
垢版 |
2021/08/23(月) 19:25:36.55ID:3GKBF5tj0
I want you so bad I can taste it
なら
どうしてもあなたが欲しい!だと思うんですが

最初を

I loveにしたら変ですか?「どうしても〜〜したい」のほかに「非常に、激しく」の
意味もあると辞書にはあるんですが
0481名無しさん@英語勉強中 (ワッチョイ 1fe1-ip5u)
垢版 |
2021/08/23(月) 19:25:36.93ID:3GKBF5tj0
I want you so bad I can taste it
なら
どうしてもあなたが欲しい!だと思うんですが

最初を

I loveにしたら変ですか?「どうしても〜〜したい」のほかに「非常に、激しく」の
意味もあると辞書にはあるんですが
0482名無しさん@英語勉強中 (ワッチョイ 1f6c-W4K4)
垢版 |
2021/08/23(月) 20:33:54.92ID:h5BqiGKq0
>>479
根拠がないので納得できない

have biography written about them

have O C
OをCにする

biographyはかかれたもの 
書かれたものを 書かれた状態にするという意味に解釈するのは不自然に思える
0483名無しさん@英語勉強中 (ワッチョイ 1f6c-W4K4)
垢版 |
2021/08/23(月) 20:39:11.42ID:h5BqiGKq0
彼らについて書かれた状態にする と解釈する場合
伝記一般に彼らの内容が書かれる意味になって不自然

受け身のhaveととる場合
上記の同じ理由で不自然

使役のhaveととる場合
王の死後 王の命令なしに歴史家が伝記を書く場合があって不自然
0486名無しさん@英語勉強中 (ワッチョイ 1f6c-W4K4)
垢版 |
2021/08/23(月) 20:53:11.46ID:h5BqiGKq0
なんだ
英語ができないバカか
馬鹿が書きこむばかりで有益なレスないな
0487名無しさん@英語勉強中 (テテンテンテン MM4f-kdlH)
垢版 |
2021/08/23(月) 21:00:05.15ID:x4ApZ0cqM
>>459
こんな恥ずかしいレスしてるやつがそれを言うのか
0488名無しさん@英語勉強中 (ワッチョイ 1f6c-W4K4)
垢版 |
2021/08/23(月) 21:02:01.52ID:h5BqiGKq0
あほが自演してる
死ねばいいのに
0489名無しさん@英語勉強中 (ワッチョイ 1f6c-W4K4)
垢版 |
2021/08/23(月) 21:03:14.49ID:h5BqiGKq0
英語ができないあほは 煽るしかないから
すぐにあほってわかる
自分もすぐばれるあほ 誰の役にもたたないあほは死んだ方が社会のためになる
0490名無しさん@英語勉強中 (ワッチョイ 1f6c-W4K4)
垢版 |
2021/08/23(月) 21:05:08.39ID:h5BqiGKq0
英語ができないあほの自演

484名無しさん@英語勉強中 (アウアウクー MM73-HUiG)2021/08/23(月) 20:40:39.87ID:ZSDv12T/M
どういう人がこの質問してんだ?ポレポレとか言ってるから大学受験せいか?
なんかいろいろ大変そうだな
485名無しさん@英語勉強中 (アウアウアー Sa8f-HUiG)2021/08/23(月) 20:48:52.21ID:/rLGRD0Ha
この人はcook the dishesとかも「料理はもう作られてるんだから、cookは不自然」とかって言うんだろうかw
487名無しさん@英語勉強中 (テテンテンテン MM4f-kdlH)2021/08/23(月) 21:00:05.15ID:x4ApZ0cqM
こんな恥ずかしいレスしてるやつがそれを言うのか
0491名無しさん@英語勉強中 (ワッチョイ 1f6c-W4K4)
垢版 |
2021/08/23(月) 21:07:17.76ID:h5BqiGKq0
あほをさらす

479名無しさん@英語勉強中 (オッペケ Sr73-YOT6)2021/08/23(月) 17:02:40.43ID:NWZ/yPcDr
そのhaveは
I had my purse stolenと同じでしょう

名前:名無しさん@英語勉強中 (ワッチョイW 1f92-p/uk) 2021/08/23(月) 19:59:06.79 ID:Cgb0+EVp0
これはhave biographiesに伝記を書いてもらうの意味はなくて、伝記を所有と捉えられるからあきまへんやろと
アメリカ人さんから指摘していただいたんとちゃいまんの?
0492名無しさん@英語勉強中 (ワッチョイ 1f6c-W4K4)
垢版 |
2021/08/23(月) 21:10:59.35ID:h5BqiGKq0
こいつ予備校マニアだな
受験英語で挫折して ずっと受験英語だらだら勉強して
けっきょく だれかに解説してもらわないと いまも難関大学の入試解けずにいるんだろうな

16 名前:名無しさん@英語勉強中 (ワッチョイW ffc5-p/uk) 2021/08/20(金) 04:55:23.70 ID:I0ds8IGM0
そうでないものはポスト構文主義という表の流派
構文によらない奥野の流派,
富田派、薬袋のFor 派
とう
0493名無しさん@英語勉強中 (ワッチョイ 1f6c-W4K4)
垢版 |
2021/08/23(月) 21:13:30.49ID:h5BqiGKq0
受験参考書の予備校講師の解説を神のように信じて疑わないやつなんだろう
このレベルの英語力だといろいろ大変そうだな
0496名無しさん@英語勉強中 (アウアウウー Sa63-BMF6)
垢版 |
2021/08/23(月) 23:41:52.93ID:sSqMchgqa
The doctor had the patient taken for an X-ray.
医師は患者をレントゲン撮影に連れて行くようにと指示した

この様な例文もあることだし、何がなんでも後置修飾でなければならないとは言えないだろう
どちらかと言ったら、私は使役で取った方が歴史が歪められている感じが出ていいと思うが。
0497名無しさん@英語勉強中 (ワッチョイ ff33-Awm0)
垢版 |
2021/08/24(火) 05:58:35.49ID:jJfl8fYQ0
>>459
使役動詞の使い方として、特に不自然な表現とは思いません

『Nonfiction』(2008年)
George Washington. Abraham Lincoln. Martin Luther King Jr. What do all of
these people have in common? They all have biographies written about them.

『The Price of Greatness』(1995年)
those who became famous enough to have biographies written about them

一般に、ネクサスと後置修飾のどちらの解釈も可能な場合はネクサスが優先する、
と考えておくと良いと思います
0498名無しさん@英語勉強中 (ワッチョイ 7ff0-YC+D)
垢版 |
2021/08/24(火) 07:51:22.88ID:bS9joDHO0
>>471 以下の議論を整理すると・・・・まずアメリカさんの説明は、
次のことを言ってるんだよな。

>biographies written about themをhaveするってのは
「自分の伝記を書かせる」・・・・という意味にならない。

つまり
その意味の英文にするには、people などの名詞に言及して、即ち、付け加えて、
biographies written about them(selves) by people のようにしなければならないと。

なぜなら「自分の伝記を書かせる」の意味での ”受け身ではない” 英文は、
"have people write (auto)biographies about themselves"なのだから。
0499名無しさん@英語勉強中 (ワッチョイ 1f6c-zH/y)
垢版 |
2021/08/24(火) 10:42:09.78ID:VcmGWmb/0
>>497
個人がそういうルールで解釈するのはいいと思う

例文は後置修飾の例って考えられる

英語に
ネクサスが後置修飾より優先されるってルールがあるのかは知らない
0500名無しさん@英語勉強中 (ワッチョイ 1f6c-zH/y)
垢版 |
2021/08/24(火) 10:49:47.10ID:VcmGWmb/0
have their biographies written 
have biographies of them written
ならhave O Cの構文だとすぐわかる

英語表現がただしいかどうかは知らんが

write 〜 about [on] 〜という英語表現がある
write biographies about themを基にした
biographies written about them が考えられる
0501名無しさん@英語勉強中 (ワッチョイ 1f6c-zH/y)
垢版 |
2021/08/24(火) 11:00:44.85ID:VcmGWmb/0
Some 15,000 books have been written about Lincoln
これは 人[many people] have write some 15,000 books about Lincolnの受け身
Is History Written About Men, by Men?
Men write history about men.の受け身

Though many a biography of him has been attempted, ・・・
この表現あるから
have biographies of them writtenは通じるだろう

some people have many biographies written about them
この文も使役とか受け身とかhave O Cで解釈するのか?
0502名無しさん@英語勉強中 (ワッチョイ 1f6c-zH/y)
垢版 |
2021/08/24(火) 11:17:41.75ID:VcmGWmb/0
President Donald Trump has had quite a few books written about his presidency
これもhave O Cで解釈するのか?
0503名無しさん@英語勉強中 (ワッチョイ 1f6c-zH/y)
垢版 |
2021/08/24(火) 11:37:40.25ID:VcmGWmb/0
Who wrote the books about Trump?
How do I write a biography about myself?
Anyone can learn how to write a perfect biography essay about someone else's life
by making sure to focus on true, objective facts about a specific person.
writeと aboutはコロケーションの関係がある

Here are all of the books that have been written about the Trump White House so far.
write 〜 about 〜の受身形

a biography written about a person living in that country
What's he done to get a biography written about him?
後置修飾の例
0504名無しさん@英語勉強中 (ワッチョイ ff33-Awm0)
垢版 |
2021/08/24(火) 11:44:45.67ID:jJfl8fYQ0
>>502
・some people have many biographies written about them
・President Donald Trump has had quite a few books written about his presidency
いずれも「have O C」ですよね
「トランプさんがたくさんの本を所有している」という話ではないでしょ?

もう少し自分で整理してから書き込んだ方が良いと思いますよ
0505名無しさん@英語勉強中 (ワッチョイ 1f6c-zH/y)
垢版 |
2021/08/24(火) 12:03:20.83ID:VcmGWmb/0
>>504
ここのhaveを自宅に所有するという意味で解釈しなくてはいけない理由はない

Events, actions, experiences and activities
I’m going to have a bath.

もっとhaveの用法勉強してから書き込んだ方がいいんじゃないの?
0506名無しさん@英語勉強中 (ワッチョイ 1f6c-zH/y)
垢版 |
2021/08/24(火) 12:19:10.02ID:VcmGWmb/0
I'm going to have a bathのhaveを
All our luxury bedrooms have a private bath.
のhaveと同じ意味で解釈しなくてはいけない理由はない
0510名無しさん@英語勉強中 (ワッチョイ 1f6c-zH/y)
垢版 |
2021/08/24(火) 13:06:37.93ID:VcmGWmb/0
>>508
自分で考えたら?
have O Cのhaveでいんじゃね?
>>509
自己紹介するなよ
0511名無しさん@英語勉強中 (ワッチョイ ff33-Awm0)
垢版 |
2021/08/24(火) 13:07:14.40ID:jJfl8fYQ0
>>474
> 自分に関する伝記をもつ(=自分の伝記が書かれる)
> この解釈でなんでだめなんだ

イコールが成立しないからですよ
中途半端にSVOCの訳を取り入れてるだけで、そんなSVOの用法はありません
0512名無しさん@英語勉強中 (ワッチョイ 1f6c-zH/y)
垢版 |
2021/08/24(火) 13:27:56.84ID:VcmGWmb/0
>>511
自分が正しいと思ってればそれでいいんじゃねw
0513名無しさん@英語勉強中 (ワッチョイ ff33-Awm0)
垢版 |
2021/08/24(火) 13:37:04.25ID:jJfl8fYQ0
> 自分で考えたら?
> 自分が正しいと思ってればそれでいいんじゃねw

わかりやすい敗北宣言ありがとう

以前のポレポレにはミスがいくつもあって、
ちゃんとした指摘に対しては著者が改訂版で対応してるよ

あなたの指摘も、まだ誰も気づいていないミスなのかもしれない
著者に言ってみると良いと思うぞ。がんばれ
0520名無しさん@英語勉強中 (ワッチョイ 9fe2-DepB)
垢版 |
2021/08/24(火) 15:10:55.38ID:W7WQU9Qp0
・He has his critics.
・He has his criticisms.

意味ががらっと変わる。
0521名無しさん@英語勉強中 (ワッチョイ 9f3b-zH/y)
垢版 |
2021/08/24(火) 15:11:44.24ID:mQbSVU920
後置修飾でhave [biographies written about them]だと考えると
日本語で考えたら伝記を持つ→伝記が書かれたという含意もなくはないと思いますが
英語だと単純に何らかの形で手に入れたとか献本されたぐらいのニュアンスになってしまうのかもしれませんね
0522名無しさん@英語勉強中 (ワッチョイ 9fe2-DepB)
垢版 |
2021/08/24(火) 15:24:19.01ID:W7WQU9Qp0
○ There are many books on the subject.
╳ The subject has many books.

○ There are many biographies about Shakespeare.
╳ Shakespeare has many biographies.
0523名無しさん@英語勉強中 (ワッチョイ 1f6c-zH/y)
垢版 |
2021/08/24(火) 17:22:46.34ID:VcmGWmb/0
>>513
こいつ なにいっても自分が正しいって思いこんでるバカだから相手にしない方がいい
0524名無しさん@英語勉強中 (ワッチョイ 1f6c-zH/y)
垢版 |
2021/08/24(火) 17:33:15.71ID:VcmGWmb/0
最初の文のhave 所有のhaveとかって誤読するんだろうなw
受験レベルの英語もないやつ → (ワッチョイ ff33-Awm0)

How many biography does Lincoln have?
Some 15,000 books have been written about Lincoln — more books,
says Tetreault, than have been written about any other person in world history,
with the exception of Jesus Christ. Nearly 7,000 of these books are contained in the tower.
0526名無しさん@英語勉強中 (ワッチョイ 1f6c-zH/y)
垢版 |
2021/08/24(火) 17:48:28.68ID:VcmGWmb/0
受験英語レベルすらできないhave O Cマンは
用例だしても否定するマン

ネイティブの回答例
The Guinness Book of World Records says it’s Mohammed Ali
(the boxer, not the Prophet). See Muhammad Ali: His Life and Times

リンク先の英文
By 1980, according to The Guinness Book of World Records, Muhammad Ali had
surpassed Lincoln, Christ, and Napoleon as the most written-about person
in history.
0527名無しさん@英語勉強中 (ワッチョイ 1f6c-zH/y)
垢版 |
2021/08/24(火) 17:56:48.35ID:VcmGWmb/0
And since the kings were more likely than farmers to influence world events
and to have biographies written about them,

some people have many biographies written about them
President Donald Trump has had quite a few books written about his presidency
『Nonfiction』(2008年)
George Washington. Abraham Lincoln. Martin Luther King Jr. What do all of
these people have in common? They all have biographies written about them.

ここのhaveはモノの所有のhaveと解釈する必要がないことが明らかになったw
ネイティブはこういう英語表現で伝記が書かれるって解釈してるw
0530名無しさん@英語勉強中 (ワッチョイ 1f6c-zH/y)
垢版 |
2021/08/24(火) 18:39:59.00ID:VcmGWmb/0
ピューリッツァー賞とったひとがこういう英語表現つかってるけど?
0532名無しさん@英語勉強中 (ワッチョイ 1f6c-zH/y)
垢版 |
2021/08/24(火) 19:08:33.25ID:VcmGWmb/0
これ入試の和訳問題にしたらけっこう難問になるな

"Finally Muhammad Ali has a biography as big, complex, and memorable as the man himself
—or as close as any book can come. From panoramic views of Ali's place in racial,
political, and cultural conflicts, to gripping accounts of his fights, to vivid close-ups
of his outsized personality and relationships based on new sources, Ali will fascinate you
from beginning to end."
0534名無しさん@英語勉強中 (ワッチョイ 7f33-Awm0)
垢版 |
2021/08/24(火) 20:00:30.57ID:VZ0LGxih0
アメリカ人がいなくなりましたね。
0536名無しさん@英語勉強中 (オッペケ Sr73-YOT6)
垢版 |
2021/08/24(火) 23:28:23.79ID:7vdl+E0Tr
限定用法でしか使えない形容詞として「文から臨時に作った形容詞」が紹介され、その例として次の例文がありました
…it's so different from that inhuman you-mind-your-business-and-I-will-mind-mine of a city(それは、都会の、あの非人間的な「お節介をするな、こっちもお節介をしない」という態度とは大違いですよ)

これは限定用法の形容詞なのでしょうか。その後に名詞もありませんし。
ofの前にattitudeなどがあれば理解できるのですが
(現代英文法講義より)
0537名無しさん@英語勉強中 (ワッチョイ 7f33-Awm0)
垢版 |
2021/08/25(水) 00:30:13.96ID:Q9ugh9xv0
The following result is not in the least surprising.

文脈から多分、「次の結果は驚くに足らない。」というような意味だと思いますが、解説をお願いします。
0541名無しさん@英語勉強中 (ワッチョイW 1f89-JhKR)
垢版 |
2021/08/25(水) 02:57:14.32ID:yaANm3Lp0
1. She had servant [carry] the bags.

2. I'll have the taperecorder [fix] immediately.

[ ]内の動詞を適当な形に変えよという問題なのですが解説に1は原型不定詞でcarryのままで2はfixedとだけしか書いてありませんでした。
この場合2はなぜ原型不定詞にならないのでしょうか?
詳しく教えていただけると嬉しいです。
0542名無しさん@英語勉強中 (アウアウウー Sa63-BMF6)
垢版 |
2021/08/25(水) 03:04:29.35ID:DFrAyaRqa
能動の関係になっているか受動の関係になっているかで決まってくる

慣れないうちはservant「が」carryするから原形、taperecorder「を」fixするから過去分詞形という風に思っておけば、おおかた間に合うだろう
0545名無しさん@英語勉強中 (ワッチョイ ff89-zH/y)
垢版 |
2021/08/25(水) 07:28:41.67ID:ymJAEz070
>>541
>>1. She had servant [carry] the bags.

原文では、servant の前に the があるでしょ?原文はきちんと引用してね。
0546名無しさん@英語勉強中 (ワッチョイ ff89-zH/y)
垢版 |
2021/08/25(水) 07:29:40.28ID:ymJAEz070
>>541
the servant か、あるいは her servant だな。
0547名無しさん@英語勉強中 (ワッチョイ 7f33-Awm0)
垢版 |
2021/08/25(水) 07:32:01.95ID:Q9ugh9xv0
>>539

ありがとうございました。
0548名無しさん@英語勉強中 (ワッチョイ ff89-zH/y)
垢版 |
2021/08/25(水) 07:38:44.00ID:ymJAEz070
>>536
安藤貞雄「現代英文法講義」、p.478

それは、著者の安藤氏が of a city の前に attitude か何かの名詞があったはずなのにそれを
引用し忘れたか、あるいは原文の引用は正しかったけど、本当はそれを名詞として
紹介すべきだったのに、うっかりそれを「形容詞」としての用例に入れてしまっただけ
のことだろう?
0549名無しさん@英語勉強中 (ワッチョイ ff89-zH/y)
垢版 |
2021/08/25(水) 07:39:59.65ID:ymJAEz070
>>547
not in the least は頻繁に出てくるフレーズだが、そこらへんの英和辞典に熟語として
きちんと掲載されている。
0551名無しさん@英語勉強中 (ワッチョイ 9f3b-zH/y)
垢版 |
2021/08/25(水) 12:08:12.10ID:PLF5sF5Z0
さらっと調べた限り引用は間違ってなさそうです

あんまりいい例とは思いませんが、contraceptiveという単語を例に取ってみると
形容詞の用法(しかも限定用法のみ)もありますし(a contraceptive pill)名詞の用法もあります(an oral contraceptive)
語尾が-iveなので元は多分形容詞だったんだと思いますが、contraceptiveまでいえばだいたい何のことを指してるのか伝わりますし、
単語が単語なのでぼかしたとともとれますが、とにかくcontraceptiveまでで止めることが多くてそのうち名詞として扱われるようになったのではないかと思われます

深読みしすぎかもわかりませんが、
このように、形容詞だけで言いたいことが伝われば(あるいは後の名詞がだいたい何か予想できる場合には)形容詞だけで済ませることもあるということを安藤先生は言いたかったのかもしれません
0553名無しさん@英語勉強中 (ワッチョイ 9f96-hCLa)
垢版 |
2021/08/25(水) 19:25:25.73ID:mradPljP0
>Participation in elections is high. Yet if one asks the average Indonesian why she likes democracy, she will not praise it as a vessel for liberal values, but as a system for delivering prosperity.
https://www.economist.com/asia/2021/08/21/indonesias-president-promised-reform-yet-it-is-he-who-has-changed

ここでsheが使われてるのって何か理由あるんでしょうか?
国名や船名がsheで受けるのと似たようなもんでしょうかね?
0554名無しさん@英語勉強中 (ワッチョイ 7f9d-73jI)
垢版 |
2021/08/25(水) 19:37:04.65ID:9SFGyoJ+0
「Him coming is a problem.」という文のように、動名詞の意味上の主語が
目的格でしかもそれが文頭に置かれる場合があると聞いてビックリしたんですが
動名詞の意味上の主語として所有格はネイティブの会話などでほぼ使われないのでしょうか?
0555名無しさん@英語勉強中 (ワッチョイ 9f96-zH/y)
垢版 |
2021/08/26(木) 00:45:39.87ID:I1gWcf5N0
https://dotup.org/uploda/dotup.org2572205.mp3
これは何と言っているのでしょうか。
最初のrefleshmentは聞き取れたんですが、後がわかりません。
0557名無しさん@英語勉強中 (ワッチョイ 9f96-zH/y)
垢版 |
2021/08/26(木) 01:03:12.07ID:I1gWcf5N0
>>556
早速の回答ありがとうございます。
まさに下の文のようにしか聞こえなかったです...
0559名無しさん@英語勉強中 (ワッチョイ ff89-zH/y)
垢版 |
2021/08/26(木) 05:06:21.71ID:13d+6bI50
>>553
昔なら(たとえば1960年代終わりくらいまでなら)ためらいもなくここでは
he と書いていただろうね。

それを
he/she
s/he
he or she
などと書くようになった。しかしそれでは煩雑だから they (singular they) を使う
ようになって、それが現代ではかなり流布している。しかしときどき
she を使う人もいる。女性ライターがそう書く時が多いのかどうかは知らない。
昔はためらいもなく he をみんなが使っていた傾向への反発としてそういう
動きが見られるのだろうと思う。

よく似た用例を検索するのは俺にとってはちょっと難しいけど、たまたま引っかかったものを
列挙してみる。

(1) I have seen clients recommend that they ask a worker whether ●she● is registered and, if she will not provide the proof, then try to use that ...
(オーストラリアの公式サイトより)

(2) he had to know it was wrong to even ask a coworker if ★she★ wanted to see him jack off, let alone to do so.
(新聞サイトに寄せられた個人のコメントより)

You tell a coworker ◆she◆ is ripe and let’s see how long you are employed.
(新聞サイトに寄せられた個人のコメントより)
0560名無しさん@英語勉強中 (ワッチョイ ff89-zH/y)
垢版 |
2021/08/26(木) 05:12:00.85ID:13d+6bI50
>>554
>>動名詞の意味上の主語として所有格はネイティブの会話などでほぼ使われないのでしょうか?

「ほぼ使われない」とまで言えるくらいまで俺は書き言葉と話し言葉との両方に
通暁しているわけではないけど、確かに会話文では所有格は使われにくく、
圧倒的に目的格が出てくるのを見る。まあ、面白い現象だとも言えるだろうな。
興味がある人は自分でそういう用例をしっかり探してレポートを書いてみたらいいだろうな。
いや、そんなことをしなくても、どうせそこら辺の大学院生あたりがそういう
論文をじゃんじゃん書いているだろうけど。
0561名無しさん@英語勉強中 (ワッチョイ ff89-zH/y)
垢版 |
2021/08/26(木) 05:19:16.24ID:13d+6bI50
>>554
動名詞の意味上の主語(らしきもの)として所有格じゃなくて目的格を使っている
気楽な用例としては、たとえば次のようなものがある。

(1) Me singing "Firework" by Katy Perry
(自分が歌を歌っているところを見せるビデオを投稿している)

(2) Me Eating a Sandwich
(自分がサンドイッチを食べているところを見せている)

そういうのは、実にたくさん YouTube 上で出回っている。
0565名無しさん@英語勉強中 (ワッチョイ 7f33-Awm0)
垢版 |
2021/08/26(木) 16:24:02.30ID:3uQfuCg60
myはなぜ代形容詞ではなく、代名詞なのですか?
0566名無しさん@英語勉強中 (ワッチョイW 1f92-p/uk)
垢版 |
2021/08/26(木) 16:47:07.31ID:0Gqq7Zo/0
Not the least of its virtues is that it destroys base people as certainly as it fortifies and dignifies noble people.
の訳ですがnot the least を全然ーないとせずになぜ
その美徳の最小でないものは、として、金の美徳は、貧乏人を破滅させることだという訳になるんでしょう?
0567名無しさん@英語勉強中 (ワッチョイ ff89-zH/y)
垢版 |
2021/08/26(木) 16:54:44.38ID:13d+6bI50
>>566
(1) not in the least = not at all
(2) not the least = 最小のものではない

上の2つは、まったく違う。

the least は、「最小のもの」
0568名無しさん@英語勉強中 (ワイエディ MM8f-zH/y)
垢版 |
2021/08/26(木) 17:33:52.13ID:ZqPMAgRmM
すみません。でる1000問の476センテンスに

Tanton City passed legislation requiring all drivers age 75 and older to pass a vision test before renewing their driver's license.

とあり、この中の all drivers age 75 and older は訳が「75歳以上の全ての運転者」となっています。
この age はどのような文法で解釈すれば良いか分からず困っています。
どなたかよろしくお願いします。
0571名無しさん@英語勉強中 (ワッチョイW 1f92-p/uk)
垢版 |
2021/08/26(木) 17:39:50.49ID:0Gqq7Zo/0
>>553
平均的なインド人と言ってもアンケートをとって平均出してないだろう
今コロナで大変なのにどうやってアンケートなんか取れるのか、と思う。
記者が、普通のインド人を捕まえたと思ってるだけ。その一人は
駅を歩いている女性だった、みたいなことでしょ。
0573名無しさん@英語勉強中 (ワッチョイ ff89-zH/y)
垢版 |
2021/08/26(木) 18:10:18.41ID:13d+6bI50
A woman age 32 was...
これは
a woman (age 32) was...
a woman, age 32, was...
というふうに書くこともあるだろ?ということは、age 32 の部分がもともとは
分詞構文なんだろうと思う。つまり
a woman, her age being 32, was...
という形が大昔からあって、それが age 32 だけになったのだろう。

これと同じように、分詞構文が元の形からかなり簡略化されたものがたくさんあるだろ?
たとえば

The male driver, also ★age unknown★, of the other vehicle was airlifted to Saint John’s Hospital with unknown injuries.
https://taylorvilledailynews.com/local-news/588593

The other memorable moment comes late in the film from an old clip, ●origin unknown●, of a younger Tina Fey doing a tongue-in-cheek confessional, where
https://taylorvilledailynews.com/local-news/588593

こういう省略表現として定着した言い回しは、たくさんあると思う。
0575名無しさん@英語勉強中 (ワッチョイ ff89-zH/y)
垢版 |
2021/08/26(木) 18:23:49.42ID:13d+6bI50
cap in hand
humbly asking for a favour.
"we have to go cap in hand begging for funds"

上のような idiom もあるだろ?これも元は分詞構文だったんだろうよ。
with his cap in his hand だったんだろうけど、それが簡略化したんだろう。
0576名無しさん@英語勉強中 (ワッチョイ ff89-zH/y)
垢版 |
2021/08/26(木) 18:32:01.10ID:13d+6bI50
Katie and her new boyfriend are so cute, always going around ★hand in hand★.

これももともとは、"with her hand in his hand" とか "her hand being in his hand" という
ような言い回しがどんどん省略されていったと考えたら面白い。

ただし本当にこういう言い回しがもともと分詞構文だったのかどうかを
突き止めようとしたら、大昔からの文献をつぶさに追跡して研究しないといけない
のはわかっている。

俺としては、別にこんなものどうでもいい。
aged 32 が age 32 に訛っただけなのか、her age being 32 が簡略化されたのか
なんて、どいうでもいいことだ。問題は、A woman, age 32, was.... という
言い回しが厳然と存在していて、自由自在にそれを使いこなせたらそれでいいのだ。
0577名無しさん@英語勉強中 (ワッチョイ 9f33-YC+D)
垢版 |
2021/08/26(木) 18:42:10.29ID:9VELSl6f0
妹がまだ20代なのに体が熊田曜子みたいなのめっちゃ気にしてるんだが
なんて励ましたらいいんだ
別に多少垂れててもそんだけでかくてウエスト細けりゃよくないかって思うんだが

ワクチンの注射すら打った後涙目だったのに整形したいって言いだした
0581名無しさん@英語勉強中 (ワッチョイ 9f9d-FcIJ)
垢版 |
2021/08/26(木) 19:13:32.87ID:cRuIq6Re0
When the cameras were out, the cat was everything but.
ここでbutで切れることは文法上ありえるのでしょうか?プリントなので、もしかしたら制作者のミスかもしれません。
また、ここでのcamera is outは普通にカメラが回ってない時という意味でしょうか?
0583名無しさん@英語勉強中 (ワッチョイ ff89-zH/y)
垢版 |
2021/08/26(木) 19:27:26.29ID:13d+6bI50
>>581
everything but [out] っていう意味だろうな。少なくとも普通は、everything but とか
anything but で、but の後に何もつけずにセンテンスを終わらせているときには、
その直前に出てくる名詞とか形容詞を指しているもんなんだけど、今回の場合は
本当に everything but out という意味なのかどうか、文脈がないと俺にはきちんとは
判断できない。

anything but をセンテンスの最後に持ってきた用例についての海外フォーラムでの
質疑応答なら、次のリンク先にある。
https://www.usingenglish.com/forum/threads/271700-The-actual-state-of-affairs-is-anything-but
0584名無しさん@英語勉強中 (ワッチョイ ff89-zH/y)
垢版 |
2021/08/26(木) 19:31:22.41ID:13d+6bI50
>>581
その文章の文脈が俺にはわからんけど、俺の勝手な想像では、もしかして
カメラを何台も屋外に持ち出して猫を撮影しようと張り切ってスタンバイしているのに、
猫が部屋の外へは出ようとせず、コタツで丸くなってるんじゃないかな?
0585名無しさん@英語勉強中 (ワッチョイ 9f9d-FcIJ)
垢版 |
2021/08/26(木) 19:33:26.14ID:cRuIq6Re0
>>582
ありがとうございます。全体としてはロックダウン中にシェルターから猫を引き取る人が増える一方で、責任がとれなくなって猫を返す人が増加しているという話です。

The staff said a cat is not just for lockdown.
When the cameras were out, five-year-old Mii(猫の名前) was everything but.
She was lucky enough to be adopted, but was back just 24 hours later.
次からシェルターに戻された経緯と話が展開されます。

ここの二文目はどのような意味になるのでしょうか?
0586名無しさん@英語勉強中 (ワッチョイW 1f92-p/uk)
垢版 |
2021/08/26(木) 19:34:37.95ID:0Gqq7Zo/0
誤植かもね。aged 35
とかくところをdを付けなかったという。
けっこうあるからね。しんぶんだと、誤植が。
0587名無しさん@英語勉強中 (ワッチョイW 1f92-p/uk)
垢版 |
2021/08/26(木) 19:35:35.96ID:0Gqq7Zo/0
but も誤植。
0588名無しさん@英語勉強中 (ワッチョイW 1f92-p/uk)
垢版 |
2021/08/26(木) 19:37:20.23ID:0Gqq7Zo/0
こういうのを破格文というんだね
0589名無しさん@英語勉強中 (ワッチョイ 9f9d-FcIJ)
垢版 |
2021/08/26(木) 19:39:34.89ID:cRuIq6Re0
>>583
ありがとうございます。
分かりづらくて申し訳ありません。>>582で文脈を補足させていただきました。
everything butで終わっているので、しばらくの間は引き取った人が猫を大切にしていたが〜みたいなニュアンスだと思うのですが
cameras were outとどのように繋がるかが検討つきません。
0591名無しさん@英語勉強中 (ワッチョイ 9f3b-zH/y)
垢版 |
2021/08/26(木) 19:58:56.36ID:2Q54yzMK0
everything butではなくてanything butだと仮定したら意味が通るかもしれません
あくまで一つの仮説として提示します

とある古本で買った本にanything butの例文があったので引用します
Those easy-to-use home medical tests can be anything but.
「こうした過程で手軽に使える診断テストは、とてもじゃないが使えない。」

これから類推するにご質問の文は、「写真を取り終わったら、その猫はお払い箱になった。」のような意味なのではないかと想像します
0592名無しさん@英語勉強中 (ワッチョイ 9f9d-FcIJ)
垢版 |
2021/08/26(木) 20:35:08.68ID:cRuIq6Re0
>>591
ありがとうございます。
everything butからそのような意味と解釈することはできないでしょうか?
カメラで写真を撮ってすぐに飽きられたという話なら理解できるのですが、cameraに定冠詞がついてる理由などもあるのでしょうか?
cameraがoutというコロケーションがあんまりピンとこないのですが、一般的にはカメラが使われているとか、壊れているというニュアンス以外の可能性も考えられますか?
0593名無しさん@英語勉強中 (ワッチョイ 9f3b-zH/y)
垢版 |
2021/08/26(木) 20:38:23.60ID:2Q54yzMK0
>>566
not the least ofを「最小でないもの」と訳してもあんまりぱっとしませんよね
「中でも見逃すことができないのは」あるいはもっと簡単に、「とりわけ」などとしておくのが良さそうです

https://eigo.rumisunheart.com/2017/07/07/not-the-least-of/
こちらを参考にしました

ウィズダムに載ってた用例も載せておきます
Well, several reasons exist, not the least of which is he's a fraud.
「そうですね、いくつか理由があります。とりわけ大事なのは彼は詐欺師だということです。」

他にもlast but not leastといえば(もともとの意味:最後に述べるが決して軽んずべきことではないことであるが)→「大事なことを言い残したが」、「最後になりましたが」
という表現もありますね
比較級・最上級は複雑怪奇なのでまだまだ知らないことが多くて難しいですね…
0594名無しさん@英語勉強中 (ワッチョイ 9f3b-zH/y)
垢版 |
2021/08/26(木) 21:07:16.27ID:2Q54yzMK0
>>592
everything but〜は「〜以外なら全部」という意味ですね
イディオムとしてはeverything but the kitchen sink「なんでもかんでも」があるようです
everything butで考えたときに私の狭い想像力ではイメージが湧いてこなかったためやむなくanything butの方で解釈しました

outについても確かに「(物が)壊れた」の意味もありますよね
私もまずこの可能性を考えましたが、なんとなく「写真を取り終えた」の方がいい気がしたのでそっちで訳しました
「カメラが壊れてしまって、猫がいらなくなってしまった」と考えても文意は通ると思います

camerasに定冠詞がついてる件については、引き取り手の持っているカメラを指しているのではないでしょうか?
camerasと複数形になってるのはたしかに引っかかりますね。「壊れた」の意味で取るとそんなに一斉にカメラが壊れることがあるのかと疑問になりますね。

ただしcameraに「写真を撮ること」という意味はさっと調べた限りなかったので(あったとしても複数形にはならない気がします)私の解釈も微妙だと思います・・・
0595名無しさん@英語勉強中 (ワッチョイ 6e89-4thN)
垢版 |
2021/08/27(金) 05:34:56.60ID:DSEN9h9U0
>>585
まず、anything but と everything but との微妙な意味の違いについては、この下の
リンク先に説明がある。

(1)
https://jakubmarian.com/all-but-vs-anything-but-vs-everything-but-in-english/
(2)
https://grammarist.com/usage/all-but-vs-anything-but-vs-everything-but/

>>The staff said a cat is not just for lockdown.
>>When the cameras were out, five-year-old Mii(猫の名前) was everything but.

さて、少し多めに文脈を示してくれたので、考え直してみる。上の2行を読む限りでは、
もしかして次のような意味合いではないかという気がする。

野良猫を新たに引き取ってくれる人を探す団体のスタッフが、「猫というものは、
ロックダウン中にだけ引き取って、ロックダウンが解除されたら都合よく
手放すなんてことをするためのものではありません (A cat is not just for lockdown.)」
と言っている。

住民たちもその原則を守ろうという気配を、少なくともマスコミによる
取材中 (when the cameras were on) には見せる。しかし記者たちのカメラが回っていないとき
(when the cameras were out)には、途端に住民たちの自分勝手な本音が露わになり、
猫は「ロックダウン中にだけ都合よく利用されるもの」(everything but "not just for lockdown")
となってしまう。だから24時間ほど経ったらさっそく猫をシェルターに返すという
不届きな連中が現れる。

以上のようなことを考えてみた。新たに別の文脈などを質問者が提示してくれたら、
またもやこのような俺の想像や推測がすべて覆されるかもしれないけど、
一応は中間報告しておく。
0596名無しさん@英語勉強中 (スップ Sd22-3DV1)
垢版 |
2021/08/27(金) 06:52:03.98ID:7L3ZAaFVd
お願いします
下記の文章はWSJのリード文です
そのために文末にピリオドはありません
but以下ですが、toldが過去形になっています
その後のthat節ではobjectが述語動詞かと思われますが、何故ここはobjectedではなくobjectsと現在形なのでしょうか?
自分の中で「時制の一致」や間接話法についてごっちゃになっているのかも知れませんが、非常にもやもやしているのでどなたか回答よろしくお願いいたします

After Afghan withdrawal, U.S. military hopes to position forces temporarily near Afghanistan, but Putin told Biden that Moscow objects
0598名無しさん@英語勉強中 (ワッチョイ 82f0-4h9J)
垢版 |
2021/08/27(金) 07:38:25.64ID:++lhk/ni0
>>585
「Mii がシェルターに戻される経緯と話」の部分の英文かその日本語による説明を
記載した方が、回答者たちももっと的を絞った回答ができると思う。

というのは、質問の段落の英文で、シェルターにいる猫(あるいは猫一般)のこと
についてスタッフが語ったのか、あるいは、a cat が意味を持ち、 特に Mii のこと
を取り上げて話題にしているのかがそれによって明らかになるし、その違いで、
一文目、二分目の解釈が多少違ってくるかも知れない。・・・・まあ前者だとは
思うけど、念のため・・・・・。
0599名無しさん@英語勉強中 (ワッチョイ a93b-7gXt)
垢版 |
2021/08/27(金) 08:19:08.15ID:O934V29a0
これがニュースかインタビューの映像を説明している段落で、
「カメラが引きの画(ズームアウト)になったとき、他に映っていたのはMiiだけだった。」という解釈も可能かもしれません。
それでその後に三文目でその猫の説明がされるという流れです。

カメラが複数形になっているのは複数のカメラで撮影していたから(1カメ、2カメ…のような具合で)
everything butのbutの後ろにはthe staffが省略されていると考えました。
staffというのは日本語のスタッフと違って職員全体を指すというのがこの説の難点ですが…
0600通りすがり (ワッチョイ 4531-3005)
垢版 |
2021/08/27(金) 13:00:51.79ID:9OBiGCap0
>>596
詳しい文法書の「時制」の中の「現在時制」の項を読めば載ってると思うけど。

いわゆる「常習、慣行的動作は現在形を用いる」に当たると思う。

ロシアは過去に反対したし、現在も反対であるし、将来もほぼ間違いなく
反対するであろうから、「常習、慣行」的な振る舞いに当たる。だから現在形。
0601名無しさん@英語勉強中 (ワッチョイW 4d92-llB9)
垢版 |
2021/08/27(金) 15:54:59.98ID:Ua16qsY40
>>599
インスタに載せる写真が欲しいだけで、とったらお払い箱というのは
けっこうあると思う。例:タピオカドリンク屋。
コロナで在宅勤務の間はペットを飼って、在宅が明けたら要らないから返す
も結構あるみたい。多くの人間って自己中なんだね、とニュースあった気がするし
誰でも知っているはず。だけどそう取らずに何か意味があるはず、
と考えるのがスレの趣旨にあってるということなんだろうな
0602名無しさん@英語勉強中 (ワッチョイW 4d92-llB9)
垢版 |
2021/08/27(金) 16:09:55.14ID:Ua16qsY40
>>600
黄色ロイヤルには今も当てはまる事柄を表す時に意味を明確にするために現在形又は未来形のままにしておく方が普通である
と書いてたよ。Did you know the film is now out in DVD?という例文が載ってた。
0603名無しさん@英語勉強中 (ワッチョイW 4d92-llB9)
垢版 |
2021/08/27(金) 16:23:55.08ID:Ua16qsY40
>>593
I wasn’t the least frightened.
少しも怖くなかった
もあるし、

他方でnot leastの項目で
とりわけ,特に
Trade has been bad, not least because of the increased cost of raw materials.
商売はこのところ振るわないが,それはとりわけ原材料費が高くなっているため
とあるのですが、
今の場合、not the least of whichでとりわけなので複雑ですね。
うまく切り分けができていないというか。
0604名無しさん@英語勉強中 (ワッチョイW 4d92-llB9)
垢版 |
2021/08/27(金) 16:27:38.66ID:Ua16qsY40
but って次の行のbut と同じ意味なんだと思うけど、
一行目と二行目がうまく繋げてない。
こういう場合どう文を書き直せばいいんだろうかと思う。
猫は全てだ。だけど、、、
みいは拾われてらっきーだったけど、だけど24時間で戻されて、、、
0606600 (ワッチョイ 4531-3005)
垢版 |
2021/08/27(金) 21:50:25.33ID:9OBiGCap0
>>602
そう、その「今も当てはまる事柄を表す」用法の可能性は自分も考えた。

ここからは、時事問題の知識からの類推が考慮に入ってくる。

アメリカがアフガニスタンの近くに軍を配置することをロシアは決して認めない
ーーー 過去も認めなかったし、現在も認めないし、今後も決して認めないで
あろう、そういうプーチンの強い意思を、この文章から自分はどうしても読み
取ってしまうのだ。

思い込みにすぎないかもしれないので、>>600の解釈が絶対に正しいと言い張る
つもりはない。質問者は好きな方を選んでもらってかまわない。
0607名無しさん@英語勉強中 (スッップ Sd22-3DV1)
垢版 |
2021/08/28(土) 12:32:29.61ID:69NYjCh0d
>>596
but Putin told Biden that Moscow objects

Moscow will/must object
がプーチンさんの言外にはあるように思えます

このobjectが時制の一致により過去形や過去完了になると意味的におかしくはなってしまいますよね
「反対した」わけではないですもんね

安藤先生の基礎と完成の言葉では「普遍的な習慣」、「現在も変わらぬ事実」が提示されているときは「時制の一致を適用しなくてもよい」と書いてあります
0609名無しさん@英語勉強中 (スッップ Sd22-3DV1)
垢版 |
2021/08/28(土) 12:36:16.60ID:69NYjCh0d
やはり英文法て奥深いなあ…
青ロイヤルは持ってるんですが黄色ロイヤルも手元に置きたくなりました
安藤先生基礎と完成でわからない箇所を青ロイヤルで確認したりしています
0612名無しさん@英語勉強中 (ニククエ Sac9-PO1v)
垢版 |
2021/08/29(日) 18:27:59.38ID:1iQemgB2aNIKU
「サミング・アップ」行方昭夫先生の誤訳? asとmight haveの用法 - 英文解剖学
https://tmneverdies.hatenablog.
com/entry/2021/08/29/070719

どう思う?
個人的には結構納得したけどシンプルな短文を長々と説明的に訳さざるをえない解釈は大概誤訳の確率が高い気もする
しらんけど
0620名無しさん@英語勉強中 (ニククエW 928a-uITW)
垢版 |
2021/08/29(日) 23:01:12.75ID:SkVlw3Jd0NIKU
>>612
全然興味ねええええええと思ったけど一応その後読んでみたわ
結論としては誤訳というか、訳がヘタクソという感じなんじゃないの
まぁ誤訳といえば誤訳かもね
てかこの長い分析自体に一体何の意味があるのかと思ってしまった
めちゃくちゃわかりにくい文ならまだしも、読んでその場で普通に意味わかるやろこれ
0622名無しさん@英語勉強中 (アウアウウー Sa85-6CFN)
垢版 |
2021/08/30(月) 01:44:05.68ID:IqEM7o+pa
日本語でもうっかり20000万円などとやらかしてしまことも珍しくはないし、同じような感覚なのかもしれない
しかし本当に2,500,000,000,000dollarsのことを言っている可能性も否定できないので、最終的には空気を読んで解釈する必要があると思う
0623名無しさん@英語勉強中 (アウアウウー Sa85-FhuH)
垢版 |
2021/08/30(月) 06:51:36.62ID:scgs340Ma
二十歳の誕生日にプレゼントを送ります
メッセージカードを添えて、誰からも愛される素敵な大人になってねって書きたいんだけど、
become a wonderful woman loved by everyoneで大丈夫かな?
womanをadult、byをfromの方が 良いですか?
0625名無しさん@英語勉強中 (ワッチョイ 6e89-4thN)
垢版 |
2021/08/30(月) 07:59:20.03ID:MZFdWW/00
>>623
>>become a wonderful woman loved by everyone

俺だったら、そういう書き方を避ける。なぜかというと、「みんなに愛されるような
素敵な大人になってね」という言葉は、一応は日本では受け入れられているけど、
他のまったく違う文化圏の人に受け入れられるかどうか不安だからだ。
そういう大人に「なってほしい」ということは、今はまだそういう人ではない、
つまりあなたはまだ大人じゃないし、さらには素敵でもないし、みんなに愛されている
わけじゃない。だから努力しないとそうなれないよ、と言っているのだと解釈でき
なくもないからだ。だから俺なら、次のように書く。

(1) Happy birthday to you!
You're a wonderful woman worthy of the love of everyone.

(2) Happy birthday to you!
You're a wonderful woman. I'm positive everyone loves you.
0626名無しさん@英語勉強中 (ワッチョイ 6e89-4thN)
垢版 |
2021/08/30(月) 08:01:57.82ID:MZFdWW/00
>>623
>>womanをadult、byをfromの方が 良いですか?
woman でも adult でもいいだろうと思う。become a ... woman loved ★from★ everyone は間違い。
0631名無しさん@英語勉強中 (ワッチョイW c996-rm36)
垢版 |
2021/08/30(月) 16:30:07.95ID:Lh5BAVQp0
https://englishhub.jp/news/gabastyle-synonym-must-haveto.html

“must”と“have to”の使い分け
話し手の強い意思により「しなければならない」場合は “must”
規則や状況などの外的要因により「せざるを得ない」場合は “have to”
“must” はとても強い表現なので、基本的には “have to” を使うほうが無難


この記事、間違ってないか?
0632名無しさん@英語勉強中 (ワッチョイ 82f0-U7Lh)
垢版 |
2021/08/30(月) 16:51:20.75ID:xPI1CdI70
学校英語の知識でグダグダ
しょーもないスコラ哲学
0634名無しさん@英語勉強中 (ワッチョイ 82f0-q8ax)
垢版 |
2021/08/30(月) 20:20:02.71ID:UQZG20DS0
>>612     
問題の文章は、
今は作家だけど、仕事として選んでその作家を続けているこの流れはたまたまで、
医者か弁護士か作家になるという選択の可能性はすべて同等だったということを
示唆しているので、・・・・・・、行方先生の訳文は飛躍しているようだけど、
誤訳ではないんじゃないの。

ざっと読んだだで、結論を書き込むのは恐れ多いが、・・・・・いずれにせよ、
解説にあるような複雑な分析を要するような文章というより、それなりのネイティブ
なら一読して理解できるレベルの文なんじゃないのかい?
0637名無しさん@英語勉強中 (ワッチョイ 82f0-q8ax)
垢版 |
2021/08/30(月) 21:05:10.78ID:UQZG20DS0
>>635
それも考えたけど、
あなたの指摘のように、あのブログの解説者の解釈であるし、問題の
最初の文章の意味を読み解く上で必要なのかな。

確かに、モームのあのエッセイには以下にもそれらの職業の特徴について各々
いろいろ言及しているようだけど、それはまた別の話なんじゃないの?

(ややこしいので、あまり深入りしたくないけど、後でもう少し考えてみるわ、
 今日はこれくらいで・・・・・)
0640😏三年英太郎😉 ◆3CZBjOt3.Y (ワッチョイW 0286-/HQx)
垢版 |
2021/08/30(月) 23:49:21.96ID:pxhJY/6B0
法助動詞の根源的&認識的用法は伝統文法超えてはいるが、そのブログの問題点は、一言で言えばクソつまらないとこだよね🤔

クソつまんないものを我慢して読んで、それで得られる報酬もちっちゃ過ぎる😨
0642名無しさん@英語勉強中 (ワッチョイ 4d92-ihe+)
垢版 |
2021/08/31(火) 00:35:16.63ID:JUuVk2xX0
医者をやってたんでしょう?
でも今は作家をしている。
医者をしていたかもしれないように、今作家をしている。
0643名無しさん@英語勉強中 (ワッチョイ a93b-4thN)
垢版 |
2021/08/31(火) 07:02:15.71ID:gQXtXYiF0
I might have been a rich man. (なろうと思えば)金持ちになれたものを 【新英和大】
might-have-been (実現が期待された)そうなったかもしれないこと; (もっと)ひとかど[有名]になったかもしれない人.【ジーニアス】

might have beenにはこのようなニュアンスが含まれてるっぽいですね
それをもとに考えると対照のasが近いような気がします…(現実と理想の対比みたいな感じ)
理想というと語弊があるかもしれないので、潜在的可能性と言い換えておきます。
0644名無しさん@英語勉強中 (ワッチョイ 0633-Ctl7)
垢版 |
2021/08/31(火) 08:30:59.21ID:uqC18wH90
ブログ主の主張:
「過去の人生の選択次第では職業として『医者や弁護士』を営む可能性もあったが、
現実の私はその場合と同様に職業として作家業を営んでいる」

「私の職業の潜在的可能性」という点では、ブログ主も同意見
ただ、ブログ主はasを「様態のas」だと考えている
0645名無しさん@英語勉強中 (オッペケT Sr51-U7Lh)
垢版 |
2021/08/31(火) 09:07:21.78ID:eylMhYArr
英語そのものではないのですが、今イギリス人と英会話のレッスンをしていて、
時々イギリスの歴史の話になります。

XXの戦いでフランスと戦った将軍が着用していたからこの名が付いた

とか。

で、俺理系だったので高校で世界史やっていない(取りたかったけど理系は強制的に生物だった)
ので知識としてほとんど何もない状態です。

イギリスの歴史のいい入門書があったら教えて下さい。
0649名無しさん@英語勉強中 (ワッチョイ 0633-Ctl7)
垢版 |
2021/08/31(火) 11:11:08.16ID:uqC18wH90
>>647
自分はasの分類には興味がないので、特にブログ主にコメントしたいことはありません

行方さんの訳がベストとも思いませんが、このasは「同時性」というだけで十分だと思います
0653名無しさん@英語勉強中 (ワッチョイ c996-+pMA)
垢版 |
2021/08/31(火) 14:04:40.89ID:x/BDNPKf0
Insects were among the original specialty foods in the American gourmet marketplace—inspired, impractical provocations that, like runway styles in retail clothing, drove the sales of more basic goods. In the early nineteen-forties, Max Ries, a German-Jewish textile manufacturer, came to Chicago and established himself as a purveyor of imported cheese to an American public that was beginning to be fascinated by exotic food. Ries was slim and dashing; he wore handmade suits and twirled his cigars.
Alongside tinned tiger and elephant meat—culled from zoos and sold at department stores—he presented “French-fried ants” from Venezuela and baby bees from Japan, conversation pieces that lent glamour to his company, Reese Finer Foods, which actually made its money selling canned water chestnuts, artichoke hearts, and baby corn.

ここの”conversation pieces”って額面通りの意味で理解してもいいんですかね?
0657名無しさん@英語勉強中 (ワッチョイ 2157-4thN)
垢版 |
2021/08/31(火) 14:23:03.59ID:LkADvx1O0
英文解釈教室の7.3.2、原文はT.S.エリオット。頭のいいお前らに教えを乞うっす

It is just the literature that we read for "amusement," or "purely for pleasure" that may have the greatest influence upon us.

just は冠詞と名詞に挟まれていないから形容詞ではなく副詞、
It is の後に来ているのは実質的に名詞 the literature 。
そのあとの that まで見た時点では 以下の4つの可能性がある。
@ It that の形式主語(⇒ that 節内部は完全)
A 関係代名詞 That (⇒ that 節内部は不完全)
B It that の強調構文(⇒ that 節内部は不完全)
C 同格の that (⇒ that 節内部は完全)

その後 that 節の中を読み進めると、read の目的語がないので、A 関係代名詞 かB It that の強調構文。
ところがさらに読むとまた that が出てきて、しかもその中では主語が欠けている。すると、こちらもA関係代名詞 かB it that の強調構文。

どちらも関係代名詞だとすると二重限定だが、二重限定では同じ関係代名詞を使えないから関係代名詞の二重限定ではない。
とすると It thatの強調構文と関係代名詞が合わさっているはず。

さて、じゃあどっちが関係代名詞でどっちが強調構文なの?という話です。
勿論文脈で判断するってのも一つの方法だけど、文法的にどう考えるのかを知りたい。

とりあえず考えていることとしては、
二番目のthatが関係代名詞だとしたら、その先行詞はfor amusement or Purely for pleasure あたりと見込まれる。
しかし、もし or がなくて for amusement だけだったなら、前置詞の目的語である amusement という名詞が先行詞になっているとはいえるが、
for amusement or Purely for pleasure の並列を先行詞とする場合、amusement 以下を先行詞と考えるとしても、
amusement or【 for purely】 pleasure 、つまり【 for purely 】が残っちゃうので先行詞としては適切でない。
※ 換言すれば、もし for amusement or pleasure だったら先行詞となり得る

だからこの2番目の that は関係代名詞ではなく強調構文の that で、よって最初の that は関係代名詞である、
という説明くらいしかできないのかなあと思っているのだが、どうだろうか?
0658名無しさん@英語勉強中 (ワッチョイ 2989-3Axr)
垢版 |
2021/08/31(火) 14:38:51.23ID:thjQN+A60
0000-0500 ラテン語のローマ時代
0500-1000 古英語のアングロ・サクソン時代 Old English
1000-1500 フランス語のノルマン時代 Middle English
1500-1900 近代イギリス Modern English
1900-2000 Contemporary English

ローマ時代 ケルト系ブリトン人
410 ローマ、ブリテン島撤退
449 ヨーロッパ大陸からゲルマン民族(アングル、サクソン、ジュート)侵入
アングロサクソン王国 アルフレド大王(the Great)
1066 フランス、ノルマンディ公ギョーム2世がイングランド王を破りイングランド王ウイリアム1世として即位(the Norman Conqest)
12世紀後半-13世紀初 アンジュ―帝国
1215 マグナ・カルタ
1337-1453 百年戦争(英仏)
バラ戦争(ランカスター家ヨーク家)
1485 チューダー朝
エリザベス1世
1603 スチュアート朝
スコットランド王ジェイムズ6世がイングランド王ジェイムズ1世として即位
1649 共和政
チャールズ2世処刑 クロムウェル
1660 王政復古
1688 名誉革命
1707 スコットランド統合
1714 ハノーヴァー朝
ドイツハノーヴァー選帝候ジョージがイングランド王ジョージ1世として即位
第2次英仏百年戦争
産業革命
アメリカ合衆国独立
ヴィクトリア女王
ウィンザー朝に改称
0662名無しさん@英語勉強中 (ワッチョイ 0633-Ctl7)
垢版 |
2021/08/31(火) 15:28:44.97ID:uqC18wH90
>>657
・「二重限定では同じ関係代名詞を使えない」というのは誤り
・「It is 名詞 that S V」が関係詞か強調構文かは、形だけでは決まらない(文脈依存)
・657の例文は教室の説明どおり、1つ目のthatは関係詞、2つ目のthatは強調構文(文脈依存)
0670名無しさん@英語勉強中 (ワッチョイ a93b-4thN)
垢版 |
2021/08/31(火) 17:55:43.26ID:gQXtXYiF0
>>657
本を読むことで人生観が広がるとはいうが、
そのためにはシェークスピアやダンテ、ゲーテ、エマーソン、カーライル、その他錚々たる面子の著書と本腰を入れて対決しなければならず、
The rest of our reading for amusement is merely killing time. だと人は考える。
だが、私はというと…
But I incline to come to the alarming conclusion that it is just the literature that we read for “amusement,” or “purely for pleasure” that may have the greatest and least suspected influence upon us. It is the literature which we read with the least effort that can have the easiest and most insidious influence upon us.
という文脈ですね。

たしかに、英文解釈教室に掲げられた部分のみでは二重限定にも取れなくもないですね
しかし、元の文章を読めばなんとなく直感的に強調構文であることがわかると思います。
https://www.proginosko.com/2017/05/t-s-eliot-on-reading-for-amusement/
0671名無しさん@英語勉強中 (ワッチョイ 525f-Ctl7)
垢版 |
2021/08/31(火) 22:22:24.96ID:3+nU7KB70
英語を口に鳴らすために独り言で英語を話すようにしているのですが、
その際、言いたい英語の文面(字面)が頭に浮かんでしまうのですが、これはやはり習熟度が足りないことに起因するのでしょうか?
単に、口をついて出るほどにフレーズを完璧に覚えられていないため頭の中で思い出そうとしていることが原因だと思うのですが、
「字面が頭に浮かぶ」ということが未熟な学習者に当てはまる一般的な現象なのか気になりました。
同じような経験をした方は他にもいらっしゃいますか?また練習を重ねればそういったことはなくなりましたか?
0675名無しさん@英語勉強中 (アウアウウー Sa85-6CFN)
垢版 |
2021/09/01(水) 17:04:17.10ID:oKiQeBnea
>>671
文字媒体で覚えた表現を使おうとしているからだと思う
もしも映画等で覚えた言い回しを使ってみようとして文字が浮かんできたのであれば、英語を頭で考えすぎなのではないだろうか?
0677名無しさん@英語勉強中 (ワッチョイ 525f-Ctl7)
垢版 |
2021/09/02(木) 01:07:02.12ID:KokxTyn90
>>675
確かにその傾向は強いかもしれないですね…
やはり文字を媒介せず、母語でも無意識にやっているように、感情と口を直接結びつける練習が必要なようです
ありがとうございます
0678名無しさん@英語勉強中 (ワッチョイ 2989-q8ax)
垢版 |
2021/09/02(木) 19:54:19.90ID:3QKWIFMV0
おねがいします
I think our new fan heater would appeal to elderly people and families with them.
という英文のwith themは、どのように解釈して日本語にすればよいか教えてください。
0680名無しさん@英語勉強中 (ワッチョイ 2989-q8ax)
垢版 |
2021/09/02(木) 20:02:02.18ID:3QKWIFMV0
>>679 ありがとうございます。
「お年寄りと、彼らと一緒にいる家族」の心をとらえる
のような感じでよろしいでしょうか。
0681名無しさん@英語勉強中 (アウアウウー Sa85-6CFN)
垢版 |
2021/09/02(木) 20:45:08.09ID:tRza0vq5a
横からで済まないが、
「お年寄りと、お年寄りのいるご家庭」、と言った方が正確だと思う。
考えすぎかもしれないが、その訳だと、「お年寄りとその家族」とも読める。
0682名無しさん@英語勉強中 (ワッチョイ 8796-lMUy)
垢版 |
2021/09/03(金) 00:29:50.00ID:FTz7c4+20
“What a pleasant change from the labourer’s unvarying meal of bread, lard, and bacon, or bread and lard without bacon, would be a good dish of fried cockchafers or grasshoppers”
これwhat a X S + Vの感嘆文だと思うんだけど、S + Vの部分が倒置してるんです?
文構造どうなってるか教えて下さい
0683名無しさん@英語勉強中 (アウアウウー Sa8b-fLcm)
垢版 |
2021/09/03(金) 01:04:56.59ID:jFv+Q2QZa
How pretty is this flower!
ロイヤル英文法に拠れば、このように文語調では、V+Sの語順でも可能であるらしい。
あるいは、Sにあたる部分が、長い名詞句になっているために、倒置したとも考えられる。
0687名無しさん@英語勉強中 (ワッチョイ 873b-Sfec)
垢版 |
2021/09/03(金) 20:41:49.30ID:ine/xkuU0
MRS. Rachel Lynde lived just where the Avonlea main road dipped down into a little hollow, fringed with alders and ladies’ eardrops and traversed by a brook that had its source away back in the woods of the old Cuthbert place;
it was reputed to be an intricate, headlong brook in its earlier course through those woods, with dark secrets of pool and cascade;
but by the time it reached Lynde’s Hollow it was a quiet, well-conducted little stream, for not even a brook could run past Mrs. Rachel Lynde’s door without due regard for decency and decorum;
it probably was conscious that Mrs. Rachel was sitting at her window, keeping a sharp eye on everything that passed, from brooks and children up, and that if she noticed anything odd or out of place she would never rest until she had ferreted out the whys and wherefores thereof.

最後の方にある from brooks and children upはどういう意味でしょうか?なんとなくfrom brooks to childrenという意味な気はするのですが確証はないです

出典は赤毛のアンの冒頭部分です
https://www.gutenberg.org/files/45/45-h/45-h.htm
0688アメリカ人 (ワッチョイW 7f05-FaOx)
垢版 |
2021/09/03(金) 21:04:48.94ID:n1AR9ArM0
>>687
そうですね、「from brooks to children」がいいです。
この本は1908年のものなので、そういう紛らわしい古い話し方もあるんですよ。
僕は英語の母国語でも、そのようなものは現代の作品よりもはるかに読みづらいものです。
0693名無しさん@英語勉強中 (ワッチョイ 5fe2-Sfec)
垢版 |
2021/09/03(金) 23:08:18.19ID:tkgHuZxx0
In May, Mr. Mike is to travel to Japan to meet with company executives to discuss similar plans for Italia and the China.

Mike氏は5月には、イタリアと中国における同様の計画について会社の幹部と意見を交わす会談のために日本を訪れる。


この文章の
In May, Mr. Mike is to travel to Japan to meet〜

be+分詞 で to 〜to〜  が続くんだけど、意味が分かりません
文法的にどうやって理解すればいいんですかね?

In May, Mr. Mike travels to Japan to meet〜
ならわかりますが
なんで is to travel なんでしょうか?倒置法?みたいなので文法的に説明がつくのでしょうか?
0695名無しさん@英語勉強中 (ワッチョイ 873b-Sfec)
垢版 |
2021/09/03(金) 23:47:23.86ID:ine/xkuU0
>>688
ありがとうございます。古い言い回しは辞書にも載っていないことが多いので、やっぱり今の英語にたくさん触れて、見たことのない言い回しであっても大体の意味を推測できる勘を育てていかなければならないと痛感しました。
it probably was conscious that...についてもit is certain that...のようなものだと思っていて、it = a brookだと気づくのにかなり時間がかかったので、もう少しこの本は寝かせてもっと英語力をつけてからまた挑戦しようと思いました。

>>689
ありがとうございます。from the ground upという用例が辞書にあったので、もしかしたらここもfrom <brooks and children> up ということなのかもしれませんね。
ちょっと調べてみたんですが、新潮文庫の村岡訳(私は持っていません)は、「小川からこどもにいたるまで」となってるそうです。

>>692
「現代の作品よりもはるかに読みづらい」とおっしゃっているので、あくまで相対的に、ということだと思います。あまり強い言葉は使わないほうがいいと思います。
0696😏三年英太郎😉 ◆3CZBjOt3.Y (ワッチョイW 5f86-539L)
垢版 |
2021/09/04(土) 03:56:29.57ID:sEPcZ4Ux0
1.
andは並列で、着点を意味するtoを代用するとは思えない🤔
from Tokyo and Paris ≠ from Tokyo to Paris

2.
everything が from brooks to children のはずがない🤔
こーゆー詮索好きFBIオバちゃんの最大の関心は大人(のゴシップ)なのだ

従って、大人は当然のこと、家の前を通る水や子供でさえも、おばちゃんは見逃さないという意味でないのか🤔

現代英語で読めるものを、古い英語の言い回しとか言われたら、へ?🤯😳ってなりませんかね
0701名無しさん@英語勉強中 (ワッチョイW 5fa7-zLw1)
垢版 |
2021/09/04(土) 05:06:09.26ID:9vmtZmdP0
>>695
栄太郎さんの書いてるのが正解だけど、
こういう表現を理解するのは訳とか英語力とかそういうのと
違うと思った。ギャツビーでテニス女の顔の表現で、目尻が皺というのを
何か表現してたけど、その表現でアメリカ人が議論してた。
訳見たら訳さず飛ばしてたよ。
訳じゃなくて文学作品の解釈の問題なので、どんなに英語力を
つけようとしてもできるようにならないよ
0704名無しさん@英語勉強中 (ワッチョイ 7f89-lYlz)
垢版 |
2021/09/04(土) 05:35:00.86ID:Jh8RYgr/0
from brooks and children up に関連して、ちょっと検索してみた。

(1) I write "performance appraisals" for people from laborers up to engineers.
https://www.airwarriors.com/community/threads/o4-list.39377/page-7#post-707181

(2) ... where we are looking at everything from children up through and including the elderly .
https://books.google.co.jp/books?id=4kPWO1mAmycC&;pg=PA92&lpg=PA92&dq=%22everything+from+children+up%22&source=bl&ots=JQWLHKzKnJ&sig=ACfU3U3T_vrsX-tJ8sP8g13_UYHV8yr6Ow&hl=en&sa=X&ved=2ahUKEwjpxsOhz-PyAhUpG6YKHQLyAZ4Q6AF6BAgCEAM#v=onepage&q=%22everything%20from%20children%20up%22&f=false
0705名無しさん@英語勉強中 (ワッチョイW 5fa7-zLw1)
垢版 |
2021/09/04(土) 05:36:26.95ID:9vmtZmdP0
ニューロマンサーめちゃむずいと思ってたけど、
そうでもないみたいやね。
僕は英文法ができんから読めなかったりするいうことなんやろか
>>695のit is conscious that sv
はit is sure that svとかhe is possible to speakみたいにあかんやつやと
閃いたけどな
0707名無しさん@英語勉強中 (ワッチョイW 5fa7-zLw1)
垢版 |
2021/09/04(土) 06:15:10.45ID:9vmtZmdP0
婦人レイチェル・リンデが住んでいたのは、アボンレアの幹線道路が小さなくぼみに入り込んだところで、
そこにはハンノキやミズナラが生い茂り、旧カスバート邸の森に源を発する小川が横切っていた。
その森の中を通っていた頃は、複雑で急流のような小川で、プールやカスケードの暗い秘密があると言われていました。
しかし、Lynde's Hollowに到着する頃には、静かでよく流れる小さな流れになっていました。
レイチェル・リンデ夫人の家の前を、礼儀と節度を無視して小川が流れることはありませんでしたから。
レイチェル夫人が窓辺に座って、小川や子供たちまで、行き交うすべてのものに目を光らせていて、
何か変なものや場違いなものに気づいたら、その理由や原因を解明するまで休むことができない、
と意識していたのでしょう。

DeepL訳
すごく上手だよ
0709名無しさん@英語勉強中 (ワッチョイ bf23-nTGN)
垢版 |
2021/09/04(土) 07:06:46.47ID:pVLF5JbG0
英文法解説ってやたら神格化されてるイメージあるんだけど、今はアレ読むなら黄ロイヤルとか読んだ方がよくね?
現代英文法講義はレベルがかなり上の文法書になるから除外するとして
0713名無しさん@英語勉強中 (ワッチョイ 5f33-/op0)
垢版 |
2021/09/04(土) 10:06:04.74ID:3B43IuqX0
たとえば、数学の場合、大人が高校生向けの参考書を読むということはあまりありませんが、
英文法の場合、なぜか大人でも皆高校生向けの英文法書を読みます。

大学生やそれ以上向けの分かりやすい英文法書ってないんですか?
0714名無しさん@英語勉強中 (ワッチョイ e79d-nTGN)
垢版 |
2021/09/04(土) 10:11:25.49ID:bRkBAnDU0
decideとdetermineの違いについて質問です。
「首相は解散を決断した」を表現する場合、

A) 順風満帆の政権中期で、ここなら有利だろうと思って解散を決めた
B) 政権末期で求心力もなく、解散に追い込まれた

という状況の違いがある場合、AがdecideでBがdetermineという使い分けは正しいですか?

質問の仕方を変えると、

「Bにはdetermineを使えるが、Aにdetermineを使うのは違和感がある」

という私の感覚は正しいでしょうか?
0715名無しさん@英語勉強中 (ワッチョイW e791-bFZP)
垢版 |
2021/09/04(土) 10:21:00.13ID:r/6z8iGt0
ケンブリッジ英英辞典を引くと、determineは"to make a strong decision"とあるので、何か難しい決断とか、重要な決断について使うのだと考えられます
Bの場合は苦渋の選択という状況なので、まさにdetermineが相応しいと言えるでしょう
逆にAの場合は、難しくなく自然に行われる選択ですので、determineは相応しくないと言うことができそうです
0718名無しさん@英語勉強中 (ワッチョイ e79d-nTGN)
垢版 |
2021/09/04(土) 10:46:19.89ID:bRkBAnDU0
>>715
ありがとうございます。

たとえば、Bの他にも
C:子供の頃大手術で命が助かり、それがきっかけで将来医者になることを決めた
D:部屋のサイズによって定員が決まってくる

にもdetermineがつかわれるようですが、

Cの子供がそれをきっかけに大工を志すことはないですし
Dは物理的に決まっていることです

このように
「自由に選択できない」「宿命的に、条件的に、そうなることが決まっている」
ようなものにdetermineを使うのかなと、個人的には思っているのですが…
0719名無しさん@英語勉強中 (ワッチョイW 5fa7-zLw1)
垢版 |
2021/09/04(土) 10:46:22.89ID:9vmtZmdP0
>>710
そんな見せ本なんてあるんか。
ジーニアスが昔見せ本だったというのを聞いたわ
これはいい辞書です、と宣伝したり、ずっとジーニアスの宣伝をしてて
お金もろうたんかい、
としか思えへんことがあったらしい
0723名無しさん@英語勉強中 (ワッチョイW 5fa7-zLw1)
垢版 |
2021/09/04(土) 10:55:57.46ID:9vmtZmdP0
そや、大人用の文法書なんやろか、考える手がかりは、大人は何しとるかということや。
小説読んどるマイナーはんは置いといたら、大抵はトーイックやっとるな
そやからトーイック受験にいっちゃんええ文法書を探せばええんや。
金のフレーズいうて文法書やったっけ?
0724名無しさん@英語勉強中 (ワッチョイ 7f89-lYlz)
垢版 |
2021/09/04(土) 10:56:09.69ID:Jh8RYgr/0
>>718
decide よりも determine の方がはるかに使用頻度が低いので、自然と大げさな意味合い
の時に使うことが多くなる、というのは容易に想像できるわな。でも、decide が
けっこう深刻な場面でも使われることもあるわな。俺自身は、decide は少なくとも
あらゆる場面で使われ、determine だけはある特定の場面に限られるんだろうなと
思っている。(ただし、このことは質問者も回答者もすでに承知のことかもしれないけど。)

(1) When my son died of cancer I decided to wage war on the disease myself
https://www.independent.co.uk/voices/cancer-killed-my-son-waging-war-treatment-breakthroughs-disease-therapy-tobacco-industry-a8046686.html

(2) In 1914 and again in 1939, Germany decided to wage war on the rest of Europe.
https://www.lowellsun.com/2018/07/27/nato-should-stop-complaining-about-trump/

determine が自動詞として「決断する」という意味で使われているのを、俺自身はあまり見たことが
ないような気がする。ただし、科学やビジネスの分野で事務的にあるいは工学関係の
手続きの一環として determine を「他動詞」として「〜を決定する」とか「規定する」
というような意味で使う場面は、よく見るわな。
0725名無しさん@英語勉強中 (ワッチョイ e79d-nTGN)
垢版 |
2021/09/04(土) 10:57:27.96ID:bRkBAnDU0
>>720
すみませぬ・・・・

英文読解をしているのですが、
determineが関係代名詞的に修飾している語のニュアンスを知りたいのですが
そのためにdetermineのニュアンスを知りたいからなのです・・・・
0727名無しさん@英語勉強中 (ワッチョイ 7f89-lYlz)
垢版 |
2021/09/04(土) 11:00:39.88ID:Jh8RYgr/0
>>718
それから、determine を受け身として
I was determined to [do]
という形は、日常でもよく使うわな。
0729名無しさん@英語勉強中 (ワッチョイ e79d-nTGN)
垢版 |
2021/09/04(土) 11:02:14.10ID:bRkBAnDU0
>>724
詳しくありがとうございます。

やはりdetermineは、つまるところ「自由さがない」ときの「きめる」「きまる」のような気がしますね。
宿命的に決まったり、物理的に決まったり。

どうもお手数をかけてすみません。参考になりました!
0730名無しさん@英語勉強中 (ワッチョイ 7f89-lYlz)
垢版 |
2021/09/04(土) 11:02:30.42ID:Jh8RYgr/0
>>725
>>determineが関係代名詞的に修飾している語のニュアンスを知りたい

それなら、あなたが直面しているその英文を見せてほしいね。それを見せてくれたら、
回答者たちも回答しやすくなる。
0733名無しさん@英語勉強中 (ワッチョイ 7f89-lYlz)
垢版 |
2021/09/04(土) 11:06:19.66ID:Jh8RYgr/0
>>718
This value is determined by the balance of emissions from terrestrial biospheres (-25.7 ‰) and absorption (-26.0 ‰), emissions from the ocean (-9.5 ‰) and ...

理科系の分野とかビジネスや法学の分野で、何かが必然的に別のものによって
決まってしまう、というときにこの determine がよく使われるみたいだね。
人間が「決断する」という意味で(他動詞ではなくて)自動詞の determine するという
言い方をしている例を、俺自身は寡聞にしてあまり(あるいはまったく)
見聞きしたことがないような気がする。
0734名無しさん@英語勉強中 (ワッチョイ 475a-KEQA)
垢版 |
2021/09/04(土) 11:09:05.23ID:FGZknfCb0
>たとえば、数学の場合、大人が高校生向けの参考書を読むということはあまりありませんが、
>英文法の場合、なぜか大人でも皆高校生向けの英文法書を読みます。

>大学生やそれ以上向けの分かりやすい英文法書ってないんですか?

「わかりやすい」を求めるなら素直に「高校生向け英文法書」を読めばいいのでは?

「わかりやすい」から先はロイヤルか、その先も必要なら英語なんだから英語で書かれた文法書に行くのが筋。
0735名無しさん@英語勉強中 (ワッチョイ 7f89-lYlz)
垢版 |
2021/09/04(土) 11:10:54.43ID:Jh8RYgr/0
>>718
同じように、宿命が決まるときにもこの determine がよく使われるよね。

Your fate is determined by the choices you make.

上のセリフは、ネット上に盛んに出てくる。
0737名無しさん@英語勉強中 (ワッチョイ 7f89-lYlz)
垢版 |
2021/09/04(土) 11:30:25.39ID:Jh8RYgr/0
なるほど。determine の語源は de- + terminus であり、terminus は terminal や term と
語源を共有しているけど、determine はもともと「terminus (つまり limit や boundary) を決める」
という意味のようだな。

decide はもともと、 de + cidere (caedere = to cut) から来ており、「切る」という言葉から
派生したのだな。だからこそこの decide やそれに似たフランス語などを明治の日本人は
「決断」つまり「決めて断じる(= 切る)」という感じを当てたんだろうな。
0738名無しさん@英語勉強中 (ワッチョイ 7f89-lYlz)
垢版 |
2021/09/04(土) 11:32:34.09ID:Jh8RYgr/0
>>737
>>だからこそこの decide やそれに似たフランス語などを明治の日本人は
「決断」つまり「決めて断じる(= 切る)」という感じを当てたんだろうな。

訂正:
だからこそ、この decide やそれに似たフランス語 (decider) などの訳語として、明治の日本人は
「決断」つまり「決めて断じる(= 切る)」という漢字を当てたんだろうな。
0740名無しさん@英語勉強中 (ワッチョイ 5fe2-KEQA)
垢版 |
2021/09/04(土) 13:15:28.26ID:fZStMv+O0
determined
形《be 〜》決心[覚悟]している
・If you are that determined, I will help you. : そこまで決意が固いのなら、私も力を貸すよ。

determination
名 決意すること、決断に至ること 決断力、意志の強さ


decision
名 決断[決心・決定]したこと[事項]

decisive
形〔議論や競争などを決着させる〕決定的な
・The baseball team had a decisive victory in the tournament. : その野球チームはトーナメントで決定的な勝利を挙げた。
0741713 (ワッチョイ 5f33-/op0)
垢版 |
2021/09/04(土) 14:20:25.04ID:3B43IuqX0
みなさん、ご回答、ありがとうございました。

>>722
ロイヤル英文法は高校生向けの参考書だと思っていたのですが、違うのでしょうか?
0742名無しさん@英語勉強中 (アウアウウーT Sa8b-MUkt)
垢版 |
2021/09/04(土) 14:32:41.21ID:bw/UiIt0a
普通の読み書き会話なら受験英語の文法参考書でお釣りが来るくらい十分。

大学レベルから先の英語は英語「学」になって言語学の範疇、学校英語とがらっと変わる世界で興味深くはあるけど実用性には欠ける。
0744名無しさん@英語勉強中 (ワッチョイ 07e2-KEQA)
垢版 |
2021/09/04(土) 15:17:26.08ID:Oi6fkfM60
『ロイヤル英文法』

----

本書は,高校生以上の英語学習者ならびに教授者を対象にした,『ロイヤル英文法』の改訂新版である。本書の旧版は幸いに読者のご好評を得て,文法辞典の機能も備えた詳細な英文法書として,高校生から一般社会人までの幅広い読者層の支持を受け,10年以上の間愛読され続けてきた。
0749😏三年英太郎😉 ◆3CZBjOt3.Y (ワッチョイW 5f86-539L)
垢版 |
2021/09/05(日) 00:44:02.43ID:D5nKSTz/0
そうは言っても日本の英語レベルって最悪じゃん
それはやっぱり英語教育にも問題あったからでしょ🤔

これから先はそれで国際社会に渡り合って行けるの?🤔
内向的島国根性で移民政策も乗り遅れて衰退国家じゃん?🤔
0750名無しさん@英語勉強中 (アウアウキー Sa9b-th+e)
垢版 |
2021/09/05(日) 00:56:26.47ID:YzTJCHLta
ぶっちゃけ日本が英語できるようになる日は来ない
韓国みたいに受験戦争・超高学歴社会にすればできるようになるかもしれんけど副作用として出生率が激減して社会が崩壊する
0753名無しさん@英語勉強中 (ワッチョイ 475a-KEQA)
垢版 |
2021/09/05(日) 01:43:27.32ID:Ahi5sYeV0
>>752
テスト特化対策しまくりの替え玉カンニング他不正しまくりので少々点取ったところで実際に英語ができる証左になりはしない。それですらアジア上位にすら入いらない中途半端なレベルを目指す意味が不明。

TOEFL Test and Score Data Summary 2020
Singapore  98
India  96
Malaysia  94
Pakistan  94
Hong Kong  91
Philippines  90
Bangladesh  88
Indonesia  88
Viet Nam  88
China  87
Korea, Democratic People's Republic of  87
Macao  87
Korea, Republic of  86
Myanmar  86
Nepal  86
Sri Lanka  85
Taiwan  85
Mongolia  83
Thailand  83
Cambodia  77
Japan  73
0755名無しさん@英語勉強中 (ワッチョイ 475a-KEQA)
垢版 |
2021/09/05(日) 01:55:56.01ID:Ahi5sYeV0
>>754
しょっちゅう新聞で報道されてる風物詩。

・「韓国人の英語の実力に疑問」…驚愕の試験問題流出手口が捜査で明らかに(中央日報)
・業界トップのTOEIC塾、特殊カメラ使って問題流出(東亞日報)
・TOEIC問題漏えい、知的財産権を無視する韓国社会(朝鮮日報)
・無線イヤホンで集団カンニング、30人摘発(朝鮮日報)
・TOEIC成績表を偽造した斡旋業者代表を検挙
・TOEIC試験で大規模なカンニング発覚 ハイテク機器使った組織的犯行
・改編控えたTOEIC試験、携帯電話不正疑惑
・「TOEIC成績偽造」のコミュニティー登場、衝撃
・TOEIC成績表偽造の不正行為を摘発
・TOEIC試験紙、事前に流出

朝鮮日報「偽証罪は日本の165倍、誣告事件は305倍、詐欺事件は13.6倍」
0757名無しさん@英語勉強中 (アウアウウー Sa8b-fLcm)
垢版 |
2021/09/05(日) 02:06:04.16ID:aj6hDyX7a
For they have no conception how a rational creature can be compelled, but only advised, or exhorted;
because no person can disobey reason, without giving up his claim to be a rational creature.

この、"but only"は、どのような意味で、用いられているのでしょうか。
ジョナサン・スウィフトの名言より、引用しました。
0758名無しさん@英語勉強中 (ワッチョイ 475a-KEQA)
垢版 |
2021/09/05(日) 02:39:14.59ID:Ahi5sYeV0
>かつてイギリス領だったり英語が公用語だったり

朝鮮戦争以来のアメリカへの経済難民が今でも年間約2万人、アメリカ移民出身国第9位。
日本からアメリカに移民する者など今では極少数で日系アメリカ人人口も留学生数も下降の一途。

アホの極み。
0759名無しさん@英語勉強中 (アウアウキー Sa9b-th+e)
垢版 |
2021/09/05(日) 02:51:51.20ID:gnftM7pba
>>757
but only = just

>>758
アメリカ移民が韓国国内のtoeic平均になんか影響あんの?アホか
ネトウヨなんか遅くても高校生で卒業しろよ

韓国はエンタメをはじめとして海外志向が強い点も英語力強化に一役買ってるだろうな
日本は良くも悪くもガラパゴスすぎて国内にしか目を向けない。海外旅行も全然しないし
文化的な違いの影響は大きい
0760名無しさん@英語勉強中 (ワッチョイ 7f89-lYlz)
垢版 |
2021/09/05(日) 05:50:27.46ID:pAiywuCp0
>>757
他の人もすでに回答しているけど、補足しよう。

>>For they have no conception how a rational creature can be compelled, but only advised, or exhorted;

ここでの but only のうちの only はもちろん just という意味だよね。「〜だけ」とか「単に」
という意味だ。but は、その前の no をうけていて、いわば "not... but..." 構文の
変化球としての "no... but..." という形になっていて、「〜ではなくて、〜だ」という
意味合いになっている。

しかしそうだとすると、no と but が離れすぎていて、忠実に厳密に考えると
普通の no... but (not... but) の関係とは違っているような気がしてしまうよね。
そういう点を質問者は理解に敷くかったから、今回ここで質問しているのではないかと思う。

For they have no conception how a rational creature can be compelled, but only advised, or exhorted;

俺なりの和訳:
というのも、彼らは、理性的な生き物というものが(何かをするように)強制される
という考えはなく、単にそう推奨されたり強く促されたりするだけなのだと
考えているのだ。
0761名無しさん@英語勉強中 (ワッチョイ 7f89-lYlz)
垢版 |
2021/09/05(日) 05:50:49.33ID:pAiywuCp0
>>757 への回答の続き
no (または not) が conception にかからずに、そのあとの compelled にかかっていれば、
俺たちになじみ深い no (not) ... but ... という形になって、すぐにわかるわけだ。
でも今回のような形もあるんだな、ということがこれでわかるのだ。

今回の英文をパラフレーズして、not... but... を俺たちにとって分かりやすい形にしてみる。

(1) For, to them, rational creatures cannot be compelled, but only advised or exhorted.
(2) For they believe that rational creatures cannot be compelled, but only advised or exhorted.

ここでの to them は「彼らから見れば」とか「彼らにとっては」というような意味だ。
それはともかく、無理矢理に (1), (2) のように書き換えてはみたけど、スウィフトの
書いたように最初の方に no (または not) を持ってくるのが自然なのだから、
俺の書き換え文はあくまで参考程度に流してほしい。簡単な例を挙げると、
I think it's not good. よりも I don't think it's good. のように、not を前の方に持ってきた
方が自然なんだよね。
0762名無しさん@英語勉強中 (ワッチョイ 7f89-lYlz)
垢版 |
2021/09/05(日) 05:54:09.79ID:pAiywuCp0
>>760
間違い: そういう点を質問者は理解に敷くかったから、
訂正後: そういう点を質問者は ★理解しにく★ かったから、
0767名無しさん@英語勉強中 (ワッチョイW 5fa7-zLw1)
垢版 |
2021/09/05(日) 10:43:12.69ID:fmwdZyLE0
>>766
ググレカス

I should here observe to the reader, that a decree of the general assembly in this country is expressed by the word hnhloayn, which signifies an exhortation, as near as I can render it; for they have no conception how a rational creature can be compelled, but only advised, or exhorted; because no person can disobey reason, without giving up his claim to be a rational creature.
0768名無しさん@英語勉強中 (ワッチョイW 5fa7-zLw1)
垢版 |
2021/09/05(日) 10:57:49.85ID:fmwdZyLE0
というのは理性のある生き物が、アドバイスされたり勧められるだけではなくて、
どのように強制されたら強制された通りにするかを、彼らは考えていなかったからだ。
誰でも理性に従わないときには、自分自身が理性のある生き物だという主張を諦めることになる。
0771名無しさん@英語勉強中 (ワッチョイW 5fa7-zLw1)
垢版 |
2021/09/05(日) 11:04:03.12ID:fmwdZyLE0
というのも、合理的な生物がどのように強制されるのかということは考えられず、ただ助言されたり、励まされたりするだけだからです。なぜなら、人は合理的な生物であるという主張を放棄することなく、理性に背くことはできないからです。

DeepL訳
0773名無しさん@英語勉強中 (スッップ Sd7f-vxms)
垢版 |
2021/09/05(日) 12:12:13.64ID:2jnYrVrad
>>768
貼ってくれた前段を読むと、

法令では助言や推奨の表現が用いられていて、どうしたら「理性的動物」たる人間が強制され得るか(あるいは強制しても許されるのか)についての概念を持たない。

ということをスウィフトは皮肉混じりに言ってるように私は思ったけど全然違うかな
0778名無しさん@英語勉強中 (ワッチョイ 5fe2-lYlz)
垢版 |
2021/09/05(日) 14:08:36.55ID:Leq0o6Hj0
I think he's had meetings all day.

このセンテンスですが、 he's had の 部分が意味わかりません

he had ではなくて he's had となっています

そもそも's は is なんですか?

be +動詞+現在分詞  そんな構文があるのでしょうか?
0779名無しさん@英語勉強中 (ワッチョイ 5fe2-lYlz)
垢版 |
2021/09/05(日) 14:09:14.93ID:Leq0o6Hj0
この場合のmeetingは会議で名詞でした、スイマセン
0780名無しさん@英語勉強中 (ワッチョイ 5fe2-lYlz)
垢版 |
2021/09/05(日) 14:21:10.11ID:Leq0o6Hj0
短縮形を外せば
I think he has had meetings all day.

という事でよろしいでしょうか?
0782名無しさん@英語勉強中 (ワッチョイW 7f29-bcJK)
垢版 |
2021/09/05(日) 15:08:26.83ID:+Gnxgly+0
機械のシャッターを開いたまま動かそうとした時に

【警告】シャッターが開いています!

って表示したいんですけど英語だとなんて言ったらいいですか?
車の半ドアとか給油口開きっぱ警告みたいなニュアンスです
よろしくお願いします
0783名無しさん@英語勉強中 (ワッチョイ 2733-nzYM)
垢版 |
2021/09/05(日) 15:11:09.51ID:cON+ncnG0
脚のcalfの部分がバイクのマフラーに当たって(触れて)、火傷をしてしまった と言いたいです

I accidentally hit the motorcycles's muffler in the calf by accident and I burnt.
touchは手の部分かと思って使いませんでした。修正お願いします。
0784名無しさん@英語勉強中 (ワッチョイ 7f89-lYlz)
垢版 |
2021/09/05(日) 15:18:28.61ID:pAiywuCp0
>>782
【警告】シャッターが開いています!

文字通りに訳すと
WARNING: Sutter open

しかし実際には、シャッターが開いているからどうしたんだよ、と言いたくなるのが
英語ネイティブの発想であり、具体的に何をせよと言いたいのかはっきりしろ
と彼らは思うらしい。だから次のように書く方が賢明だ。

Close the shutter first.

あるいは、

WARNING: Shutter open. Close it first.
0785名無しさん@英語勉強中 (ワッチョイ 7f89-lYlz)
垢版 |
2021/09/05(日) 15:29:32.57ID:pAiywuCp0
>>783
>>脚のcalfの部分がバイクのマフラーに当たって(触れて)、火傷をしてしまった
>>I accidentally hit the motorcycles's muffler in the calf by accident and I burnt.

(1) motorcycles's は変だ。motorcycle's なら、一応は正しい。しかし motorcycle の muffler
なら 's をつけないで "motorcycle muffler" だけで十分だ。

(2) accidentally と by accident とが重なってしまっている。

(3) I burnt と言えば、焼身自殺のときみたいに自分が炎を上げて燃え上っているみたい
なイメージを与えてしまうのではないかと俺は不安なので、I got burnt (burned). を
使っておきたい。

(4) 手であろうが脚であろうが hit でも touched でもどちらでもいいと俺は
思っているが、口語では hit をよく使うよね。

日本語原文:
脚のcalfの部分がバイクのマフラーに当たって(触れて)、火傷をしてしまった

俺なりの英訳:
My calf hit my motorbike muffler by accident and got burned.
0786名無しさん@英語勉強中 (ワッチョイ 2733-nzYM)
垢版 |
2021/09/05(日) 16:03:42.58ID:cON+ncnG0
>>785
修正ありがとうございます
ミス多すぎたので次回からもっと確認します(accident被り. motorcycles's).
・・・のでは殆ど'sをつける癖があるので、つけないほうが自然になりそうな場合はつけないように気をつけます
0787名無しさん@英語勉強中 (ワッチョイ df08-1CxL)
垢版 |
2021/09/05(日) 16:25:47.34ID:nfMlh+qJ0
関係者のみんなへ

いつも山田くんのすてきな写真を撮影してくれてありがとうございます

っていうとき
いつも」はどこに入れたらいいですか?

To all the staff

Thank you so much for taking such wonderful pictures and videos o fMr.Yamada always.

でいいんでしょうか?
0788名無しさん@英語勉強中 (ワッチョイ 7f89-lYlz)
垢版 |
2021/09/05(日) 16:31:35.26ID:pAiywuCp0
>>787
>>To all the staff
>>Thank you so much for taking such wonderful pictures and videos o fMr.Yamada always.

「山田くん」の first name がたとえば Tsutomu なんだったら、それを使った方が
いいんじゃないかと思う。

To whom it may concern:
Thank you so much, as always, for producing such wonderful photos and videos of Tsutomu
0789名無しさん@英語勉強中 (アウアウウー Sa8b-fLcm)
垢版 |
2021/09/05(日) 17:06:31.38ID:mu7GGqbAa
757です。皆さんご解答して下さりありがとうございます。

私は、...but they think it can only be advised or exhorted.ということだと思いましたが、果たして文法的にこのように補うことは可能なのかと疑問に持ち質問致しました。
他の可能性として、He knows a lot of things, but only superficially.の様な用法であるとか、
compelled, advised, exhortedの三つをor繋いでいるのではあるまいか、とか、ということも考えましたが、あまり合点が行きませんでした。

>>760さんの解釈が最も合理的だと思い私もそれに倣おうと思います。感謝に堪えません。
0791名無しさん@英語勉強中 (ワッチョイ 873b-Sfec)
垢版 |
2021/09/05(日) 18:24:01.86ID:AJQCcEH70
>>787
To all concerned, thank you so much for always taking such wonderful pictures and recording such nice videos of Mr. Yamada.

>>783
I got a burn on my calf since I accidentally touched the muffler of my motorcycle.

I accidentally burned my calf with the muffler of my motorbike.

私も作ってみました。100点ではないと思いますが参考にしていただければ幸いです。
0792名無しさん@英語勉強中 (ワッチョイ 8789-S2fQ)
垢版 |
2021/09/05(日) 18:28:06.79ID:9WWMZwaX0
isとareの使い分けってisを短縮してsにすると複数形と区別つかないから
areにするんじゃないの?
例えばyous とyou isが簡単にわかるようにyou areと言う
学校でもこうやって教えてくれれば間違えなかったのに
誰も教えてくれないあkら自分で発見したんだけど
0795名無しさん@英語勉強中 (オッペケ Srbb-k2kw)
垢版 |
2021/09/05(日) 18:52:20.23ID:ggjUIoQ0r
ちなみにareがyouに後続する理由は元々youが専ら複数を表していたからだよ
二人称単数ではthouが用いられてた
まぁ本人の納得のためであればどんな理屈付けも肯定されるだろうけど先生がそれを言ったら流石にね…
0796名無しさん@英語勉強中 (ワッチョイ 8789-S2fQ)
垢版 |
2021/09/05(日) 19:04:58.74ID:9WWMZwaX0
>>791
yous isを短縮するとyous'sになってユーズズと言って喋りにくいから
yous areというようになったということ
俺は発音主体の学習法を自分で考案して実践している
0798名無しさん@英語勉強中 (ワッチョイW 5f5a-+qJX)
垢版 |
2021/09/05(日) 20:41:48.33ID:zGVwdRbC0
>>759
これ以上はスレ違いだけど
韓国人は日本人より多少は英語ができるのは事実だけど、就職率も日本より低くて自殺率は日本より高いし、英語ができても別に幸せかどうかとは一致しないよね
0801名無しさん@英語勉強中 (ワッチョイ 5f9d-lYlz)
垢版 |
2021/09/05(日) 22:17:16.33ID:gaT9hoZe0
最終更新日時:2021/09/05 22:03:00
本日の総投稿数: 213 順位: 440/1,029

日曜日で213投稿じゃもうEnglish板も終わったな
このスレだけで52投稿は工作のしすぎ
0804名無しさん@英語勉強中 (ワッチョイW 5fa7-zLw1)
垢版 |
2021/09/05(日) 22:27:38.08ID:LxRU8vWn0
>>798
だから竹中とかが韓国人が日本で働きやすいようにしようと考えているん
だろうな。高市は竹中と親しいから実行は高市が総理大臣になったら
そういうこともあるかもしれないのか。
韓国は働き場所が財閥企業しかなくて、つまり働き場所が少ないと
いうことですね
0805名無しさん@英語勉強中 (ワッチョイW 5fa7-zLw1)
垢版 |
2021/09/05(日) 22:30:53.91ID:LxRU8vWn0
>>750
なんであんなに韓国は極端なんだろう。
大学進学率が高いのはたぶん9割の大学進学率のオーストラリアより
韓国は進学率が低いと思うが、韓国は進学して不幸になってるんじゃないのかな。
オーストラリアは大学進学率がめちゃくちゃ高いが幸せに国民が暮らしていそうだ。
0808名無しさん@英語勉強中 (ワッチョイ 5fa7-MUkt)
垢版 |
2021/09/06(月) 12:03:14.12ID:RNscelm60
>>806
人種差別というけど、白人さんには対自国民以上の優遇を一般人は与えて、
対アジア人とか対発展途上国人で、恐る恐る接している
日本人って差別しないよね。日本語で話ができるかできないかで、日本人と
外国人を区別しているみたいな感じで。
0809名無しさん@英語勉強中 (ワッチョイ 67e3-nTGN)
垢版 |
2021/09/06(月) 13:39:00.48ID:oEhpNxgk0
>>808
「区別しているだけ」って言い方が既に世界的には差別とみなされる」ってのを自分からやってしまって「日本人は人種差別が酷いを」実践してしまっているのは素晴らしいというほかない。
0812名無しさん@英語勉強中 (ワッチョイ 8789-S2fQ)
垢版 |
2021/09/06(月) 18:48:53.16ID:lyw4OARd0
日本人で色黒なひとが黒人って言われても気にしないから人種差別はないよね
逆に黒人と言われて差別と感じたら、自分は黒人は劣っているとみなしている
から言われて嫌な気分になるんだよな
同様に馬鹿と言われて嫌な気分になる人は馬鹿が人より劣っているとみなしている人だから
馬鹿を差別してるってことだよ
人種差別はないけど馬鹿差別はあるよな
0813名無しさん@英語勉強中 (ワッチョイW 5fa7-zLw1)
垢版 |
2021/09/06(月) 21:12:19.96ID:hT03utab0
>>811
保証人でしょう。それは日本人でも大変
内向きと言うのは外国から帰ってきた時、日本語しか街の人たちが話してないから思うことだよね。
でも外国行ったら英語しか話してな場合もあるので同じだと思う。
0814名無しさん@英語勉強中 (ワッチョイW 5fa7-zLw1)
垢版 |
2021/09/06(月) 21:17:27.83ID:hT03utab0
>>812
そこまで博愛でなくて良いと思うよ。
例えばもろに歴史的な身分制に関わる差別用語をあげて、自分はこれこれの身分になりたくないと話したとする。
特定のカテゴリーを嫌ったわけで、それは非難されるものではない。
問題は、特定の人たちや人を、その歴史的な否定的カテゴリーと結び付けて、だからその人たちはダメなんだとか嫌なんだと言うこと
は差別になるよね。
0818名無しさん@英語勉強中 (アウアウウー Sa8b-fLcm)
垢版 |
2021/09/06(月) 22:27:46.60ID:Sz71Rntya
Satire is a sort of glass wherein beholders do generally discover everybody's face but their own;
which is the chief reason for that kind reception it meets with in the world, and that so very few are offended with it.

すみません、もう一つだけスウィフトから質問させてください。
終盤にあります"and that"は、どのような働きでしょうか。
for that kind ~ in the worldとthat以下を繋いでいるように感じますが、文法的役割の異なるものをandで結ぶことは可能なのでしょうか。
もしくは、reasonを修飾するという点では、機能が同一である為、andで結ぶことが出来るのでしょうか。
"Jonathan Swift quotes"と画像検索をかけていただければ、直ぐに引用元の画像は見つかると思います。
0824名無しさん@英語勉強中 (JP 0Hff-rLLl)
垢版 |
2021/09/07(火) 01:12:46.05ID:PIZ7nfQGH
物理攻撃型の人種差別するような輩は
話が全く通じないから反論とかすると危険
単独の抗議が成り立つのは陰湿型の人種差別の場合だけ
0827名無しさん@英語勉強中 (ワッチョイ 7f89-lYlz)
垢版 |
2021/09/07(火) 05:38:28.58ID:P6+ZsFy20
>>818
>>which is the chief reason for that kind reception it meets with in the world,
and that so very few are offended with it.

前後の関係から見て、これは
which is the chief reason for that kind reception it meets with in the world,
and ★the chief reason★ that so very few are offended with it.

という意味なんだというふうにしか解釈できないけど、Swift のこういう英文には
俺自身は出会ったことがない。Johnathan Swift は 1667 年生まれなので、かなり古い
英文だね。その時代にはそういう英文が普通だったんだと考えるしかないんじゃないかな?

それよりさらに 100 年くらい古くなると Shakespeare になるけど、Shakespeare は
韻文を中心とする特殊な文体や語彙を駆使したとはいえ、それでも文法が現代とは
かなり違っている。John Locke も 1632 年生まれで Swift と同時代人だけど、
Locke の文章に現れる文法も現代とかなり違って面食らう。(Locke の文章を
詳しく Twitter 上の Mr. BIG つまり北村一真氏が盛んに解説していた時があったけど、
現代語とはかなり違うということを思い知らされた。)

Shakespeare の時代の英文の文法について体系的に説明した本を俺は持っているけど、
通読したわけでもないしマスターしたわけでもないから、まだまだ俺にとって難しい。

A Shakespearian Grammar An attempt to Illustrate Some of the Differences Between Elizabethan and Modern English
E.A. Abbott

この本のどこかにそれについて書いてあるとは思うけど、今はそれを探す情熱がない。
0828名無しさん@英語勉強中 (ワッチョイ 7f89-lYlz)
垢版 |
2021/09/07(火) 05:52:17.90ID:P6+ZsFy20
>>818
こういう場合の that の古い用法については、その "A Shakespearian Grammar" という
本の 284 番と 285 番あたりでもしかしたら触れているかもしれない。

以前にも俺はこの部分を読んだことがあって、再び読み返してはみたものの、
はっきりと今回の場合にも適合するような解説が見つけ出せなかった。
質問者は興味があればその部分を読んでみたらいいと思う。2ページくらいにわたって
詳しく書いてある。
0829名無しさん@英語勉強中 (ワッチョイ 7f89-lYlz)
垢版 |
2021/09/07(火) 05:56:29.88ID:P6+ZsFy20
>>818
ともかく、Johnathan Swift みたいな 17 世紀とか、あるいはそれよりも古い時代の
文献を読むときには、俺がさっき触れたその本 "A Shakespearian Grammar" あるいは
それに似た本を本棚に置いておき、たまに開いてみたらいいと思う。
0832名無しさん@英語勉強中 (ワッチョイ 2731-zDG2)
垢版 |
2021/09/07(火) 10:45:24.21ID:2/PoTfHP0
>>818
それは、慣用表現としての「and that 〜 (しかも〜)」で説明できるかも
しれない。 辞書で and that を引いて確認してみてください。

and that 〜 は「前に出た節、文の意味を強めるため、それを繰り返す
代わりに用いる」。

例文: I accepted it, and that with thanks. (私はそれを承諾したーー
しかも、ありがたく承諾した)
= I accepted it, and I accepted it with thanks.

問題の文章を、and that を使わずに書き換えてみると(and that の that
が示す「前に出た節または文」に当たるのは which is the chief reason と
考える)、以下のようになります。

〜 which is the chief reason for that kind reception it meets with
in the world, and which is the chief reason so very few are offended
with it.
(そして、それが、世間で「風刺」があのような寛容な受け取り方をされる
主な理由であり、しかも、「風刺」によって気分を害される人がほとんど
いない主な理由である)→(そして、それが、世間で「風刺」があのような
寛容な受け取り方をされ、しかもまた、それで気分を害される人がほとんど
いない主な理由である)

(〜 reason と so 〜 の間には関係副詞の why が省略されていると考え
ます)
0833名無しさん@英語勉強中 (アウアウキー Sa9b-th+e)
垢版 |
2021/09/07(火) 19:56:10.40ID:dk/6M+XQa
和文英訳は英語学習者にとって有害だと思うんだけど、どうだろう

同じことを書こうとしても日本語と英語では発想や論理構造が異なることが多い
これは和訳についても同様で、英文の中に詰め込まれた要素の組み合わせが日本語では不自然に感じられることが多い
日本語にとって必要な表現が欠落していたり、逆に余分な表現があって訳すと冗長になったりとか、よくあるでしょ
その辺りをうまく意訳しても、日本人が一から日本語で書いた文とはどこか異なる佇まいの文になってしまいがち
で、日本人は自然な日本語を熟知しているからそういった文を「翻訳調」などとして相対化できるので問題ない
だが英訳となると、自然な英語というのをあまりよく分かっていない日本人はネイティブにとっては「翻訳調」の英文を書き慣れてしまうことに繋がりかねない

というか実際もうそうなっているんじゃないか?日本人の書く英文ってどれも似たり寄ったりで、よほどの達人でもない限り、ああ日本人だなとわかるような書き方をしてくるし
0838名無しさん@英語勉強中 (ワッチョイ 07e2-MUkt)
垢版 |
2021/09/07(火) 21:11:27.03ID:cii3QcVv0
>和文英訳は英語学習者にとって有害だと思うんだけど

まぁ問われる能力が違うね。

書く内容を考える段階から英語で考え、具体的に文章化する際もどこかで見た英語の慣用表現などを用いるようになれば、より自然な英文になってくる。
0841名無しさん@英語勉強中 (アウアウウー Sa8b-fLcm)
垢版 |
2021/09/07(火) 21:41:39.23ID:bPjgijUxa
>>832
ありがとうございます。
前文を受けるand thatの可能性は私も考えました。

しかし、辞書に掲載されている例文を見ると、多くても3語程度の副詞(句)を付加するに過ぎないものしかなく、この場合にも適用しうるのか慎重にならなければならないと思いました。

また、前文または節を受けるのであれば、which is the〜in the worldまでを繰り返すという可能性もあるのではないか、とも考えました。

>>830
ありがとうございます。
基本的に前置詞の後ろにthatを取ることは、in thatやexcept thatなどの少数の例外を除けば、出来なかったはずです。
もっとも、この当時の英語においては、接続詞の後にthatを付けることが、ままあった様なので、「〜というのも」という意味のforにthatを付けることは可能であったのかもしれません。

もしも、確実にreasonの後ろに、 for that節をとる用例を知っていらっしゃれば、ご教示して頂きたく思います。

また、kind receptionが倒置で前に出ているという件ですが、その部分が修飾語句で非常に長くなっている場合など、相応の理由がなければ倒置にはならないと思います。従って、私は、ここは単に先行詞であると考えます。
0844名無しさん@英語勉強中 (アウアウウー Sa8b-fLcm)
垢版 |
2021/09/07(火) 22:02:23.85ID:O/r64S9za
>>827
ありがとうございます。
北村一真氏の、John Lockeの英文を読むために必要な英文法、というツイートを発見致しました。熟読したいと思います。

A Shakespearian Grammarについては、著作権が切れているせいか、ネット上にpdfが上がっていたので、参照しましたが、スウィフトですらこの有様なので、シェークスピアや欽定訳聖書等からふんだんに例文が引かれているこの本を使いこなすのは私の今の力では無理だと悟りました。
大塚高信氏の『シェイクスピアの文法』という本を検索して見つけたので、まずは日本語で挑戦してみようと思いました。

私が質問した箇所については、
…hearing such a question addressed to me from a man on the brink of destruction and to whom I should have supposed...
という用例が過去スレに挙げられていたのを思い出したので、
修飾するという役割が一緒であれば、文法的に性質が異なるものをandで繋いでも良い、と差し当たり納得しておくことにします。

皆さん貴重な意見を聞かせて下さってありがとうございました。
0846名無しさん@英語勉強中 (ワッチョイ c717-8qru)
垢版 |
2021/09/07(火) 22:28:45.76ID:d+V/ki2v0
>>839
どちらもありうるけど場面によって使い分けるだろうね。
例えば1時間前に明日やろうと思ったんだったらwouldがよさそうだけど、
例えば昨日の時点で2日後(今から言うと明日)にやろうと思ったんだったら
下の文(wouldの後にtomorrowが続く)は違和感を感じる。
思った時から言ったらtomorrowではないので。
tomorrowと続くならwillだな。言っている人が現在を意識している。
0847832 (ワッチョイ 2731-zDG2)
垢版 |
2021/09/07(火) 22:33:33.89ID:2/PoTfHP0
>>841
>しかし、辞書に掲載されている例文を見ると、多くても3語程度の副詞(句)を
>付加するに過ぎないものしかなく、この場合にも適用しうるのか慎重にならなけ
>ればならないと思いました。

そう、確かに辞書には3語程度の副詞句を付加するに過ぎないものばかりですね。

しかし、自分が参照した文法書(『英文法精解』木村明 著)には結構長い例が
載っていた。
参考までに書き写しておきます。

He suddenly burst out laughing, and that so heartily that, after some
hesitation, we were all obliged to join him.
(彼は突然笑い出し、しかも腹の底から笑い出したので、私たちみんなも、少し
ためらっていたが、しまいには彼といっしょになって笑わないではいられなくなった)
0848名無しさん@英語勉強中 (ワッチョイ df5f-/op0)
垢版 |
2021/09/07(火) 22:58:54.10ID:6NAsIOlV0
who knows?には、「知るか(誰も興味ない、自分には関係ない)」というニュアンスはありますか?
例えば、突然自分語りを始めた人に対して「知らんがな」と一蹴するようなニュアンスです
0850名無しさん@英語勉強中 (アウアウウー Sa8b-zpzk)
垢版 |
2021/09/07(火) 23:29:15.45ID:II0EgmA7a
>>843,846
ありがとうございます
どちらでもオーケーで場合によるのですね
私は時制の一致とやらでwould にしないといけないかと思ったのですが違和感がありました
場面としては、昨日に「明日やろうと思った」ことを今日書いています
やるべきことをしなかった言い訳をしてるイメージです
0854名無しさん@英語勉強中 (ワッチョイ 873b-Sfec)
垢版 |
2021/09/08(水) 02:20:49.17ID:XDVl/ANr0
>>848
who cares? とかno one asked.あたりがいいんじゃないでしょうか
細かいニュアンスはわかりませんがなんとなくキツい表現な気がするので、good for youぐらいにとどめておくのが無難かもしれませんね(これもちょっと皮肉ですが・・・)

who knows?は「さあね」とか「そんなの知るか(誰もわからないという意味で)」とか、あるいは「でもどうなるかわからないよ?」という感じで使う気がします。
0855名無しさん@英語勉強中 (ワッチョイW c7e3-wvkC)
垢版 |
2021/09/08(水) 03:35:17.90ID:E5qUUMOF0
ハロートークのラフェイとサムという男には注意。あいつら日本人女性とやりたいことしか考えてない低脳。サムは複数アカウント持ってて毎回電話相手探している。ラフェイはナルシストで日本人見下し
0856名無しさん@英語勉強中 (ワッチョイ 7f89-lYlz)
垢版 |
2021/09/08(水) 05:04:39.99ID:z6vRXxV/0
>>852
I thought I'd do it today. でいいんじゃないかと俺は思うけど。

参考:
"★I thought I'd do it today★ because on Monday we've got skinfold tests, so it saves (strength and conditioning coach) Matty Hass bringing the barbecue tongs - so I thought I'd knock it out of the way now."
https://www.afl.com.au/news/444439/mcgrath-to-retire-as-lions-golden-chapter-ends
(スポーツ関係のネット記事より)
0858名無しさん@英語勉強中 (ワッチョイ 7f89-lYlz)
垢版 |
2021/09/08(水) 09:08:03.13ID:z6vRXxV/0
>>856 のような英文でもいいけど、
I thought I was going to do it today.
でもいいみたいだな。少なくとも
https://www.google.com/search?q=%22i+thought+i+was+going+to+do+it+today%22&;rlz=1C1FQRR_enJP949JP949&oq=%22i+thought+i+was+going+to+do+it+today%22&aqs=chrome..69i57j33i160l2.12948j0j7&sourceid=chrome&ie=UTF-8
このリンク先を見ると、二つの小説の中で使われている。
0859名無しさん@英語勉強中 (ワッチョイW df5f-Z9o8)
垢版 |
2021/09/08(水) 09:14:17.89ID:X5d3wxD40
>>854
ありがとうございます!
0860名無しさん@英語勉強中 (ワッチョイ 7f89-lYlz)
垢版 |
2021/09/08(水) 09:36:30.38ID:z6vRXxV/0
"I was going to do it today." というセリフを、Donald Trump が使っているな。
「今日やるつもりだったけど、いろいろと状況が変わって、(結局は〜となった)」
という文脈。

I was all set to do it. In fact, I was going to do it today. I was going to do as we're sitting here. I would've had to delay you. I was going to do it today. I was going to terminate NAFTA. But they called up and they said, "Would you negotiate?" And I said, "Yes, I will negotiate."
https://www.cbsnews.com/news/trump-interview-full-transcript-face-the-nation/
0861名無しさん@英語勉強中 (ワッチョイ c717-8qru)
垢版 |
2021/09/08(水) 09:50:50.67ID:gAZwFazw0
>>858
I thought I was going to do it today.
だったら今日やるつもりだったんだけど、みたいなニュアンス。
なので多分今日やらずに他の日になる。
I thought I would do it today.
だったら単に今日やろうかなと思っただけで気が変わらなければ今日やる。
0862名無しさん@英語勉強中 (ワッチョイ 7f89-lYlz)
垢版 |
2021/09/08(水) 10:53:29.87ID:z6vRXxV/0
>>861
なるほど。最初そのコメントを読んだ時「ほんまかいな」と疑ったけど、それぞれの
表現の用例を見て、その前後を読み取ってみると、確かにあなたの言う通りらしい
と感じられてきた。
0863名無しさん@英語勉強中 (ワッチョイ 7f89-lYlz)
垢版 |
2021/09/08(水) 11:28:00.65ID:z6vRXxV/0
>>861 が正しいことを検証する試み

(1) >>860 のリンク先では、Trump が「今日やるつもりだったんだ。でも、いろいろと
今は状況が変わって、私が予期していたものとは違っている」というようなことを
言っている。だからたぶん、今日はやらないんだろうと思う。

(2) Max Wirestone · 2017 · ‎Fiction
“Well, I thought I was going to do it today—because you know, scenic beautiful steamboat and everything. That's why I wanted to advance to day two—but then ...

この (2) の小説の一節についても、話者の男はは「今日やろうと思ったんだけど、相手の
女がこれこれこういう衣装を着ているから、俺は彼女の恰好とはまるでちぐはぐになって
しまった」というようなことを言っていて、どうも今日はやれそうもないということを
ほのめかしている。

その一方で、(実際に印刷された本の例文でないのは残念だが)
https://twitter.com/sethabramson/status/1433110125645078530
ここに出てくる I thought I'd do it today. では、>>861 が言ってくれたように、
「今日やろうと思った」と軽く言っていて、実際に今日それをやっている。

なぜこの二つにこういう違いが生まれるのか、と俺なりに考えてみたけど、
もしかして
(1) I am going to do
の方が
(2) I will do
よりも 「必ずやろう」という意思が強く感じられ、それゆえに I thought をつけると
「そういう強い意志を持って "やろう" と思ってたんだけどね」という意味合いになり、
したがって「実際には今日はやらないことになったんだ」という意味合いが生じることになる。

しかし I will do it は軽いので、I thought I'd do it today. と言っても、大した強い意思を
感じさせず、単に軽く「今日やろうと思ったんだよ」と言ってるだけで、
状況は大して変わってないので、少し前に考えたように、実際に今やってるんだよ、
という意味合いが生じるのではないかと俺の悪い頭で考えてみたのであった。
https://twitter.com/5chan_nel (5ch newer account)
0865名無しさん@英語勉強中 (ワッチョイW 5f5a-+qJX)
垢版 |
2021/09/08(水) 11:31:11.74ID:uouKpCrd0
「パソコンのデータを、壊れる前の状態に戻す」って英語でなんて言うんでしょうか?
Restore the data before the computer gets broken
だと、パソコンが”壊れる前に”データを復元する となってしまいますよね?
0867名無しさん@英語勉強中 (アウアウウーT Sa8b-mUqo)
垢版 |
2021/09/08(水) 11:34:28.51ID:jzicDtQPa
restore it to the state when it wasn't broken
0868名無しさん@英語勉強中 (ワッチョイ c717-8qru)
垢版 |
2021/09/08(水) 12:01:42.31ID:gAZwFazw0
>>863
いろいろと分析ありがとうございます。
be going toの方はもともとやろうと予定していたこと、
willはその場でやろうと思いついたこと、ということになります。
なので日本語訳でむりやりニュアンスを合わせると下記のようになると思います。
be going toの方は
今日しようと思っていた。
wouldの方は
今日しようと思った。
みたいな感じでしょうかね。
0869名無しさん@英語勉強中 (ワッチョイ 7f89-lYlz)
垢版 |
2021/09/08(水) 12:02:38.01ID:z6vRXxV/0
>>865
パソコンのデータを、壊れる前の状態に戻す

>>867 による良質の回答を受けた上での後出しになってしまうけど

(1) restore the data from the PC to the state before it got broken
(2) restore the data from the PC to its original unbroken state
(3) restore the data from the PC to its undestroyed state
(4) unbreak the data from the PC
0871名無しさん@英語勉強中 (ワッチョイ c736-nzYM)
垢版 |
2021/09/08(水) 14:07:14.49ID:yWPohwoG0
A pig is no less sensitive a creature than a person.
豚は人間に劣らず傷つきやすい動物である。

「表現のための実戦ロイヤル英文法」の例文ですが、no less の後は
no less a sensitive creatureとしてはいけないのでしょうか?
0873名無しさん@英語勉強中 (ワッチョイW e791-bFZP)
垢版 |
2021/09/08(水) 14:47:24.00ID:OUIeldTI0
no less a sensitive creatureはだめです
no less sensitive a creatureが正しい
同じ黄色ロイヤルにある、
Arizona is such a nice place that…(略)
→Arizona is so nice a place that…
の書き換えも同じ理屈です
lessとsoはどっちも副詞だから、名詞にはかからない
0874名無しさん@英語勉強中 (アウアウウーT Sa8b-mUqo)
垢版 |
2021/09/08(水) 15:26:44.43ID:MxxX05Sya
>>871
形容詞まで含んで名詞句として扱えるかどうかによるのでは。

The soldier was no less a social animal in war than he had been in civilian life.
A whale is no less a mammal than a horse is.

「sensitive creature」を「傷つきやすい動物」じゃなくて「敏感生物」と捉えるならありかも知れないけど、まぁ変な感じではある。
0876名無しさん@英語勉強中 (ワッチョイ e708-1CxL)
垢版 |
2021/09/08(水) 21:52:37.66ID:llcqz6tB0
What a commanding appearance he gives! を書き直すと

★ How commanding appearance he gives !じゃなくて
How commanding his appearance is!になるのでようか?

★でも正しい?
0877名無しさん@英語勉強中 (ワッチョイ c736-nzYM)
垢版 |
2021/09/09(木) 01:18:43.85ID:zlDPxVmg0
The last week saw more attacks. 「先週には、さらなる攻撃がありました。」

lastの前にtheが付くと、「ここ〜」の意味になるかと思うのですが、そうすると
ここの訳は「ここ1週間は、さらなる攻撃がありました。」になるのではないでしょうか?
0878名無しさん@英語勉強中 (ワッチョイ 873b-Sfec)
垢版 |
2021/09/09(木) 02:08:59.64ID:GU3c2kXZ0
>>877
ウィズダムによれば、「無冠詞のlast week [month]が「先週[月]」であるのに対し,定冠詞を伴うと「今日までの一週間[一か月]」の意味になる」とありますので、
ご提示の通り「ここ一週間」で正解だと思います

the next dayといえば「その次の日」ですが、「その前日」は、the last dayとはせずにthe previous dayとするそうです。that dayは「その当日」となります。
だから、ここではthe last weekを「その前の週」という風にはとれないみたいですね(もちろん「先週」もダメです)
0879名無しさん@英語勉強中 (ワッチョイ 7f89-lYlz)
垢版 |
2021/09/09(木) 03:32:08.05ID:ypbdvGur0
>>876
How commanding an appearance he gives!
0880名無しさん@英語勉強中 (ワッチョイW 5fa7-zLw1)
垢版 |
2021/09/09(木) 05:16:29.01ID:Az+3Z8WZ0
>>871はa less sensitive animal than a manでは豚はない
という表現がno less sensitive an animal than a man

であって、
a pig is no less a sensitive animal than a man
だと
a pig is a sensitive animalとa man is a sensitive animal.
を比較してる。
0885名無しさん@英語勉強中 (キュッキュ bf33-/op0)
垢版 |
2021/09/09(木) 09:04:17.71ID:fiSYO9vR00909
>>871

比較級は、程度を問う場合(連続体、スペクトラム、アナログ)と、
真偽を問う場合(是か非か、0か1か、デジタル)があります。

クジラ構文や874の例などは、形は比較級でも、実際の内容は「真偽」を問うものです
・クジラは、ウマと同様に哺乳類か、ウマとは異なり非哺乳類か
・戦場の兵士は、社会的動物か、非社会的動物か

これに対し、871の例文は、「程度」を問うものです
・(○)豚は、人間と比べて傷つきやすい生き物か、傷つきやすくない生き物か
・(×)豚は、人間と同様に敏感生物か、人間とは異なり非敏感生物か
0886名無しさん@英語勉強中 (キュッキュ MM4f-nTGN)
垢版 |
2021/09/09(木) 10:04:57.94ID:enCBxxVqM0909
同僚と英会話の練習していて、複数形や三単現の動詞を正しく言えてないと指摘されます

例えば2〜3分ほど喋ると
Many employee ..
Our company have ...
とかが3つぐらい指摘されます。

こういうのに気を付けながら喋るとそれに気を取られて今度はそもそも自分が言いたかったことが
なんだったかが分からなくなって余計にグダグダになってしまう。。

皆さんはこの辺をどうやって乗り越えましたでしょうか?

※昔やった瞬間英作文のこの辺の章をまた徹底的にやり直す?
0888名無しさん@英語勉強中 (キュッキュ 5fe2-KEQA)
垢版 |
2021/09/09(木) 12:57:17.65ID:IymgfMeo00909
While civil cases permit more discovery of facts and opinions prior to the trial,.. their process is NO LESS adversarial and NO LESS a threat to the therapist-patient dyad.

Christ, who was called the prosopon of God with NO LESS assurance (if with less frequency) than He was called God's Word or Wisdom.

To undertake to ensure that the standards of travel are maintained to NO LESS a standard than that which they inherit.
0896名無しさん@英語勉強中 (キュッキュ 5fe2-KEQA)
垢版 |
2021/09/09(木) 13:59:38.96ID:IymgfMeo00909
>>886
>こういうのに気を付けながら喋るとそれに気を取られて
>今度はそもそも自分が言いたかったことが
>なんだったかが分からなくなって余計にグダグダになってしまう。
>…どうやって乗り越えましたでしょうか

頑張るだけ。非ネイティブなら尚のこと意識して発話するよう努める。

>昔やった瞬間英作文

不要。
0897名無しさん@英語勉強中 (キュッキュ 6729-3cBl)
垢版 |
2021/09/09(木) 15:37:56.18ID:Wun3B1aC00909
How many about yes
でしょ文脈的にも
0899名無しさん@英語勉強中 (キュッキュT Srbb-nTGN)
垢版 |
2021/09/09(木) 19:13:34.52ID:fD0OOoC4r0909
886です。

ありがとうございます。

中学生レベルの例文を気を付けながら音読することと、
同僚との練習でも多少グダグダになってもいいから、まずは
複数形や三単現を気を付けながらやっています。
0900名無しさん@英語勉強中 (キュッキュ MM8f-MKt8)
垢版 |
2021/09/09(木) 19:20:44.57ID:27xvUaQOM0909
音読じゃなくて自分で作る練習しないと意味ないよ
0903名無しさん@英語勉強中 (ワッチョイ eae3-dGLa)
垢版 |
2021/09/10(金) 04:04:03.63ID:j5X2vKuZ0
ネイティブの感覚しらんから敬語もない気楽な言語だって質問フォーラムに英語で書き込みしてたら
I want to じゃなくて I would like to って言えってモデレーターに校正されたんだがI want to ってそんなにだめな表現なんかね
今までいろんなところで質問してきたがこんなんされたの初めてでカルチャーショックよ
0905名無しさん@英語勉強中 (ワッチョイ 69db-xJJl)
垢版 |
2021/09/10(金) 12:06:56.95ID:jO3IYKTf0
When you're all done making your food and eating your food, blah blah blah ...

このとき "making your food" の部分は文法的にどう説明されるんでしょうか
done の目的語のようにみえるけど、ここで done は形容詞なので目的語をとらんぽいし
かといって with を使って you're all done with making your food とは言わんぽいし
0917😏三年英太郎😉 ◆3CZBjOt3.Y (ワッチョイW ea86-8WL0)
垢版 |
2021/09/10(金) 13:18:49.42ID:/kxy62f50
914まちがい
be done doingは決して古い表現じゃないなりよー

アメドラ見てると頻出するし、むしろインフォーマルな場で使われていることが多い気がするなりね🤔確信はないけど
0919名無しさん@英語勉強中 (ワッチョイ 3d17-aIS6)
垢版 |
2021/09/10(金) 13:50:40.64ID:6V8bOj/a0
doneは過去分詞ではありえないので形容詞と考えるしかない。
突き詰めて言うとおかしいけどfinishedと同じ意味で慣用的に使われている。
文法的に説明するのは無理だと思うよ。英語は例外も多いので。
0920😏三年英太郎😉 ◆3CZBjOt3.Y (ワッチョイW ea86-8WL0)
垢版 |
2021/09/10(金) 14:16:40.20ID:/kxy62f50
be perfect は移動や変化を表す自動詞に起こった。歴史的に do には起こってない。現在は(決まり文句以外)廃用。

自動詞/他動詞の do にのみ起こる新用法の be perfectを想定するより、形容詞と見なしたほうがよっぽど簡単に説明できりじゃん🤔
0924名無しさん@英語勉強中 (ワッチョイ 253b-dgdc)
垢版 |
2021/09/10(金) 16:54:34.21ID:mHUyQupc0
>>905
「全部終わったら、つまりご飯を作って、食べたら…」
I'm doneみたいな感じのdoneでその後で補足で説明を加えているようにも読めなくもないですね
when you're all done, making your food and eating your food,...
文法的にオッケーなのかわかりませんが口でしゃべるならまあアリなんじゃないかな〜と考えました
0925名無しさん@英語勉強中 (ワッチョイ 3de2-xJJl)
垢版 |
2021/09/10(金) 17:07:37.09ID:hXKxK9fS0
Thank you. Is the tip included in this?

Yes, 17% gratuity is automatically included for parties of six or more.

これの
Yes, 17% gratuity is automatically included for parties of six or more.
の意味を教えてください

for parties of six or more.

が意味わかりません
0926名無しさん@英語勉強中 (ワッチョイ 3de2-xJJl)
垢版 |
2021/09/10(金) 17:09:15.99ID:hXKxK9fS0
17% gratuity is automatically included
17%のチップが自動的に含まれます

for parties of six or more. ← これがわかりません
0928名無しさん@英語勉強中 (ワッチョイ 6689-xJJl)
垢版 |
2021/09/10(金) 19:01:10.61ID:jd1L2+kk0
レストランなどでの客の数の数え方

a party of one --- おひとり様
a party of two --- お二人様
a party of five --- 5人様

parties of six --- 6人様のグループが複数
parties of six or more --- 6人のグループが複数ある場合
0930名無しさん@英語勉強中 (ワッチョイ 6689-xJJl)
垢版 |
2021/09/10(金) 19:20:17.63ID:jd1L2+kk0
>>928 訂正
parties of six or more --- 6人 ★以上★ のグループが複数ある場合
0931名無しさん@英語勉強中 (ワッチョイ 6689-xJJl)
垢版 |
2021/09/10(金) 19:29:36.81ID:jd1L2+kk0
(1) Party Of One (お一人さま) という題名の歌がある。Brandi Carlile という歌手が歌っている。

(2) Party of Five (5人さま) というタイトルのアメリカドラマが何十年も前に流行ったみたいだ。
けっこう美男美女で演技のうまい人たちがたくさん出演して、しかも内容の濃いドラマだった。

その他、"a party of..." とか "party of..." で検索すると、とても面白い例文が並ぶ。

レストランなどで "a party of two, three, etc." という言い回しを使うのは、少し古めかしいらしい。
でも現代でもときどきそのような表現を使う人がいるそうだ。俺自身は
英語圏に行ったことがないので、あんまり知らんけど。

When entering a restaurant, sometimes you are asked "A party of how many?" You would then answer, "A party of 2,3 or however many people."
https://www.usingenglish.com/forum/threads/89378-A-party-of-one
0933名無しさん@英語勉強中 (ワッチョイ 253b-xJJl)
垢版 |
2021/09/10(金) 21:24:27.25ID:mHUyQupc0
>>932
少なくともこの文を読む限りではそうもとれる気はしますし、「〜し終わった」ともとれますね(多分こっちの意味で取るのが普通なんだとはおもいますが)
done ingで検索するといろいろ質問している人がいるので、割と広まっている表現なのかもしれませんね
あえて文法的に考えるならbusy ingみたいな感じなんでしょうか
0935名無しさん@英語勉強中 (JP 0H3e-xJJl)
垢版 |
2021/09/11(土) 12:51:44.12ID:mHxTtduMH
紙面に記載する所属先の英語表記について質問があります。
所属する部門が2つあり、スラッシュで記載するように指示を受けています。
そういった場合、スラッシュの前後にはスペースありでしょうか、なしでしょうか?
Product Planning Division/Design Division, ×× cooporation
Product Planning Division / Design Division, ×× cooporation
基本はスラッシュの前後はスペースなしだと思っているのですが、2単語以上の場合はありと書いてあるものもあったので。
よろしくお願いします。
0936名無しさん@英語勉強中 (ワッチョイ 3d17-Le98)
垢版 |
2021/09/11(土) 15:09:58.97ID:EgCOWO6I0
>>935
正確に言うとその場合はスペースあり。
スペースなしだとDivisionとDesignという二つの単語の間に
スラッシュを入れて二つの単語を並列にしているのでおかしい。
Division/Designがひとかたまりになってしまう。
要するに読み手が下記のどちらかを選択するということになる
Product Planning Division Division, ×× cooporation
Product Planning Design Division, ×× cooporation
0937935 (JP 0H3e-xJJl)
垢版 |
2021/09/11(土) 15:25:19.50ID:mHxTtduMH
936さん、ありがとうございます!
丁寧なご説明をいただき、よくわかりました。
0938名無しさん@英語勉強中 (ワッチョイ 3d17-Le98)
垢版 |
2021/09/11(土) 15:39:59.73ID:EgCOWO6I0
>>937
綴りも気をつけてね。
cooporation
0939名無しさん@英語勉強中 (ワッチョイ 6689-xJJl)
垢版 |
2021/09/11(土) 15:49:10.39ID:Z/eK2OeC0
cooperation と書くべきところを cooporation と書いてしまっているところを見ると、
どうやら cooperation と corporation とをいつも混同しているみたいだな。
何で混同してしまうかというと、この、本来ならまったく発音が違う二つの単語を
同じように発音してしまっているから、綴りまで混同してしまうんだな。

俺の友人も同じだ。おそらく日本人の大多数がこの二つの単語を同じように
発音し、その結果として書く時もよく似た感じに書いてしまうんだろうな。

別にそういう人たちを変に批判するつもりはないんだけど、この二つを
正確に発音すれば、綴りだって間違うこともないんだけどな。
こういう綴りの間違いは、L と R を日本人が区別して発音できないから、
その結果として綴りまで間違ってしまうことに似ている。
0940名無しさん@英語勉強中 (ワッチョイ 6689-xJJl)
垢版 |
2021/09/11(土) 15:50:54.25ID:Z/eK2OeC0
しまった。今回は会社名だろうから、Corporation と書くべきところだったんだな。
勘違いしてすまん。まあ、いずれにしても似たような問題ではあるんだけど。
0941名無しさん@英語勉強中 (ワッチョイ 6689-xJJl)
垢版 |
2021/09/11(土) 15:54:01.84ID:Z/eK2OeC0
どうしてもカタカナ発音しかできないのなら、せめて

(1) corporation (会社) は  コーポレイション

(2) cooperation (協力) は  コゥアーポレイション

と発音すれば混同しなくて済むんだけどな。
0942名無しさん@英語勉強中 (ワッチョイ 6689-xJJl)
垢版 |
2021/09/11(土) 15:59:22.78ID:Z/eK2OeC0
さらに、語源を活用してこの二つの単語を覚えなおしたら、混同しにくくなる。

(1) corporation (会社) は、corpus (体, body) というラテン語から来る。団体とか仲間に
対して「実体」つまり body を与える (corporate, incorporate する) というプロセスを
経過すると、それは corporation (会社) になる。

(2) cooperation (協力) は、co-operate つまり co- (com, 共に) + operate (働く、作用する)
から来る。「共に働く」から co-operate (協力する) となる。operate をアメリカ式に
発音して「アーポレイト」みたいに発音すれば、上の (1) と混同しにくくなる。
0943名無しさん@英語勉強中 (ワッチョイ 6689-xJJl)
垢版 |
2021/09/11(土) 16:10:53.49ID:Z/eK2OeC0
>>942 に出てきた corpus (体、body) という単語にピンと来なかったら、
英語の corpse (死体) とかフランス語の corps (軍団) とかイタリア語の corpo (= body)
英語の法律用語である incorporeal (無形の、無体の) とか corporeal (有形の、有体の)
とか corpus (コーパス、言語資料) など、普通の人がよく知っているたくさんの
なじみ深い単語と語源が共通していることを考え合わせると、納得がいく。
0944名無しさん@英語勉強中 (アウアウウーT Sa21-kRcQ)
垢版 |
2021/09/11(土) 16:18:16.86ID:fYT/OiKUa
corporation (n.): mid-15c., corporacioun, "persons UNITED in a body for some purpose," from such use in Anglo-Latin....

incorporate: verb [transitive], to INCLUDE something as part of a group, system, plan etc
0945名無しさん@英語勉強中 (ワッチョイ 6689-xJJl)
垢版 |
2021/09/11(土) 16:56:53.84ID:Z/eK2OeC0
>>942 の、英語における corporation (法人、会社) の直接の語源は中世フランス語
(ancient francais, Old French) だろうけど、それはもちろんラテン語から来る。
ラテン語辞典を見ると、

corporatio, corporationis
の意味は、physical makeup, build つまり「「(物の)物理的な作り」というような
ことが書いてある。(Oxford Latin Dictionary より)

corporatio (corporationis) というラテン語はもちろん、corporo (corporare) という
ラテン語の動詞から派生しているが、これについては

(1) to strike dead, to kill
(2) to form into a body, furnish with a body
(3) to form (a corporate society)

と書いている。(Oxford Latin Dictionary より)

つまり、corporare というラテン語はもともと「物に実体(体)を与える、形成する」
というところから来ていて、それが「会社を形成する」という意味へと派生してきた
わけだ。だからあくまでも、corporation という英語もフランス語も、そのもとである
ラテン語の corporatio (corporationis) にしても、「body (実体、形、体) を与える」と
いうもともとの意味から派生したものだ。そしてその body に相当するラテン語こそ、
corpus というわけだ。
0946名無しさん@英語勉強中 (アウアウウーT Sa21-fIv3)
垢版 |
2021/09/11(土) 16:58:51.19ID:wByvpAUda
The Financial Timesではco-operationと表記される(その他もpre-eminentなどと接頭辞を分岐して表記されてるからわかりやすい)
普通に英語に接していれば会社名はCorp.と略されるのを山ほど見るはずだから、両者を混同することはまずないはずだがな...
0947名無しさん@英語勉強中 (ワッチョイ 6689-xJJl)
垢版 |
2021/09/11(土) 17:06:36.42ID:Z/eK2OeC0
corporation を今度は OED (Second Edition) で見ると、次のように書いている。

corporation-em <-- corporare (to embody)

つまり、corporatio (corporationem) を to embody という意味で使っていたというわけ。
そして medieval Anglo Latin (中世イングランドで使われていたラテン語) においては、
この単語を次のような意味で使っていた、と書いている、

A number of persons united, or regarded as united, in one body; a body of persons.

ここでも body (corpus というラテン語の英訳) という言葉をしきりに使っている。
物に対して body を与えたら、当然のことながらそれは実体化され、組織になり、
a number of persons united つまり「団結した人の集まり」となる。
0948😏三年英太郎😉 ◆3CZBjOt3.Y (ワッチョイW ea86-8WL0)
垢版 |
2021/09/11(土) 18:43:50.61ID:8ijBVeTt0
この語源で「はしゃいでる」おじいちゃんの過去発言↓

********引用*******
俺としては、別にこんなものどうでもいい。
aged 32 が age 32 に訛っただけなのか、her age being 32 が簡略化されたのか
なんて、どいうでもいいことだ。問題は、A woman, age 32, was.... という
言い回しが厳然と存在していて、自由自在にそれを使いこなせたらそれでいいのだ。
0950名無しさん@英語勉強中 (ワッチョイ 6689-xJJl)
垢版 |
2021/09/11(土) 19:48:09.39ID:Z/eK2OeC0
>>949
言葉の長さは、syllables の数で決まると思う。

(1) fourteen thousand five hundred twenty yen
10 syllables

(2) 1万4千5ひゃく20えん
itch-man-yon-sen-go-hyaku-niju-en
10 syllables

というわけで、両方とも 10 syllables で引き分けだから、同じ長さに感じられると思う。
ただし、日本語の「いち」は itch のように発音することが多く、「ち」の部分は
単なる子音だけであり、母音は伴わないと思うので、「いち」を one syllable として
数えた。

日本人が英語を長く感じるのは、単に発音が悪いからであって、英語ネイティブに
近く発音すれば、実際には短いと感じられるはずだ。
0951名無しさん@英語勉強中 (ワッチョイ 6689-xJJl)
垢版 |
2021/09/11(土) 20:00:05.80ID:Z/eK2OeC0
このスレでずっと前に、"decide to do" という意味での "determine to do" つまり
自動詞としての determine (決心する) という言葉が重い意味なのではないか、という
問題提起があった。今ごろになって、俺のたまたま読んでいる小説にそれが出てきたので、
紹介する。

... and when the second day was ended, he (つまり Lord Warburton) ★determined★ to postpone
his departure till the morrow.
(Henry James, The Portrait of a Lady, Library of America LoA #29, p. 257)

ここに出てくる Lord Warburton は、客としてしょっちゅうこの家 (Mr. Touchette の家)
に来ているが、この小説の主人公 Isabel があまりに魅力ある知的かつ活発な若い女性
なので、それに魅了され、ついつい2日だけじゃなくてさらにもっと泊まっていく
ことにした (determine した) と書いている。ここでは、単に客としてその家に
「泊まることにした」と言っているだけであって、深刻な、あるいは重大なことを
「決心、決断している」という文脈ではない。だからこの "determine to 動詞" という
時の自動詞としての determine は、現代英語によくある単なる decide to do とか
make up one's mind to do くらいの意味しかないと思う。

なお、この小説は 1881 年に出版されたものなので、現代よりも 140 年も前のものだから、
英語は少しだけ古めかしい。俺が思うには、この時代には現代から見れば大げさな
単語を使うことが多かったので、ここでも decide to do とか make up one's mind to do
じゃなくて、determine to do という、syllable 数の多い、少し重々しい感じのする単語を
好んで使う人が多かったのではないかという気がする。
0952😏三年英太郎😉 ◆3CZBjOt3.Y (ワッチョイW ea86-8WL0)
垢版 |
2021/09/11(土) 21:03:48.00ID:8ijBVeTt0
言葉の長さは音節数で決まる

珍説のたれ流し

schwagerとアニ
Demeter とメガミ
Boccaccio とボカロ
発音時間も綴りの長さも違う

/イチ/ は単独・語末、または無声子音が後続する場合にうしろの/i/が無声化する。理由もある(音声学の超初歩)。/ichiman/の時はふつうは無声化しない
0953😏三年英太郎😉 ◆3CZBjOt3.Y (ワッチョイW ea86-8WL0)
垢版 |
2021/09/11(土) 21:14:00.46ID:8ijBVeTt0
demeter
デーメーテーr

特に詩の朗読の時は長母音が長く発音されたそうである

***********

イチマンについて自分で発音してみりゃいい話で、/chi/を無声化するとすごーく言いづらいのである。(超ゆっくりなら無声化する。なぜって単独で発音してると同じだから
0955三年英太郎 ◆3CZBjOt3.Y (ワッチョイ ea86-m40g)
垢版 |
2021/09/12(日) 00:42:31.81ID:siReIF2F0
わたくしの結論としては、音声言語としての経済性では
日本語のチャイナ語系数詞の方が、英語より勝ると思われるので、
相対的にはめんどくさいのではないだろうか。

*****************************

余談だが、ドイツ語で数を数える時、一部がアラビヤ数字の並びのままではない。
153なら、「100、3と50」みたいな感じ。

ドイツで障害者学校の先生とデートしてたとき、たまたまこの話題になったのが、
知的障碍者にこの数字と数(の読み)の関係を教えるのがすごく難しいって言ってた。
人間の自然な思考に逆らっている以上、ドイツ語はよりめんどくさいと言える。
0958名無しさん@英語勉強中 (ワッチョイ 6689-xJJl)
垢版 |
2021/09/12(日) 06:59:39.56ID:nxvu/lPe0
>>951 で取り上げた Henry James に触れるのは、俺としては今回が初めてだ。
modernist (モダニズム文学者) として有名で、かなり洋書を読みなれているらしい人でも
Henry James は読みにくくてたまらんと言っていたので、俺なんかには死ぬまで
まともに読める日は来ないだろうと思って、敬遠していた。しかし何となく
読んでみようと思い立って読み始めてみたら、原文ではほんの 60 ページ
(もし日本語版の文庫本なら 110 ページくらい)しか読んでいないけど、
丁寧にゆっくり読みさえすれば、まあまあ楽しく読める。

というか、文章があまりに緻密で仰天する。人物の性格や仕草などの
描写が細かいのなんの。Victor Hugo を読んでた時もそんな感じがした。
フランスやドイツやロシアには、長大な作品を見事な筆力で書ききる人が
多いみたいだが、英語圏には少ないんじゃないかと思っていた。英語圏といえば
どっちかというと短い作品が多いと思ってたのだ。

しかし Henry James は違う。
ものすごく緻密で、描写が極めて細かく、一つ一つのセンテンスが長く、
パラグラフは一つ一つが2ページも3ページも続くから、それぞれのページには
字がぎっしりと詰まった感じがするので息が詰まりそうになるが、
それなのに読者を疲れさせないのは、やはり天才的な文章力によるのだろう。
やはり英語圏で最大級の文学者と言われるだけのことはある。
https://www.rem.routledge.com/articles/james-henry-1843-1916
0959名無しさん@英語勉強中 (ワッチョイ 6689-xJJl)
垢版 |
2021/09/12(日) 06:59:55.72ID:nxvu/lPe0
modernists と言えば、Virginia Woolf と Samuel Beckett を数年前にかなり真面目に読んで
いた。Joseph Conrad は特に読みにくかったけど、1作品だけ読んだ。よく言われることだが、
modernist literature というのは、確かに読みにくい。でも、それを書く人たちは
もしもわかりやすい文章を書く必要のあるときには、天下一品にわかりやすい文章を
書く。Virginia Woof の主たる作品は本格的な modernist のものなのでわかりにくいけど、
彼女が書くごく普通の essays なんて、他のあらゆる journalists の文章を超えるわかりやすさだ。
ああいうわかりやすい文章を書くだけの能力があるからこそ、あえてわかりにくい文章を
書いてでも新しい世界を描き出そうと試みながらも、読者から受け入れられるのだろう。

Henry James は、その中でも特にわかりやすい文章の書ける人なんだろうと思う。だからこそ、
あんなに緻密で長大な文章を書き続けていても、読者を疲れさせないのだ。
とはいえ、まだこの人の作品を読み始めて間がないから、そのうちに俺も疲れてしまう
かもしれないが、ともかく頑張る。
0961😏三年英太郎😉 ◆3CZBjOt3.Y (ワッチョイW ea86-8WL0)
垢版 |
2021/09/12(日) 13:25:40.71ID:siReIF2F0
チクタク チクタク チクタク チクタク
マクドナ マクドナ マクドナ マクドナ

長文先生によると前者は2音節、後者は4音節で、長さが2倍になるはずだが、実際に声に出してそうなるだろうか?(早口言葉ではないよ)

たぶん、それほど変わりないはず。それは日本語ネイティブの頭の中には拍(mora)の感覚があるからである。母音無声化したら、母音がぬけた分、それに比例して発音時間が短くなるわけではない。

自称、言語学をお勉強なすってる学者気質の方が、母語さえ満足に観察できてないとは🤦🏻‍♂
0967名無しさん@英語勉強中 (ワッチョイ 6a33-m40g)
垢版 |
2021/09/12(日) 16:09:52.61ID:h80SWmcI0
Evergreenはいい文法書ですか?

これ1冊読んでおけば大体OKですか?
0969名無しさん@英語勉強中 (ワッチョイ 6a33-m40g)
垢版 |
2021/09/12(日) 16:15:37.37ID:h80SWmcI0
Evergreenを読んでいますが、すべて記憶しておく必要がありますか?

例えば、2つ目的語を取らない動詞(introduceなど)がいくつか紹介されていますが、こういうのはすべて記憶しておくべきですか?

それとも、英文を読んでいくうちに自然とintroduce O1 O2というパターンは出てこないなと身につけていくものですか?
0970名無しさん@英語勉強中 (ワッチョイW 3991-1Y4m)
垢版 |
2021/09/12(日) 16:22:25.63ID:nQALD2K10
>>969
実際動詞の語法なんかは規則を暗記しても使えるようにはならなくて、使っていくうちに正しい用法を覚えていく物なので。
あなたが受験するのであれば穴埋め4択クイズに正解するために覚える必要あるかもしれない。
evergreenはあのforestの後継なので普通にいい参考書
0974名無しさん@英語勉強中 (ワッチョイ 6689-xJJl)
垢版 |
2021/09/12(日) 18:55:48.68ID:nxvu/lPe0
but と言っても、「しかし」とか「でも」という意味ではなさそうに見える but が
ときどき出てくる。それについてこのスレでも、少なくとも一回は出てきた。
そういう、but らしくない but もあるのだといくら説明しても、質問者は
納得しなかったけど、納得させるための用例を検索しようとしてもなかなか
出てこなかった。実際に小説などを読んでいるとたまに出てくるんだが、
いざ研削しようとするとなかなか出てこない。今たまたま読んでいる小説の中で
出てきたので、紹介しておく。

She (= Isabel) found herself liking him (= Lord Warburton) extremely; the first impression he had made upon her
was pleasant, ★but★ at the end of an evening spent in his society she thought him
quite one of the most delectable persons she had met.
(Henry James, The Portrait of a Lady, Library of America LoA #29, p. 257; Chapter VII)
0975😏三年英太郎😉 ◆3CZBjOt3.Y (ワッチョイW ea86-8WL0)
垢版 |
2021/09/12(日) 18:58:09.08ID:siReIF2F0
チクタク チクタク チクタク
ティックトック ティックトック ティックトック

どっちも2音節だが、前者は4拍、後者は6拍だから、普通の日本人なら後者の方が長くなる🤔
0977名無しさん@英語勉強中 (アウアウキー Sa55-UBEE)
垢版 |
2021/09/12(日) 19:05:38.71ID:3GMIqIbVa
>>963
ある意味コンプはある
期待させんじゃねーよという意味で
英語できなくても簡単に偏差値70とか出てしまうから変に勘違いしてしまう
で、調子に乗ってチャットとか始めるとボコボコにされて日本の偏差値70は世界の偏差値40ということに気づかされるんよ
0978名無しさん@英語勉強中 (ワッチョイ 7992-kRcQ)
垢版 |
2021/09/12(日) 19:32:29.78ID:TVK4rx350
受験コンプレックスがあるから、高校の数学とか物理の参考書を読もうと
思ってる。今日、物理のめちゃくちゃ入門の本を買ってきた。
いうか本屋に行ったら面白そうなので買った
0979😏三年英太郎😉 ◆3CZBjOt3.Y (テテンテンテン MM3e-8WL0)
垢版 |
2021/09/12(日) 20:28:42.15ID:5Q/6yUGlM
give は「誰に」「何を」あげるか、意味的に2種類の名詞(相当句)を必然的に要求する

introduceもそうなのに、SVOOの形は取らない🤔わりと大事じゃない?🤔

SVOOの動詞は、ゲルマン語系の2音節のものが多いと覚えておくと、記憶の負担が減る(例外はあるけど、かなり少ない)✌🏻
0987名無しさん@英語勉強中 (ワッチョイW 2aa7-mq+u)
垢版 |
2021/09/13(月) 04:22:56.66ID:YIYV2UF40
>>981
なんの参考書ですか?
三年さんの答えであってるとおもう。
the world most powerful gossip column
がit’sぼそのままの抜き出しで、
that world の部分は読者に想像させてる。a billionaire を
こき下ろそうとしてたから、というのと後でcelebrity culture
とあるからcelebrity の世界

キクタンか何かで覚えるべき赤字単語は易しいのに記事は
it’sとthe world で考えさせたり、不思議な参考書だなと思った
0988名無しさん@英語勉強中 (ワッチョイ 6689-xJJl)
垢版 |
2021/09/13(月) 06:00:09.00ID:xREj3tV00
>>980
She got them big ass titties

>>986 の言う通りで、この them は those という意味の非標準的な英語だな。
通常の英和辞典の them の項目にもそう書いてある。

She got them big ass titties
だからこれは、She's got (= She has) those big ass titties. という意味だ。それから
ass titties は ass and titties の間違いかと思ったが、実はここでは「ケツみたいなオッパイ」
あるいはその反対で「オッパイみたいなケツ」という感じで、通常はものすごく
太った女のことを悪く言うときに使うことが多いみたいだけど、ここでは、歌詞の
文脈から考えたら誉め言葉だということは明らかだ。ネット上の Urban Dictionary で
ass titties のいろんな意味合いを調べてみたら、そういう気がした。
0989名無しさん@英語勉強中 (ワッチョイ 6689-xJJl)
垢版 |
2021/09/13(月) 06:04:44.32ID:xREj3tV00
>>980
"Anime Thighs" っていうその歌の歌詞を全部読んでみた。スラングだらけで完全には
理解できないけど、すげえことばかり言ってる歌だな。日本のアニメが、通常のものも
erotic なものも含めて海外でもてはやされていることは知っていたけど、こんな歌まで
作って、しかもその歌が流行っているらしいところを見ると、これはもう本格的だ。
0991名無しさん@英語勉強中 (ワッチョイW 2aa7-mq+u)
垢版 |
2021/09/13(月) 08:34:29.16ID:YIYV2UF40
時制ついての質問ですが、

今の時点が今日だとします。
昨日次のように言った。
明後日雨が降るだろう

この時に
He said that it will fine tommow.
といっても文法的には正しいでしょうか?
つまり、saidだからwouldにするという時制の一致をせずに、
the day after tommowじゃなくて、tomorrowにする。
(今日になって昨日から一日経って、明日までの日にちが二日から一日縮まったものの
雨が降ると予想された日まではまだ一日あるから。
willのままにするのは、今日の時点で明日の天気のことを言っているので。

よろしくお願いします
0992名無しさん@英語勉強中 (ワッチョイ 6689-xJJl)
垢版 |
2021/09/13(月) 08:58:12.87ID:xREj3tV00
>>990
big ass, big-ass, bigass という形容詞について教えてくれてありがとう。

a bigass truck
a big-ass boat
big-ass legs (big ass legs)
a big ass party

こういう表現も出回っていて、おもしろいと思った。
0993名無しさん@英語勉強中 (ワッチョイ 253b-xJJl)
垢版 |
2021/09/13(月) 09:18:59.97ID:FyM1OcMI0
>>991
https://www.englishcafe.jp/tense2/1-2-2.html
数日前にこれを読んだのを思い出しました
He says it will rain tomorrow.としたらスッキリすると思います(the tomorrowとするべきかday after tomorrowとするかは微妙ですが、なんとなくtomorrowのほうが適切かと思いました)

>a. Joan tells me you're getting a new car.
>(新しい車を買うそうですね、ジョーンが私に話していましたよ。)
>b. The ten o'clock news says that it's going to be cold.
>(10時のニュースが言うには、寒くなるみたいですよ。)
>(以上2例はLeech:1971a)
>上の2例において、いずれも現在時制の伝達動詞を用いて過去に聞いたことを表している。それぞれは現在においてもいまだにその内容が真実のこととして捉えている。
0994名無しさん@英語勉強中 (ワッチョイ 6689-xJJl)
垢版 |
2021/09/13(月) 09:24:46.85ID:xREj3tV00
>>991
そういうことは、実際の英語ネイティブがしゃべっている話や小説やドラマの
中での会話などをたくさん見聞きしないとわからないもんだと思う。

ただ、日本語ではどうかというと、日本人同士でもこんがらがるから、文法的にどうである
かはともかく、少しくどいくらいに説明しなおしながらしゃべるんじゃないかと思う。
少なくとも俺は、日本語でしゃべるとき、誤解のないように何度も確認する。

だから英語でもおそらく、次のようにでも言う方が気が利いていると思う。
なお、雨が降る件についてではなくて、もっと現実的に、昨日、Ted が
「明後日、Nancy のところに会いに行くつもり」と言ったとする。

Ted said yesterday, "I'm going to go to see Nancy the day after tomorrow."

ややこしいので、これを俺だったら次のように言う。

Yesterday Ted said, "I'm going to go to see Nancy the day after tomorrow."
If he hasn't changed his mind, he'll be seeing Nancy tomorrow.

上の一行目を無理に間接話法なんかには、それはしない。
0995名無しさん@英語勉強中 (ワッチョイ 6689-xJJl)
垢版 |
2021/09/13(月) 09:24:58.06ID:xREj3tV00
さらに、もっとわかりやすく
するため、the day after tomorrow なんていう言い回しはなるべくやめて、
October 7 とかいうふうに言う。もしも Ted 自身は the day after tomorrow と
口頭で言ったのなら、次のように言う。

Ted said yesterday, "I'm going to go to see Nancy the day after tomorrow."
He meant October 7. So, if he hasn't changed his mind,
he'll be seeing Nancy tomorrow, that is, on October 7.

くどいかもしれないが、こうすれば絶対に誤解は生じない。特にビジネス上の
メールや電話だったら、これくらいにくどい方がいいと思う。

ただ、今回の質問者の質問は、そういう問題についてではなくて、あくまで
質問者がもともと提示した英文が文法的に正しいかどうかだけに焦点を当てている
ことも熟知している。しかし俺の言いたいのは、文法的に仮に正しいことを言ってても、
相手にちゃんと伝わるかどうか、変な誤解が後で生じることがないかどうかは
保証できないから注意しないといけないということだ。
0996名無しさん@英語勉強中 (ワッチョイW 2aa7-mq+u)
垢版 |
2021/09/13(月) 11:17:04.82ID:YIYV2UF40
>>993
>>994
ありがとうございます。

PEUに次のように書いていました。

2 reporting present and future tenses

If somebody talked about a situation that has still not changed – that is to say, if the original speaker’s present and future are still present and future – a reporter can often choose whether to keep the original speaker’s tenses or to change them, after a past reporting verb. Both structures are common.
– DIRECT:
The earth goes round the sun.
INDIRECT:
He proved that the earth goes/went round the sun.
– DIRECT:
How old are you?
INDIRECT:
Are you deaf? I asked how old you are/were.
– DIRECT:
It will be windy tomorrow.
INDIRECT:
The forecast said it will/would be windy tomorrow.
We are more likely to change the original speaker’s tenses if we do not agree with what he/she said, if we are not certain of its truth, or if we wish to make it clear that the information comes from the original speaker, not from ourselves.
The Greeks thought that the sun went round the earth. (NOT … that the sun goes round the earth.)
She just said she was thirty! I don’t believe her for a moment.
He announced that profits were higher than forecast.

The forecast said it will…
の例があるからwillでもいいし、would でもいいということでしょうね
0997名無しさん@英語勉強中 (ワッチョイW 2aa7-mq+u)
垢版 |
2021/09/13(月) 11:18:41.62ID:YIYV2UF40
他方で次のように上に書いたものと矛盾したことも書いています。

I told them I was British.
After past reporting verbs, we usually change the original tenses even if the things the original speaker said are still true.
– DIRECT:
I’m British.
INDIRECT:
I told the police I was British. (The speaker is still British.)
– DIRECT:
You can use my car today.
INDIRECT:
Your mother said I could use her car today. Have you got the keys?
– DIRECT:
How old are you?
0999名無しさん@英語勉強中 (ワッチョイW 2aa7-mq+u)
垢版 |
2021/09/13(月) 11:49:03.83ID:YIYV2UF40
>>993
He says yesterday it will fine tomorrow.
という風にreportする事柄の中にyesterdayを入れた時に
果たして教えていただいた方法でいけるのかなと疑問が残るのですがいかがでしょう?

>>994
言い換えたものを付け足すということで誤解がは生じなくなりますよね。
10011001
垢版 |
Over 1000Thread
このスレッドは1000を超えました。
新しいスレッドを立ててください。
life time: 51日 23時間 23分 34秒
10021002
垢版 |
Over 1000Thread
5ちゃんねるの運営はプレミアム会員の皆さまに支えられています。
運営にご協力お願いいたします。


───────────────────
《プレミアム会員の主な特典》
★ 5ちゃんねる専用ブラウザからの広告除去
★ 5ちゃんねるの過去ログを取得
★ 書き込み規制の緩和
───────────────────

会員登録には個人情報は一切必要ありません。
月300円から匿名でご購入いただけます。

▼ プレミアム会員登録はこちら ▼
https://premium.5ch.net/

▼ 浪人ログインはこちら ▼
https://login.5ch.net/login.php
レス数が1000を超えています。これ以上書き込みはできません。

ニューススポーツなんでも実況